Wikipedia:Auskunft/Archiv/2016/Woche 01

aus Wikipedia, der freien Enzyklopädie
Zur Navigation springen Zur Suche springen


Dies ist ein Archiv der Seite Wikipedia:Auskunft. Es enthält alle Abschnitte, die in der Kalender-Woche 01 im Jahr 2016 begonnen wurden.

Möchtest du in einer neuen Diskussion auf dieses Archiv verweisen? Nutze folgenden Link und ersetze ________ durch den Namen des Abschnittes. Die Nummer im Inhaltsverzeichnis gehört nicht dazu:

[[Wikipedia:Auskunft/Archiv/2016/Woche 01#________]]


← vorherige Woche Gesamtarchiv nächste Woche →

Tag der offenen Tür der Bundesregierung 2016

Steht der Termin für den Tag der offenen Tür der Bundesregierung 2016 schon fest?

--85.180.136.4 04:53, 4. Jan. 2016 (CET)

Nein, 2015 wurde der Termin auch erst am 21. Jan angekündigt. --Vsop (Diskussion) 05:53, 4. Jan. 2016 (CET)
Archivierung dieses Abschnittes wurde gewünscht von: Vsop (Diskussion) 15:19, 4. Jan. 2016 (CET)

Wenn man aus einem Auto mit Sitzen aus Kunststoff steigt, bekommt man an der Tür häufig einen elektrischen Schlag. Erkläre diese Erscheinung.

--91.42.144.122 13:56, 4. Jan. 2016 (CET)

Nanu, hat die Schule schon wieder begonnen? --Magnus (Diskussion) für Neulinge 14:00, 4. Jan. 2016 (CET)
Elektrostatische Entladung? --Kapitän Turnstange ¯\_(ツ)_/¯ 14:15, 4. Jan. 2016 (CET)
Angel-Lektion: Gibt man bei Google Wenn man aus einem Auto mit Sitzen aus Kunststoff steigt, bekommt man an der Tür häufig einen elektrischen Schlag ein, trifft schlagartig einen der Schlag! Play It Again, SPAM (Diskussion) 14:17, 4. Jan. 2016 (CET)

Hier ist die Lösung: Lösung 90.184.23.200 18:52, 4. Jan. 2016 (CET)

Alles Quatsch, das kommt durch Kriechströme, die sich aus der Restladung der Autobatterie durch die metallische Karosserie bis zu den positiv geladenen Kunststoffsitzen ausbreiten, wo dann durch eine elektromechanische Kopplung ein Schlagmechanismus ausgelöst wird. --88.68.80.100 19:28, 4. Jan. 2016 (CET)
Unsinn. Es liegt daran, daß die Elektronen während der Fahrt auch etwas sehen wollen und sich deshalb Fensterplätze suchen. Beim Aussteigen wird ihnen kalt, deshalb springen sie schnell ins Auto zurück und warten dort auf die nächste Fahrt. --Kreuzschnabel 21:38, 4. Jan. 2016 (CET)
Das Marderabwehrgerät des Autos ist falsch montiert oder eingestellt. --Rôtkæppchen₆₈ 00:14, 5. Jan. 2016 (CET)
+1 – Unsinn, siehe Elektrostatik und früher montierten Eigentümer ihrem fahrbaren Untersatz ein Schwänzchen.[1] Mit geeignetem Material geht das aber auch ohne. Opel rief vor langer Zeit eine Serie Fahrzeuge, die heute schon weitgehend in der Schrottpresse gelandet sind, zurück um am Tankstutzen die Abdeckung oder ähnliche Teile auszutauschen. Es sollte der Potentialausgleich zu Zapfsäule langsam stattfinden, um Funkenbildung zu vermeiden. Das ausgetauschte Teil bestand aus einem Material, das einen definierten elektrischen Widerstand hat. Dabei wird Ladung gezielt langsam abgebaut. Material mit dieser Eigenschaft wird auch zu Verpackung elektronischer Bauteile benutzt, um zerstörerisch hohe Ströme durch die Halbleiter zu vermeiden, die durch elektrostatische Entladung, auftreten. Dabei fällt die Spannung auf dem Material ab und die Ströme bleiben so klein, dass keinen Nennenswerten Spannungen auf den Verpackten Bauteilen abfällt. Das Material der Innenverkleidung und den Sitzen von Autos wird heute ebenso ausgesucht, um die Entladung beim Aussteigen zu verlangsamen. Dabei fällte die Spannung auf dem Material ab und findet nur langsam über über den ersten Mensch statt, der aus dem Auto aussteigt. --Hans Haase (有问题吗) 06:22, 5. Jan. 2016 (CET)
Archivierung dieses Abschnittes wurde gewünscht von: Frage beantwortet. --88.68.80.100 19:28, 4. Jan. 2016 (CET)

Goethe in der U-Bahn

In der New Yorker U-Bahnstation „42nd Street – Bryant Park“ steht ein Zitat von Goethe auf einer Wand:

The unnatural - that too is natural.

Woher kommt dieses Zitat? Ich finde leider nur unzählige englischsprachige Websites, die dieses Zitat ohne Quellenangabe verwenden. --85.180.136.216 02:56, 5. Jan. 2016 (CET)

Goethe war ein deutscher Dichter, darum googlen mit unnatürlich natürlich goethe - ergibt auf Seite eins schon zwei nette Treffer: [2], [3]. --188.107.142.219 03:04, 5. Jan. 2016 (CET)
Der erste Link scheint es zu treffen. Danke. Ich hatte zu wörtlich nach "Das Unnatürliche, das ist auch natürlich" gesucht... --85.180.136.216 04:12, 5. Jan. 2016 (CET)
Das erste („Sehe ich Dich in Bewegung, so erwache ich vom tiefsten Schlafe, der manchmal ganz unnatürlich natürlich ist.“) ist aber gar nicht Goethe, sondern Zelter an Goethe und das zweite („eine aus Wahrheit und Aberglauben dunkel gemischte Vorstellung, die dem Griechen natürlich war, uns unnatürlich ist“) Friedrich Theodor Vischer über Goethes Faust.  --Vsop (Diskussion) 11:57, 5. Jan. 2016 (CET)
<seufz> Irgendwie wirds schon irgendwo passen...
Nein! Ich verwette meinen Morgenkaffee, dass es sich um Goethe-Zitat-Napping handelt (= irgend etwas aus dem Zusammenhang gerissen, umgestoppelt, übersetzt und dann für die eigenen Ansichten verwendet: „Kuckma, Goethe sacht das auch!“).
Man findet auch:
  • "Natürlich wars, aber so natürlich dass Sie's unnatürlich auslegen mussten." <= Das ist belegt. Das hat er einer Dame geschrieben.
Das engl. "Zitat" ist auch mit GoogleBooks nicht festzunageln.
Man könnte jetzt nachsehen, wann das engl. Zitat zum ersten Mal und wo auftauchte. Dann hätte man den "Zitatnapper" gefunden und könnte ihm entgegenschleudern "Das also ist des Mopses Füllung!" <snort!> Play It Again, SPAM (Diskussion) 08:49, 5. Jan. 2016 (CET)
So: Hier liegt das Kaninchem im Salz:
Education of the Visually Handicapped: The Official ... (1986) "... In a similar vein, Goethe stated, "The unnatural, that too is natural." George Bernard Shaw wrote, "Pretending that certain ways of thinking or acting are unnatural is a conspiracy to convict ..."
Nach meiner Suche die allererste Erwähnung des exakten Spruches. Und siehe: Goethe ist nicht genug! George Bernhard The Brain Shaw wird auch noch „zertiert“.
Sucht man nach ""Pretending that certain ways of thinking or acting" kommt man auf 1967 und da ist es kein Zitat, sondern eine Interpretation....
(Literarischen) Tod allen Zitatnappern ! Play It Again, SPAM (Diskussion) 09:04, 5. Jan. 2016 (CET)
Wer sowas macht, sollte Goethes Faust mal so richtig zu spüren bekommen. --Expressis verbis (Diskussion) 09:34, 5. Jan. 2016 (CET)
Macht mal langsam mit dem Napper-Bashing! Es gibt ein Fragment von 1780 namens "Die Natur", das weithin Goethen zugerechnet wird. Dort herinnen heißt es: "Auch das Unnatürlichste ist Natur, auch die plumpste Philisterei hat etwas von ihrem Genie. Wer sie nicht allenthalben sieht, sieht sie nirgendwo recht." (Goethes Werke, Hamburger Ausgabe in 14 Bänden, dtv, 1998, Band 13, Naturwissenschaftliche Schriften I, S. 45ff.) - Wenn überhauipt Napperei, dann eventuell vom Meister selbst, der sich "irrtümlich" die Autorenschaft zurechnete, wiewohl der Text tatsächlich von einem Herrn namens Christof Tobler verfasst und von Goethe nur mit handschriftlichen Änderungen versehen worden war. So wird es jedenfalls auf dieser Seite behauptet. --Joyborg 11:46, 5. Jan. 2016 (CET) PS: Und so scheint es auch zu stimmen: [4]. Ob die New Yorker U-Bahnverantwortlichen wissen, dass sie aus dem doch ziemlich großen Fundus von Goethe-Zitaten ausgerechnet eines ausgewählt haben, das nur halb von ihm stammt, weiß man nicht. Aber dass SPAM seinen Morgenkaffee ziemlich leichfertig verwettet hat, das steht fest. 2.PS: Als englische Übersetzung findet man im Internetz häufig "That which is most unnatural is still Nature" - vielleicht war das zu lang für die vorbeifahrenden New Yorker.
Georg Christoph Tobler. Die Natur (Essay). --Vsop (Diskussion) 12:54, 5. Jan. 2016 (CET)
Super, seit der BKL- und Weiterleitungslöschung finden es nichit einmal mehr die Wissenden. Brauchen wir nun Google um unsere Artikel wieder zu finden? --Hans Haase (有问题吗) 13:06, 5. Jan. 2016 (CET)
Der Kaffee ist jetzt kalt (oder noch schlimmer) - beides gebe ich gerne ab.
Aber da werden (a) Worte, die nicht von Goethe stammen, (b) übersetzt umgeschrieben (Natur = natürlich, das Unnatürlichste = das Unnatürliche), (c) aus dem Zusammenhang gerissen und (d) der zweite Teil des Satzes wird weggelassen. Das wird dann als Zitat ausgegeben. Mein Deutschlehrer hätte mich dafür lächelnd erwürgt.
Das ist so, als ob wörtlich übernommene Textstellen aus Dissertationen leicht umgestellt und verkürzt und dann dem Plagiator zugeordnet werden. Vielleicht ist das mit "unbegrenzte Möglichkeiten" gemeint? Die Leute sind jedenfalls begeistert davon. Darauf kommts wohl an ("Carl Jung, James Joyce, Mother Goose, and Johann von Goethe. What a great combination!"). Lollis für Gehirn. Wenigstens das Jung-Zitat ist in Ordnung (aber auch wieder aus dem Zusammenhang gerissen). Play It Again, SPAM (Diskussion) 14:00, 5. Jan. 2016 (CET)
Archivierung dieses Abschnittes wurde gewünscht von: ... oder? Also der Zusammenhang natürlich <=> unnatürlich <=> JWvG.
Merke: The unGoethe - that too is Goethe ... Play It Again, SPAM (Diskussion) 17:13, 5. Jan. 2016 (CET)
Ich glaube, ich erkenne jetzt, wie die Welt von innen zusammengetackert wurde. (Oh, neues Goethe-Zitat?) --Optimum (Diskussion) 17:52, 5. Jan. 2016 (CET)
Ich war der Fragesteller und möchte mich für die Recherche recht herzlich bedanken! --85.180.138.49 00:44, 6. Jan. 2016 (CET)

was bedeutet dual bei diesem system

--84.136.219.183 14:13, 5. Jan. 2016 (CET)

Bei welchem System? --MrBurns (Diskussion) 14:17, 5. Jan. 2016 (CET)
vielleicht ist Duales System gemeint? --80.133.235.86 14:25, 5. Jan. 2016 (CET)
Vielleicht ist Dualsystem gemeint. --Rôtkæppchen₆₈ 14:35, 5. Jan. 2016 (CET)
Oder dieses (Ausbildungs-) System?--Wikiseidank (Diskussion) 14:49, 5. Jan. 2016 (CET)
Ein System, das bei Dual-Plattenspielern eingesetzt wird?--Giftzwerg 88 (Diskussion) 15:04, 5. Jan. 2016 (CET)
Oder das Duale Runkfunksystem. Eines können wir ausschließen: Das System, Groß- und Kleinbuchstaben (2 Mengen → dual) sinnvoll zu mischen, kann nicht gemeint sein. --Kreuzschnabel 16:02, 5. Jan. 2016 (CET)
WeNN MaN WiLL, GeHT aLLeS. --RôTKæPPCHeN₆₈ 16:09, 5. JaN. 2016 (CeT)
Klar. Ich schloß das aus der Art der Fragestellung. --Kreuzschnabel 16:11, 5. Jan. 2016 (CET)
Ein Bier, ein Korn.--2003:75:AF0C:5100:4C66:6EE7:138B:944D 16:47, 5. Jan. 2016 (CET)
Vielleicht auch: Mal verliert man, mal gewinnen die anderen. --Kreuzschnabel 17:02, 5. Jan. 2016 (CET)
Irgendwas mit doppelt - oder zwei/zwier..?!? Luther! Play It Again, SPAM (Diskussion) 17:08, 5. Jan. 2016 (CET)
Oder vielleicht ist "Du Aal" gemeint? --Optimum (Diskussion) 17:54, 5. Jan. 2016 (CET)
Oder ein Zwiefacher? --188.174.50.152 19:46, 5. Jan. 2016 (CET)
Du sollst nicht denken und Zeitung lesen oder Wikipedia schreiben, sondern den Müll trennen, den Maschinen effektiver als MEnschen sortieren können und der hinterher wieder von der Abfallwirtschaft zusammen geworfen wird. Denn Fachkräfte kommen von unserem Dualen Ausbildungssystem, das weltweit Nachahmung findet. Nur kommen die Fachkräfte aus dem Land, dessen Abitur auf dem Level des deutschen Hauptschulabschluss liegt.[5] Hatte da Dieter Hallervorden schon vor Jahrzehnten eine Ahnung?[6] --Hans Haase (有问题吗) 15:15, 6. Jan. 2016 (CET)

Lustig ist das schon lange nicht mehr.:Archivierung dieses Abschnittes wurde gewünscht von: Vsop (Diskussion) 19:54, 5. Jan. 2016 (CET)

„20*C+M+B+16“ ?

Gestern habe zum ersten Mal Kids als Heilige-Drei-Könige-Sänger live erlebt. Als Nordlicht und ehemaliges Mitglied der evangelisch-lutherischen Kirche kannte ich das Spektakel nicht. Was bedeuten die Zeichen „20*C+M+B+16“, die am Schluss mit Kreide an die Tür geschrieben wurden? --77.10.247.222 00:49, 6. Jan. 2016 (CET)

Eine Erklärung findet sich im Artikel Sternsinger. --84.152.17.249 00:56, 6. Jan. 2016 (CET)
Ich habe aber 1946, als ich in den Süden kam, vom damaligen katholische Pfarrer gelernt, dass C+M+B für Caspar+Melchior+Baltasar steht. (Die Namen der drei heiligen Könige.) Die Deutungen im Artikel sind mir unbekannt, aber das heißt nicht, dass ich es besser wissen will.--2003:75:AF0C:5100:79EE:1B11:A145:CA4D 01:03, 6. Jan. 2016 (CET)
Der Artikel spiegelt die Auskunft Deines Pfarrers wieder: ab ca. 1950 offiziell die Abkürzung für "Christus mansionem benedicat". Vorher die Initialen der Heilige Drei Könige Caspar bzw. Kaspar, Melchior, Balthasar. --84.152.17.249 01:16, 6. Jan. 2016 (CET)
Beim Artikel Heilige Drei Könige#Brauchtum (letzter Absatz) hab ich die gleiche Erklärung für die unterschiedliche Deutung von „C+M+B“ gefunden. Danke für eure Hilfe! --77.10.247.222 01:32, 6. Jan. 2016 (CET)
Archivierung dieses Abschnittes wurde gewünscht von: --77.10.247.222 01:32, 6. Jan. 2016 (CET)
Da sag mir einer, die seien zementiert und ändern sich nicht.Und das schon 1950. Und mein Pfarrer war das nicht. Ich durfte gerade mal separiert auf der Fensterbank sitzen, als er erfuhr dass ich ungetauft nur mal schauen wollte, welcher Religionsunterricht mir besser gefällt. Die von der Konkurrenz nahmen mich dann freundlicher auf--2003:75:AF0A:E200:7161:9489:2EF4:BA32 09:53, 6. Jan. 2016 (CET)

Wer ist zenentiert? Das hat immer schon Christus Manebium Benedicat geheißen, aber weil viele Leute kein Latein konnten, haben sie's halt umbenannt. Und sich nicht überlegt, woher denn die Namen bekannt sein sollten. --Heletz (Diskussion) 09:57, 6. Jan. 2016 (CET)

Mit Verlaub, der mir das damals sagte, konnte Latein und wie. Ein Zeichen dafür, dass die es selbst nicht besser wussten, oder.--2003:75:AF0A:E200:7161:9489:2EF4:BA32 10:33, 6. Jan. 2016 (CET)
Im Artikel steht das Gegenteil und wird belegt. Wo sind deine Belege? --Eike (Diskussion) 10:35, 6. Jan. 2016 (CET)
Wen meinst Du?--2003:75:AF0A:E200:7161:9489:2EF4:BA32 10:42, 6. Jan. 2016 (CET)
Ich antworte immer einen Doppelpunkt weiter eingerückt als der Beitrag, auf den ich reagiere (wie von Hilfe:Diskussionsseiten vorgesehen); hier also Heletz. --Eike (Diskussion) 11:07, 6. Jan. 2016 (CET)
Noch so ein Latein, von dem die Leute nichts wissen.--2003:75:AF0A:E200:25C7:3AB:F209:EF3E 13:41, 6. Jan. 2016 (CET)

Thema Religionsvergleich

In welcher Religion gibt Vergebung

Siehe: Vergebung#Vergebung_in_den_Religionen oder ausführlicher: https://en.wikipedia.org/wiki/Forgiveness#Religious_views. --Buchling (Diskussion) 11:21, 4. Jan. 2016 (CET)

Frage zu Herausgabe der Personendaten durch den Provider zu einer IP

Das im Fall von Filesharing die Staatsanwaltschaft über den Provider die IP ermittelt kommt ja anscheinend häufiger vor. Jetzt habe ich aber schon mehrfach gelesen das dies auch im Fall von Beleidigungen auch bei Anonymer Anzeige geschieht. Ist das tatsächlich so? Wenn ein anonymer Nutzer den anderen anonymen Nutzer einen "dummen Hund" oder ähnliches nennt das die Staatsanwaltschaft tatsächlich Ermittlungen einleitet und die Nutzerdaten zur IP Adresse vom Provider anfragt? Dazu gibt es im Internet vollkommen unterschiedliche Angaben. Manche sagen ja das reicht auf Recht-gehabt (de) steht der Provider würde Daten nur bei Straftaten von erheblicher Bedeutung herausgeben. --84.174.66.56 07:00, 4. Jan. 2016 (CET)

1. ob der Provider diese Daten überhaupt noch hat, ist die Frage... da können auch Pannen passieren... 2. user:Eike will sowas mal gelungen sein (wegen „verp*** dich!“ oder so)... das hängt aber sicher von der Staatsanwaltschaft ab... wenn ich meine Vermieterin und den Landrat wegen Betruges anzeigte, passiert bestimmt nix... --Heimschützenzentrum (?) 09:09, 4. Jan. 2016 (CET)
Bei mir waren nicht beide anonym, er als IP hatte mich unter meinem Namen beleidigt. Da wurde dann aber tatsächlich aus der IP und dem Datum Name und Adresse ermittelt und die Person vor Gericht gebracht und... das Verfahren gegen Zahlung eines Bußgelds eingestellt oder wie das heißt. --Eike (Diskussion) 10:43, 4. Jan. 2016 (CET)
Das müsste aber dann doch im Ramen einer Privatklage passiert sein und nicht einfach mit einer Anzeige oder? --2003:66:8954:937A:4B3:DEE9:6981:D351 18:04, 4. Jan. 2016 (CET)
Schlichte Anzeige wegen Beleidigung. Die Mühe einer Klage hätte ich mir dafür dann doch nicht gemacht. --Eike (Diskussion) 18:09, 4. Jan. 2016 (CET)
n Staatsanwalt kann auch jemanden anklagen, nur weil der n Kaugummi-Päckchen geklaut hat, oder sogar auch obwohl keine hinreichenden Beweise vorliegen (Bsp: Horst Arnold (Justizopfer))... --Heimschützenzentrum (?) 00:23, 5. Jan. 2016 (CET)

Hinterkaifeck

Der ehemalige Einödhof Hinterkaifeck bei Schrobenhausen war 1922 Tatort eines ungeklärten grausamen sechsfachen Mordes, über den es mehrere Bücher gibt. Die Gebäude wurden im Jahr nach dem Verbrechen (1923) komplett abgebrochen. Wann aber wurde der Hof errichtet? Auf den historischen Karten im BayernAtlas (Flurkarte und Urpositionsblatt, beide wahrscheinlich 1. Hälft des 19. Jahrhunderts) ist davon noch nichts zu sehen. --Ratzer (Diskussion) 10:29, 4. Jan. 2016 (CET)

Zur Eingrenzung: Das Urpositionsblatt Schrobenhausen ist von 1863. --jergen ? 11:10, 4. Jan. 2016 (CET)
hinterkaifeneck.net: Baugeschichte hast du gelesen? Die Datierung um 1870 erscheint mir auch anhand der Fotos plausibel. --jergen ? 11:19, 4. Jan. 2016 (CET)

Danke an alle Wortmeldungen. Damit ist die Errichtung des Hofes zwischen 1863 (noch nicht auf Urpositionsblatt dieses Jahres) und 1869 ("vor 1870") schon ganz gut eingegrenzt.--Ratzer (Diskussion) 12:37, 4. Jan. 2016 (CET)

Dunărea

Wie kommt das deutschsprachige Google Maps (https://www.google.de/maps) dazu, den Fluss Donau in Bayern als "Dunărea" (rumänisch) anzuzeigen? Richtet sich der Name von Flüssen nach der Landessprache der Mündung? --Neitram  10:49, 4. Jan. 2016 (CET)

Wohl nicht. Jedenfalls heißt der Tejo auch an seiner Mündung bei Google Maps Tajo. --Vsop (Diskussion) 10:58, 4. Jan. 2016 (CET)

Der Fehler besteht offenbar seit Februar 2015. Hier wird spekuliert, dass es daran liegt, dass der Fluss mit 1075 km die weiteste Strecke in Rumänien zurücklegt. --77.180.1.253 11:08, 4. Jan. 2016 (CET)

Die Mosel heißt übrigens durchgehend La Moselle, also an der Mündung kann es nicht liegen. Die Ems hingegen heißt erst Ems und wird an ihrer Mündung zur Eems. Es geht wohl einfach ein bisschen durcheinander. Grüße Dumbox (Diskussion) 11:58, 4. Jan. 2016 (CET)
Bei Maps oder Earth konnte früher gewählt werden, ob „alternative Ortsnamen“ – gemeint war die Landessprache – angezeigt werden sollen. --Hans Haase (有问题吗) 12:44, 4. Jan. 2016 (CET)

Steam

Ich habe meine letzten Computerspiele gekauft lange bevor das ganze Zeug mit Registierung, Always-On etc. pp. losging und bisher noch hin- und wieder diese alten Spiele gespielt. Kürzlich ist Age of Empires II The African Kingdoms erschienen, das möchte ich aber ausprobieren. Geht es irgendwie ohne Steam auf reguläre Weise? Muss ich den Account so dann dauerhaft behalten?--Antemister (Diskussion) 11:44, 4. Jan. 2016 (CET)

Du könnste die Augen aufhalten ob gog.com es reinkriegt. --87.156.58.13 13:01, 4. Jan. 2016 (CET)
Du musst mit Steam normalerweise nicht ständig online sein, sondern dich nur gelegentlich (alle paar Wochen?) verknüpfen. Den Account kann man wohl löschen lassen [7], aber damit dürfte auch das Spiel "weg" sein. --Eike (Diskussion) 13:36, 4. Jan. 2016 (CET)
Ein über Steam gekauftes (auch 0€) Spiel bleibt mit diesem Accaunt verknüpft. Accaunt wegschmeisen ist also nicht die beste Idee. Aber bei den meisten Spielen brauchst du zum spielen keinen aktiven Accaunt, sondern nur zum herunterladen. Ist das Spiel mal auf dem Computer instaliert, brauchst du in der Regel kein aktives Steam.--Bobo11 (Diskussion) 14:08, 4. Jan. 2016 (CET)
Unser Artikel sagt was anderes: "Die Nutzung der DRM-Funktionalität ist jedoch nicht verpflichtend und wird von einer kleinen Anzahl von Spielen nicht genutzt." --Eike (Diskussion) 16:15, 4. Jan. 2016 (CET)
Steam installiert einen Service. Der ist immer aktiv, wenn die Kiste eingeschaltet ist. Ich schätze mal, noch keine Analyse gemacht, dass er sich im Hintergrund mit dem Server verbindet, für Updates und andere Sachen. --XPosition (Diskussion) 16:25, 4. Jan. 2016 (CET)
Das macht er, aber man muss Steam denke ich nicht als Service starten lassen, um es verwenden zu können. --Eike (Diskussion) 16:29, 4. Jan. 2016 (CET)
Zurück zur Ausgangsfrage: Ohne Steam geht also nichts?--Antemister (Diskussion) 16:33, 4. Jan. 2016 (CET)
Da geht anscheinend nichts. Ich hab die Kröte Steam geschluckt (Skyrim war schuld). Inzwischen bin ich ihnen sogar sehr dankbar, für ihr Linux-Engagement. --Eike (Diskussion) 16:40, 4. Jan. 2016 (CET)
Was könnte es denn überhaupt für einen Grund geben Steam nicht verwenden zu wollen wenn man eh schon Games darüber gekauft hat? Steam kostet selbst keinen Pfennig und läuft auch problemlos offline wenn man das möchte. Nur gelegentlich gibts mal einen negativ angehauchten Bericht in der Presse weil wieder einmal "Verbraucherschützer" glauben ein Haar in der ansonsten recht köstlichen SteamSuppe gefunden zu haben. --2003:66:8954:937A:4B3:DEE9:6981:D351 17:52, 4. Jan. 2016 (CET)
Ich habe bisher kein Steam (registiere mich ungern bei solchen Plattformen), deshalb die Frage. Und wenn man es von früher anderes kennt will man die ganzen (vllt.. nur gefühlten) Restriktionen nicht haben.--Antemister (Diskussion) 18:10, 4. Jan. 2016 (CET)
Es kann einem ja sogar passieren (wie mir mit Skyrim), dass man die DVD im Laden kauft und sich dann trotzdem bei Steam registrieren muss. Oder, wie anscheinend bei "The African Kingdoms", etwas nur über Steam erhältlich ist. --Eike (Diskussion) 18:14, 4. Jan. 2016 (CET)
Ich hab bisher keine schlechte Erfahrung mit Steam gemacht (Im Gegesatz zu den Hersteller-Platformen). Abe rich war sehr froh als ich meien Computer geschrottet habe, so einfach Spiele wieder zu instalieren wars selten (Bezüglich „auf einem andern Computer“ sowieso). Und eben wichtig, zum Spielen brauchst du in der Regel gar keine online Verbindung zu Steam. Einige blöde Angewohnheiten kann man Steam eh abgewöhnen. Zum Instalieren und ggf. Freischalten von Spielen ist Internett und Steam Zugriff natürlich Pflicht, aber eben sonst nicht. --Bobo11 (Diskussion) 20:38, 4. Jan. 2016 (CET)
Wie gesagt, ich befürchte, die meisten Steam-Spiele verlangen ab und zu nach der Bestätigung, dass du würdig bist (Internet-Verbindung). Was ich inzwischen klasse find, ist, dass man seinen Spielstand auswärts speichern lassen kann. Wenn man das Spiel deinstalliert und später wieder installiert, ob auf demselben oder einem anderen Rechner, ist er einfach wieder da... --Eike (Diskussion) 20:54, 4. Jan. 2016 (CET)
Ja und Nein. Richtig Steam schreit zwischen durch nach Internet, um sich bestätigen zu lassen, dass dieser PC der aktive Steam-PC ist. Es gibt allerdings Spiele die nach dem instalieren ohne Steam gestartet werden können. Es sind vorallem die Spiele die nur über die Steamoberfläche gestartet werden können, die auf eine aktualisierte Steamoberfläche angewisen sind. Das sidn aber in der Regel die Spiele die auch ohne Steam nach einer Internettverbindung schreien. Bei alten Schunken wäre mir das noch nie aufgefallen, dass die -nach dem Instalieren versteh sich- auf Steam angewiesen waren. Denn bei denen die Steam brachuen popt das geschlossen Fenster im Hintergrund auf (Wenn du es nicht sogar nur über Steamoberfläche starten kannst). >Klar hat Steam acuh seien nachteile aber die Vorteile überwiegen. Denn due hast den "CD-Schlussel" eben nicht mehr PC sondern Accaunt gebunden. Solange du zum Accaunt sogeträgst get der Schlüssel nie verloren oder wird sonstwie unbrauchbar. Auch kannst du dein Accaunt auch auf einem andern PC zwischen nutzen (z.B. während den Ferein auf dem Laptop). Wenn du allerding gewechselt hast, musst du danach zurück wechseln, udn dich bei Stema anmelden, denn abgemeldet wird der "alte" automatisch wenn du bei einem andern das Steam scharf machst. --Bobo11 (Diskussion) 21:18, 4. Jan. 2016 (CET)
Ist zwar nicht gar so schlimm wie ich dachte, aber schlimm genug: http://revuln.com/files/ReVuln_Steam_Service_Security.pdf Die updaten via Service mit system account. Eine kleine Schwachstelle und der ganze Rechner gehört euch nicht mehr und Steam könnte damit eigentlich auch alles machen. --XPosition (Diskussion) 21:56, 4. Jan. 2016 (CET)

Wann gab es den ersten Artikel in der Wiki?

und wo kann man die Entwicklung, also Anzahl der Artikel (so bis 2006) nachlesen. Dankeschön!--KleinerTimmy (Diskussion) 13:04, 4. Jan. 2016 (CET)

Zum ersten Artikel: Wikipedia:Geschichte --Eike (Diskussion) 13:07, 4. Jan. 2016 (CET)
Habs mir anders vorgestellt :-): Eingestellter Artikel im Januar 2004 Hellas live. Der Wievielte war er ungefähr? --KleinerTimmy (Diskussion) 13:26, 4. Jan. 2016 (CET)
Im Verlinkten verlinkt: Wikipedia:Meilensteine - also zwischen 40.000 und 50.000. --Eike (Diskussion) 13:34, 4. Jan. 2016 (CET)
Vielen Dank! --KleinerTimmy (Diskussion) 13:39, 4. Jan. 2016 (CET)
"in der Wiki" kann vieles bedeuten, das Prinzip gibt es seit über 40 Jahren. --Pölkkyposkisolisti 15:05, 4. Jan. 2016 (CET)

Microsoft Outlook: Reading Pane wird zum Reading Pain

Seit einem Update muss ich bei jedem Starten von Outlook "Reading Pane" auf "Right" schalten. Beim nächsten Start von Outlook ist es wieder auf "None" gesetzt. Wie kann man das permanent einschalten? Es nervt langsam. 130.226.41.19 15:04, 4. Jan. 2016 (CET)

Ich kann dir nicht helfen, aber beim Helfen helfen könnte vermutlich die Angabe, welche Version von Outlook du verwendest. --Eike (Diskussion) 15:08, 4. Jan. 2016 (CET)
Outlook 2010 130.226.41.19 16:13, 4. Jan. 2016 (CET)
Übrigens merkt sich auch andere Änderungen nicht: Ich hätte zum Beispiel gerne, dass E-Mails nicht sofort als "Gelesen" markiert werden, wenn man nur einmal kurz draufklickt. Ich kriege es einfach nicht hin, dass dies permanent bleibt, also auch nach Schliessen von Outlook. 130.226.41.19 16:57, 4. Jan. 2016 (CET)

wassily kandinsky Gemälde:las musas de

hallo,ich suche von diesem bild das herstellungsdatum und die original größe.bitte um Infos dazu. welei --88.217.83.246 19:41, 4. Jan. 2016 (CET)

Hallo welei, das ist kein Gemälde von Kandinsky, sondern eine Hommage an ihn. Wurde von einer New Yorkerin names "Chassie" im Jahr 2013 auf deviantart veröffentlicht. --King Rk (Diskussion) 20:21, 4. Jan. 2016 (CET)

Begriff gesucht (ähnlich "Illustration")

Wenn ich ein Sachverhalt an Hand eines realen oder fiktiven Beispiels erklären möchte, so ist es ein Beispiel. Wie wird aber ein Sachverhalt, der an Hand einer realen, vor Ort liegenden Gegenstands erklärt wird, genannt? Im Sinne: Wenn ich in der Schule bei mir zufällig ein paar Äpfel habe und eine Addition mit diesen Äpfeln illustrieren möchte. Gibt es dafür einen treffenden Begriff? Im Sinne: Es ist keine Parabel, kein Beispiel, keine Illustration, sondern was? Besten Dank im Voraus. --83.150.15.228 22:17, 4. Jan. 2016 (CET)

Exempel? --Buchling (Diskussion) 22:21, 4. Jan. 2016 (CET)
Der Sachverhalt bleibt ein Sachverhalt. Die Darstellung des Sachverhaltes (2+2=4) mithilfe von Äpfeln kann auch als Beispiel bezeichnet werden. --88.68.27.108 22:40, 4. Jan. 2016 (CET)
Veranschaulichung --Heldenzeuger (Diskussion) 22:57, 4. Jan. 2016 (CET)
Wiktionary: Demonstration – Bedeutungserklärungen, Wortherkunft, Synonyme, Übersetzungen
--Tobias "ToMar" Maier (Diskussion) 23:15, 4. Jan. 2016 (CET)
Ack, Demonstration paßt am besten. --88.68.27.108 00:13, 5. Jan. 2016 (CET)
Veranschaulichung geht auch noch. --Komischn (Diskussion) 17:21, 5. Jan. 2016 (CET)

Anzeige(n)

Stellt euch vor in einem Zeitungsartikel steht:

"Die beiden erstatteten am Dienstagnachmittag Strafanzeige."

Jetzt ist die Frage, was ist der Fall:

a) Es wurde eine Strafanzeige erstattet
b) Es wurden zwei Strafanzeigen erstattet
c) Man kann nicht wissen ob eine oder zwei Strafanzeigen erstattet wurden

--Distelfinck (Diskussion) 01:01, 7. Jan. 2016 (CET)

c)
Zwei Personen (die beiden) erstatteten Strafanzeige. Es ist nicht definiert, ob es jede der beiden für sich tat oder ob es beide gemeinsam taten. --2003:76:E4C:F1AD:714F:3699:8E3:9227 01:31, 7. Jan. 2016 (CET)
Also können zwei Personen eine einzige Strafanzeige gemeinsam erstatten. Danke, das wollte ich wissen. --Distelfinck (Diskussion) 02:25, 7. Jan. 2016 (CET)

Eine Anzeige ist sowieso lediglich "die Mitteilung eines Sachverhaltes an die zuständigen Strafverfolgungsbehörden". --Distelfinck (Diskussion) 02:27, 7. Jan. 2016 (CET)

Im Volks- und Pressemund wird aber nicht notwendigerweise zwischen Strafanzeige und Strafantrag unterschieden. --Rôtkæppchen₆₈ 02:31, 7. Jan. 2016 (CET)
Ah okay. Wenn ein Polizeisprecher sagt "es sind 30 Strafanzeigen eingegangen", was ist dann wohl in der Regel gemeint? --Distelfinck (Diskussion) 02:35, 7. Jan. 2016 (CET)
Damit sind in der Regel Stafanzeigen gemeint und nicht Strafanträge. Und gemeint ist damit, dass 30 Personen von einer Straftat berichtet haben - das kann durchaus 30 mal dieselbe Straftat gewesen sein, zum Beispiel wenn 30 Zeugen vom gleichen Überfall auf eine andere Person berichten: Der Anzeigende muss nicht Opfer der Straftat sein. --Snevern 07:08, 7. Jan. 2016 (CET)
Archivierung dieses Abschnittes wurde gewünscht von: --Distelfinck (Diskussion) 02:25, 7. Jan. 2016 (CET)

Titel einer deutschen Fernsehserie

Liebes agiles , dynamisches , engagiertes Antwortteam :-) , ich suche den Titel einer deutschen Fernsehserie . Sie lief vor etwa 5 Jahren im TV . Ein Astronaut reist durchs Weltall . Er erlebt viele wirre Abenteuer auf fremden Planeten . Sein Raumschiff ist sehr klein . Mit ihm fliegt ein weibliches Hologramm oder Roboter . Die Serie war insgesammt sehr crazy . Wahrscheinlich ist sie mir deshalb im Kopf geblieben . Könnt ihr mir helfen ? --Black Silent Pain (Diskussion) 01:59, 7. Jan. 2016 (CET)

Ijon Tichy: Raumpilot --Abderitestatos (Diskussion) 02:09, 7. Jan. 2016 (CET)
Wow , genial , wie schnell ihr mitten in der Nacht antwortet . Ihr schlaft wohl nie ;-) ? --Black Silent Pain (Diskussion) 02:26, 7. Jan. 2016 (CET)
Ich sollte mal wieder alle drei Staffeln Raumpatrouille und Ijon Tichy: Raumpilot glotzen. --Rôtkæppchen₆₈ 02:34, 7. Jan. 2016 (CET)
Was trägt das zur Beantwortung der Wissensfrage bei??? Hier ist nicht das Café! --178.4.107.253 02:44, 7. Jan. 2016 (CET)
Und was trug jetzt das zur Beantwortung der Wissensfrage bei? Geh und such dir einen Hausmeisterposten! Rotkaeppchen beantwortet hier mehr Fragen als die meisten anderen, und wenn dir nicht gefällt, dass er auch mal etwas schreibt, was nicht unbedingt nötig ist, dann schau doch einfach weg. Zum Beispiel an die Wand hinter dir. --Snevern 06:55, 7. Jan. 2016 (CET)
Gute Idee ! Aber erstmal schau ich Ijon Tichy: Raumpilot . Bist du Rotkaeppchen übrigens auch ein Mädchen ? Hier sind ja sonst fast nur Jungs . --Black Silent Pain (Diskussion) 02:49, 7. Jan. 2016 (CET)
Archivierung dieses Abschnittes wurde gewünscht von: Frage beantwortet. Privatplaudereien bitte auf der Benutzerdisku führen. --188.107.143.151 03:00, 7. Jan. 2016 (CET)

Verbindung zwischen CCC und Diskordianismus?

Existiert eigentlich eine Verbindung zwischen dem CCC und den Diskordianismus oder ist der Verweis auf "Chaos" und 23, der Gelegentlich auftaucht, nur ein Scherz, bzw. Zufall? --146.60.170.150 00:56, 5. Jan. 2016 (CET)

Auf jeden Fall - oder auch nicht! --Wicket (Diskussion) 02:59, 5. Jan. 2016 (CET)
Jetzt ist es 'raus. Die Tarnung war aber auch zu schwach. 84.153.83.193 07:24, 5. Jan. 2016 (CET)

Markantkaschierung (Lederoptik) und Filigrankaschierung (Strukturoptilk)

Hallo, es geht um einen Buchdruck und die Umschlagverarbeitung. Was genau bedeuten

Markantkaschierung (Lederoptik) und Filigrankaschierung (Strukturoptilk)?

Hat jemand Bilder, die es verdeutlichen? Besten Dank. --95.223.244.70 03:06, 5. Jan. 2016 (CET)

Das sollten Wortschöpfungen sein. Markant vs. filigran. Kaschiert heißt einfach nur "überzogen mit etwas". -- Janka (Diskussion) 03:29, 5. Jan. 2016 (CET)
Frag einfach mal beim Anbieter nach. Da gibt es laut Google genau einen Anbieter. --Rôtkæppchen₆₈ 07:58, 5. Jan. 2016 (CET)

Beispiele für "DDR-Kampagnen", die sich als (nachweislich) falsch herausstellten?

Immer wieder stolper ich über den Begriff der "DDR-Kampagne" insbesondere im Zusammenhang mit der Nazivergangenheit von BRD-"Größen" (Braunbuch), aktuell bei Lübke (Affäre). Gibt es Beispiele für "konstruierte" Kampagnen?--Wikiseidank (Diskussion) 13:56, 4. Jan. 2016 (CET)

Was versehst du unter "konstruiert"? Gefaket oder systematisch durchgeführt?
Der Begriff scheint teilweise von diesem Buchtitel zu kommen. Play It Again, SPAM (Diskussion) 14:22, 4. Jan. 2016 (CET)

Also in Bezug zur Nazivergangenheit ist mir nichts von "konstruierten" Aktionen bekannt. Eher schien mir die Nachrichtenlage so zu sein, daß insbesondere die STASI vieles wußte, und versuchte, damit Einfluss auf einige Politiker per Erpressung zu nehmen. Bei anderen übergaben sie die vorhandenen Daten zielgerichtet entweder neutralen westlichen Journalisten oder mit dem Osten verbundene. Diese waren aber immer "nachweislich" richtig, bzw. interpretierbar. Bei dem Lemma fielen mir eher die Kampagne mit dem Ausbruch von AIDS durch US-Labors ein, und die angebliche Mitropaentführung 1989. Aber auch das war eher niedrigschwellig auf Journalistenbasis, und nicht durch staatliche Institutionen.Oliver S.Y. (Diskussion) 14:29, 4. Jan. 2016 (CET)

Bei Victor Klemperer, "zwischen allen Stühlen", findet man aus der Frühzeit von SBZ und DDR allerlei Propagande, wie etwa, "der Westen" fliege Kartoffelkäfer ein, um die Ost-Ernte zu zestören, was K. selbst für Unsinn hielt. --Logo 14:35, 4. Jan. 2016 (CET) Wow, wir haben sogar einen Abschnitt Kartoffelkäfer als Mittel der Propaganda.

Staatlich gefördert waren eher die Vertuschungsversuche im Westen, wenn es um Nazigrößen ging. Im Osten wurden sie verurteilt, im Westen haben sie Karriere gemacht. Und das ist keine Propaganda. --Pölkkyposkisolisti 15:03, 4. Jan. 2016 (CET)

Klemperer ist jetzt nicht so "DDR-Propaganda" (im Sinne von: '"konstruiert", gefaket oder systematisch durchgeführt') schon von wegen der Reichweite. Kampagne ist erst mal ein neutrales Wort. Bei der Bezeichnung "DDR-Kampagne" schwingt immer der Begriff Hetzkampagne mit (oder?) Gab es DDR-Propaganda/Hetzkampagnien? Vielleicht analog zu diesem Kaliber.--Wikiseidank (Diskussion) 15:12, 4. Jan. 2016 (CET)
Weia. Gemeint ist natürlich nicht Propaganda, die Klemperer macht, sondern von der er berichtet; dafür ist er schließlich berühmt. cf LTI --Logo 17:28, 5. Jan. 2016 (CET)
@Pölkky - auch im Osten gab es politisch aktive Ex-Nazis: Liste ehemaliger NSDAP-Mitglieder, die nach Mai 1945 politisch tätig waren#Sowjetische Besatzungszone und Deutsche Demokratische Republik.--Alexmagnus Fragen? 15:20, 4. Jan. 2016 (CET)
Diese Liste ist aber für Österreich erstaunlich mager bestückt – da scheint es noch erheblichen Arbeitsbedarf für Wikipedianer zu geben. --Jossi (Diskussion) 17:07, 4. Jan. 2016 (CET)

Theodor Oberländer#Rücktritt. --Vsop (Diskussion) 22:57, 4. Jan. 2016 (CET)

„Niemand hat die Absicht, eine Mauer zu errichten!“. Sie wurde pünktlicher geliefert als der Trabant. --Hans Haase (有问题吗) 06:26, 5. Jan. 2016 (CET)
Das war aber keine Kampagne, sondern nur ein falsches Versprechen eines Politikers. Sowas ist eigentlich in jeder Regierung üblich. --MrBurns (Diskussion) 07:24, 5. Jan. 2016 (CET)
Kein Versprechen, sondern eine Antwort auf die Frage eines Journalisten. Aber an dem Satz ist immer wieder das Interessanteste, dass in der Frage selbst von keiner Mauer die Rede ist. Die bringt Ulbricht selbst ins Spiel.--Alexmagnus Fragen? 16:50, 5. Jan. 2016 (CET)

200 Millionen Dollar pro Tag

kostet die Einmischung von Saudi-Arabien im Jemen den Staat => [8]. Wie kommt diese für mich riesige Summe zustande? Die Armee muss sowieso unterhalten werden, und es werden ja nicht jeden Tag zwei Kampfflugzeuge abgeschossen? Irgendwo mach ich einen Denkfehler, helft mir mal. Gruss --Nightflyer (Diskussion) 22:16, 4. Jan. 2016 (CET)

Das scheint hoch geschätzt, könnte aber auch stimmen, wenn man es mit folgenden Annahmen nachvollzieht. Es kommt auf die Verluste an und den Verbrauch. Die Bundeswehr gab vor unlängst bekannt, dass die Flugstunde des Eurofighter 88 Teuro und die des NH90 45 Teuro kostet. Wenn man je 10 davon einsetzt, mit ca. 5 Flugstunden am Tag, macht das 4,4 Mio am Tag beim Eurofighter und 2,2 Mio beim NH90. Panzer etc. kosten zwar nicht so viel aber auch das läppert sich durch die größere Anzahl und Betriebszeit zusammen. Dann die Munition, allein so eine Panzergranate kostet bis 200 Teuro, eine Flugabwehrrakete geht in die Millionen. So ein Rohr am Panzer ist nach 150 Schuss krumm und reif zum Austausch. Angenommen man hat Kriegsgerät von insgesamt 200 Milliarden im Einsatz und rechnet bis 5% realistischen Verlust und Abnutzung pro Tag, sind das bereits 100 Mio Euro pro Tag. Diese Werte begründen auch eine eigene Wehrindustrie im Lande. Durch den Mittelrückfluss durch Steuern und Abgaben kommen beim Staat wieder über 70% zurück, muss man alles im Ausland kaufen, ist der Steuerzahler in den Arm gekniffen.--2003:75:AF0C:5100:4EE:7D4D:1936:93F7 08:47, 5. Jan. 2016 (CET)
Deren Steuerzahler vielleicht. Der deutsche nicht. --j.budissin+/- 12:40, 5. Jan. 2016 (CET)
Verstehe nicht, wie du auf die 100 Mio kommst. --Chricho ¹ ² ³ 13:10, 5. Jan. 2016 (CET)
Au weia, da hab ich mich mathematisch total verhauen. Die 5% Verlust an Flugzeuge und Besatzungen pro Einsatztag habe ich von Bomber (Sir Arthur) Harris, der wurde 1943/44 nur nervös, wenn es mehr war. Und die 5% Verlust für alle Waffengattungen geistern weiter als Faustformel durch die Gegend. Sie dürften bei solchen Scharmützeln bei den Saudis auch heute noch passen. Die echten Zahlen halten die Kriegsparteien auch (lange später noch) geheim. Bitte um Verzeihung, bleibe heute zur Strafe der Auskunft fern.--2003:75:AF0C:5100:4C66:6EE7:138B:944D 16:34, 5. Jan. 2016 (CET)

Arbeitsuchend-Meldung zu spät: Sperrzeit nachträglich vermeiden

Liebe Auskunft, Person A wird zum 1. März arbeitslos. A hat sich am 1. Dezember, also einen Tag zu spät, arbeitsuchend gemeldet. Die Bundesagentur verweigert jetzt für die erste Woche die Zahlung des Arbeitslosengeldes – was wohl üblich ist. Ist Folgendes richtig? Würde A erst zum 2. März arbeitslos, weil sie am 1. März für einen Tag freiberuflich tätig ist, hätte sie Anspruch auf Arbeitslosengeld ab dem 2. März. Nur falls diese Behauptung richtig ist: Kann die Arbeitsuchend-Meldung nachträglich auf den 2. März geändert werden? Vielen Dank für Antworten! --BlackEyedLion (Diskussion) 13:27, 5. Jan. 2016 (CET)

Nein, da mit der freiberuflichen Tätigkeit kein Anspruch auf Arbeitslosengeld erworben wird. -- Liliana 13:36, 5. Jan. 2016 (CET)
So wie Du es schilderst, wurde die Sperrzeit ja bereits verhängt, und wahrscheinlich der Leistungsbescheid bereits erstellt. Darum kann eine Aufhebung der Sperre wahrscheinlich nur im Widerspruchsverfahren erreicht werden. Wenn es sich wirklich nur um diesen einen Tag handelt, ist eine Erklärung für diese Verzögerung aussichtsreicher. Wichtig dabei zB., wann man die Kündigung erhalten hat, bzw. von den Umständen der Arbeitslosigkeit erfuhr. So ein simpler Trick wie freiberufliche Tätigkeit zieht da schon allein deshalb nicht, weil auch diese im Antrag bereits angegeben werden musste, und je nach Art (zB. Künstler) es lediglich eine Frage der Haupt- und Nebenbeschäftigung wäre, gerade wenn kein Einkommen damit erzielt wird.Oliver S.Y. (Diskussion) 13:55, 5. Jan. 2016 (CET)
Man kann schon die Sperrzeit vermeiden, wenn man eine neue sozialversicherungspflichtige Beschäftigung aufnimmt, die direkt an die vorherige Beschäftigung anschließt. Dann nämlich ist die alte Arbeitslosmeldung Makulatur und nicht mehr relevant. Ansonsten hast du natürlich recht, wenn es nur um einen Tag geht, gibt es andere, vielversprechendere Möglichkeiten, die Sperrzeit zu vermeiden. -- Liliana 13:58, 5. Jan. 2016 (CET)
"Sperrzeit vermeiden"? Dann gibt es schlicht keinen Anspruch auf ALG 1, und die Auflösung des anderen Beschäfitungsverhältnisses dürfte noch mehr Probleme verursachen.Oliver S.Y. (Diskussion) 14:17, 5. Jan. 2016 (CET)
(BK)Ich habe meine Arbeitssuchend-Meldung erst eine Woche nach der "Kündigung" (es war eine Exmatrikulation, vielleicht ein Sonderfall) getätigt und außer einem Rüffel ("Warum kommen Sie jetzt erst?") keine Nachteile, von denen ich weiß. Habe ab dem Kündigungszeitpunkt die Leistungen nachträglich erhalten. Welches Vorgehen üblich ist, kann ich nicht beurteilen. --95.114.193.186 14:00, 5. Jan. 2016 (CET)
A hat rechtzeitig gewusst, dass das befristete Beschäftigungsverhältnis endet und sich tatsächlich zu spät gemeldet. Es ist mir nur unverständlich, warum A schlechter gestellt wird als eine vergleichbare Person. Wie A schon bei der Arbeitsuchend-Meldung eine freiberufliche Tätigkeit am 1. März hätte angeben können, verstehe ich nicht; Wesen freiberuflicher Tätigkeiten ist ja, dass man sie einfach aufnehmen und wieder einstellen kann: A wird sich am Morgen des 1. März entscheiden, an dem Tag freiberuflich tätig zu sein (zum Beispiel den ganzen Tag über eine Beratertätigkeit auszuüben), und diese Tätigkeit am Abend wieder einstellen; wenn es sein muss, wird A dabei ausreichend viel Einnahmen und Gewinn erzielen. --BlackEyedLion (Diskussion) 14:09, 5. Jan. 2016 (CET)
Die freiberufliche Tätigkeit spielt aber keine Rolle, weil man während einer freiberuflichen Tätigkeit nicht in der Arbeitslosenversicherung versichert ist. Dementsprechend kann man keinen Anspruch auf Arbeitslosengeld erwerben, wenn man freiberuflich tätig ist. Für die Sperrzeit spielen nur solche Beschäftigungsverhältnisse eine Rolle, die mit einem Anspruch auf Arbeitslosengeld einher gehen. -- Liliana 14:16, 5. Jan. 2016 (CET)
@BEL, das hier ist eine Auskunft für allgemeine Wissensfragen. Deine Fragestellung war schon sehr individuell, auch wenn Du es mit Person A umschreibst. Du mußt nicht uns von der Sache überzeugen, sondern das Amt. Liliana hat einen maßgeblichen Grund dafür genannt, ich sage nur, daß eine "spontane" freiberufliche Tätigkeit auch keinen Einfluss hat. Näheres dafür steht in den Informationen, welche Du bei Antragstellung erhalten hast, samt deren Kenntnisnahme. Wie IP 95 sagt, gibt es einen gewissen Ermessensspielraum, der hängt aber sowohl von den Umständen bei A als auch dem Bearbeiter im Amt ab. Kann mich erinnern, irgendwo stand sogar, daß diese Umstände bei Erlangung der Kenntnis dem Amt zu melden sind, hier also bereits bei Vertragsunterzeichnung. Da wäre 1 Woche Sperre sogar noch "glimpflich".Oliver S.Y. (Diskussion) 14:23, 5. Jan. 2016 (CET)
Erstens handelt es sich um eine allgemeine Wissensfrage (das Beispiel ist fiktiv, insbesondere die angefragte mögliche Lösung), zweitens ist Lilianas Antwort zwar richtig, hier aber irrelevant: A soll nicht für die Zeit der freiberuflichen Beschäftigung Arbeitslosengeld erhalten, sondern für die Zeit danach; die Zeit der Anwartschaft (12 Monate Beitragszahlung innerhalb der letzten 24 Monate) ist erfüllt.
Ich muss noch einmal genau nachfragen: Hat eine Person, deren nichtselbstständige Beschäftigung voraussehbar endet und danach für kurze Zeit freiberuflich tätig ist, Anspruch auf Arbeitslosengeld ab dem Ende der freiberuflichen Tätigkeit, wenn nicht der erste, aber dieser letzte Zeitpunkt drei Monate nach der Arbeitsuchend-Meldung liegt? --BlackEyedLion (Diskussion) 15:04, 5. Jan. 2016 (CET)
Nochmals, Du konstruierst hier einen Sonderfall, der keine allgemeine Antwort erlaubt. Die Informationsseiten der Bundesagentur kennst Du sicher [9], [10], [11]. Dort finden sich die Antworten auf Deine Frage. Deine Frage ist grundsätzlich mit ja zu beantworten, aber im konkreten Fall mit nein, da wegen der verhängten Sperrfrist er Auszahlungsanspruch erst später beginnt. Welcher Art die Tätigkeit bzw. Nebentätigkeit ist, spielt auch keine Rolle, denn auch darüber ist das Amt rechtzeitig zu informieren. Am ersten Tag der Sperrfrist zu entscheiden, ab heute bin ich Berater, nette Idee, nur ohne finanzielle Auswirkungen, ggf. verliert man sogar den Anspruch auf Leistungen ab Woche 2, wenn die Tätigkeit ausgelegt ist, mehr als 15 Wochenstunden Umfang zu haben.Oliver S.Y. (Diskussion) 16:06, 5. Jan. 2016 (CET)
Es gibt da ein wunderschönes Beispiel in den dienstlichen Anweisungen zu § 159 SGB III, das ich gerne hier zitieren möchte:
                bis  31.03.  Das unbefristete Arbeitsverhältnis wird durch AG-
                                  Kündigung vom 10.02. zum 31.03. beendet.
             01.04.  bis  30.11.  selbständige Tätigkeit
             01.12.  Arbeitslosmeldung/Antragstellung

Der Eintritt der Sperrzeit ist bezogen auf die Beendigung des Arbeitsverhältnisses am 10.02. zum 31.03. zu prüfen.

Die siebenmonatige selbständige Tätigkeit spielt keine Rolle für die Verhängung der Sperrzeit, da es keine arbeitslosenversicherungspflichtige Beschäftigung ist. Somit tritt die Sperrzeit ein, da man sich bereits am Tag nach dem 10. Februar hätte arbeitssuchend melden müssen. Dass man danach eine selbständige Tätigkeit aufnimmt, ist egal. -- Liliana 16:19, 5. Jan. 2016 (CET)

Wie kann man Inhalte aus wikipedia-Seiten entfernen lassen

Es geht um die Seite über SKODA Auto in der deutschsprachigen Wikipedia - https://de.wikipedia.org/wiki/%C5%A0koda_Auto. Zum Abschnitt "Geschichte" gehört als erstes "1895 bis 1905 – Gründung von Laurin & Klement", darauf folgt "1914 bis 1925 – Der Verkauf von Laurin & Klement an Škoda" was an sich eine Manipulation von Fakten ist. Denn so sieht es aus, als ob Skoda aus dem nichts entstanden wäre - denn das ist die erste Erwähnung von Skoda im Artikel. Wo ist die Geschichte wie Skoda entstanden war? Ist es unwichtig weil Skoda gar nichts spannendes bis dahin entwickelt hat? Daraus folgt auch dass die Erfolge von Laurin & Klement zu den erfolgen von Skoda zählen, wobei die Marke Laurin&Klement danach verschwand. Ich bitte den Abschnitt "1895 bis 1905 – Gründung von Laurin & Klement" gegen einen Link Laurin&Klement zu ersetzen, denn das war eine separate Firma, und es ist unprofessionel, die Geschichte des Unternehmens SKODA mit einem anderen Unternehmen anzufangen, das später gekauft wurde. --94.216.59.90 21:12, 5. Jan. 2016 (CET)

Hinweis: Für Fragen speziell zur Wikipedia gibt es ein eigenes Diskussionsforum: Fragen zur Wikipedia. -- FriedhelmW (Diskussion) 21:33, 5. Jan. 2016 (CET)

Zwoter Hinweis: Anmerkungen zu einzelnen Artikeln gehören auf die jeweilige Diskussionsseite, also hier Diskussion:Škoda Auto. Aber um eine Manipulation handelt es sich hier sicher nicht. Die Autoherstellersparte von Škoda, das heutige Škoda Auto, begann als Laurin & Klement, und wurde dann von Škoda (Maschinenbau) aufgekauft. Und wie diese Firma entstanden ist, steht im selbigen Artikel. Man muss das nicht so gliedern, aber falsch ist das auch nicht. --King Rk (Diskussion) 21:38, 5. Jan. 2016 (CET)
Laut Vorposter ist es eben schon falsch, weil die Autoherstellung von Laurin & Klement erst ab dem Zeitpunkt etwas mit Skoda zu tun hatte, als Skoda das Unternehmen aufkaufte. Es ist darum plausibel dass jegliches was davor passierte nicht in den Artikel Skoda gehört. --Cubefox (Diskussion) 22:53, 6. Jan. 2016 (CET)
Der Hinweis auf Škoda findet sich schon in der Einleitung, wirkt also sicher nicht wie aus dem Nichts entstanden. Den gewünschten Link auf Laurin & Klement gibt es auch schon, bloß dass daraus auch gleich einiges wiedergegeben wird, um den Kontext herzustellen. Sollte Škoda schon vor 1925 Autos gebaut haben (und nicht nur irgendetwas „spannendes“), wäre das freilich ein Grund umzustrukturieren. Ist das denn so? --Chricho ¹ ² ³ 12:02, 6. Jan. 2016 (CET)
Archivierung dieses Abschnittes wurde gewünscht von: Speravir (Disk.) 01:38, 8. Jan. 2016 (CET)

5-Euro Kursmünze

Stimmt es, dass am 14. April eine neue 5-Euro-Kursmünze ausgegeben wird, die einen blauen Polymer-Ring besitzt? 188.103.255.199 22:02, 5. Jan. 2016 (CET)

Ja. Wobei das Datum nicht genau genannt wird. Gruss --Nightflyer (Diskussion) 22:14, 5. Jan. 2016 (CET)
Nein. Kursmünze. Sammlermünze. --Studmult (Diskussion) 22:21, 5. Jan. 2016 (CET)
(BK)Im verlinkten Text steht Sammlermünze, was etwas anderes als Kursmünze ist. --Rôtkæppchen₆₈ 22:22, 5. Jan. 2016 (CET)
OK, keine Kursmünze. Wobei jede in DE geprägte Münze auch kursgültig ist, sonst wäre es eine Medaille. Zu Beginn des Euro-Zeitalters habe ich mir manchmal 10er als Silbereuros von der Bank geholt, sie dann aber beim Einkaufen ausgegeben. Gab immer lustige Diskussionen :-) Gruss --Nightflyer (Diskussion) 22:43, 5. Jan. 2016 (CET)
Na ja, wenn dieser Fünfer zum Nennwert verkauft wird und wenn er nicht aus Edelmetall ist, kannst Du ihn in großen Mengen beziehen und ausgeben. Er wird dann zur Kursmünze. 188.103.255.199 00:06, 6. Jan. 2016 (CET)
Ich habe mal einen "Sammel"-10er als Rückgeld bekommen. Zum Glück wusste ich, dass es die Münze gibt, gab also keine Diskussion.--Alexmagnus Fragen? 23:55, 6. Jan. 2016 (CET)

Geld kauft man gegen Geld! Verstanden! --KleinerTimmy (Diskussion) 18:05, 6. Jan. 2016 (CET)

Als Geld bezeichnet den Tauschwert, zum Beispiel "10 Euro". Die hat aber noch nie jemand gesehen, er hat keinen Körper. Der Tauschwert ist was ideelles. Statt dessen sieht man aber oft Münzen und Geldscheine im Werte von 10 Euro. Münzen und Geldscheine sind aber selber Waren, die in einander getauscht werden können, nur dass sie selbst keinen Gebrauchswert haben. Nach Karl Marx haben sie überwiegend Tauschwert, sieht man mal von ihrer Funktion als Unterlage für wackelige Bildschirme und ihren Metallwert ab.
Münzen und Geldscheine kauft man mit anderem Geld. In Form von Kontoüberweisungen oder anderen Münzen und Geldscheine. Zum Beispiel kauft man 100 Pfund, in dem man ihren Gegenwert in Euro bezahlt. Und man kann auch vier 5-Eurostücken kaufen, in dem man 20 Euro bezahlt. 188.103.255.199 19:08, 7. Jan. 2016 (CET)
Archivierung dieses Abschnittes wurde gewünscht von: Speravir (Disk.) 01:37, 8. Jan. 2016 (CET)

Wieviel Gefängnisplätze gibt es in Deutschland?

Ich habe das Thema ins Cafe verlegt. Die Frage war beantwortet, aber offenbar bei einigen noch immer ein sehr starker Kommunikationsdrang. --2003:45:463D:4600:2D8F:BB30:ECA8:D017 19:42, 7. Jan. 2016 (CET)

Archivierung dieses Abschnittes wurde gewünscht von: Speravir (Disk.) 01:37, 8. Jan. 2016 (CET)

Quizfrage: Wo stehen die beiden ...nadeln?

Neulich war in einem Quiz die Frage: "Wo stehen die beiden ...nadeln?" (ich hoffe ich gebe das richtig wieder). Leider kann ich mich an "..." nicht mehr erinnern. Ich glaube es geht um Bauwerke (Obelisken?), und "..." war eine weibliche altägyptische Gottheit oder Pharaonin. Da die Wikipedia-Suche mit "*nadel" derzeit nicht funktioniert meine Frage hier. --Zulu55 (Diskussion) Unwissen 11:21, 6. Jan. 2016 (CET)

Nadeln der Kleopatra? Antwort: London und New York. Grüße Dumbox (Diskussion) 11:24, 6. Jan. 2016 (CET)
Super, das wars, danke. War das bei Wer-wird-Millionär? --Zulu55 (Diskussion) Unwissen 11:25, 6. Jan. 2016 (CET)
Das weiß ich leider nicht, noch nie geguckt. :( Grüße Dumbox (Diskussion) 11:28, 6. Jan. 2016 (CET)
Archivierung dieses Abschnittes wurde gewünscht von: Speravir (Disk.) 01:37, 8. Jan. 2016 (CET)

Lied toter Truthahn auf dem Sofa (Männerstimme, ca. 2000er-2010er)

Wie heißt das Lied mit folgenden Textbestandteilen: Da liegt ein toter im Zimmer und ein Truthahn auf dem Sofa. Die Leiche schicke ich zum Bundestag, die haben genug Leichen im Keller, oder zum Irak, da fällt es nicht auf. --93.134.183.203 13:05, 6. Jan. 2016 (CET)

’ne Leiche. Auf dem Sofa liegt aber „ein Toter“, kein „Truthahn.“ ;) --L47 (Diskussion) 13:19, 6. Jan. 2016 (CET)
’Ne Leiche von SDP -- Hey Teacher (Diskussion) 15:43, 6. Jan. 2016 (CET)
Archivierung dieses Abschnittes wurde gewünscht von: Speravir (Disk.) 01:36, 8. Jan. 2016 (CET)

Grillen auf dem Bürgersteig

Hallo, da ich online dazu nichts finden konnte. Ist es eigentlich verboten auf dem Bürgersteig zu grillen? Es handelt sich um eine ruhige Straße einer deutschen Großstadt. Recht wenig befahren, Passanten haben Möglichkeit vorbeizugehen, ist halt nur auf einem Bürgersteig in der Öffentlichkeit. Grüße--178.6.217.196 18:45, 6. Jan. 2016 (CET)

Wo kein Kläger, da kein Richter. --Heletz (Diskussion) 18:47, 6. Jan. 2016 (CET)
Kinder benutzen den Gehweg zum Gehen, Spielen und Radfahren. Das ist auch im Gesetz so vorgesehen. Diese Kinder werden durch offenes Feuer gefährdet. Kommt ein Ordnungshüter vorbei, dürfte es mit Sicherheit Ärger geben. --Pölkkyposkisolisti 18:49, 6. Jan. 2016 (CET)
Ohne Genehmigung ist das verboten, da es sich beim Grillen ganz offensichtlich um eine Sondernutzung des Gehwegs handelt. Yellowcard (D.) 18:59, 6. Jan. 2016 (CET)
(BK)Das ist landesrechtlich eine genehmigungs- und kostenpflichtige Sondernutzung. Näheres bei der zuständigen Ordnungsbehörde. --Rôtkæppchen₆₈ 19:00, 6. Jan. 2016 (CET)
Das ist der Artikel, den ich suche, vielen Dank!--178.6.217.196 19:08, 6. Jan. 2016 (CET)
"Verboten" ist das eine, soziales und umweltverträgliches Verhalten das andere/wichtigere. Wie kommt man auf so eine Idee? Hat man das woanders gesehen? Wenn nicht, warum wohl hat man so etwas noch nicht gesehen? Nur weil etwas möglich ist, muss man es nicht auch unbedingt machen.--Wikiseidank (Diskussion) 21:02, 6. Jan. 2016 (CET)
Dem habe ich nichts hinzuzufügen. Gruß 79.224.219.66 22:20, 6. Jan. 2016 (CET)
Archivierung dieses Abschnittes wurde gewünscht von: Speravir (Disk.) 01:33, 8. Jan. 2016 (CET)

double Dateityp bei Java

Kann mir jemand sagen warum man hier beim Beispiel 3 ein d in den dobule Dataityp schreiben darf? Thx.--Sanandros (Diskussion) 20:00, 6. Jan. 2016 (CET)

Du meinst jeweils am Ende der Zahl? Das ist schlicht der Kennzeichner für Double-Zahlen. [12] --Eike (Diskussion) 20:17, 6. Jan. 2016 (CET)
Schau dir bspw. Python (Programmiersprache) an, da ist das glaube ich alles ein wenig einfacher... Da muss man in der Regel nicht reinschreiben was das für eine Art von Zahl nun sein soll. Zumindest mir kommt das alles ein wenig einfacher vor als Java. --87.140.192.144 21:46, 6. Jan. 2016 (CET)
Ist das zwingend bei so grossen Zahlen? Und wie komme ich von 2147483642d auf 2.147483642E9 oder steckt im den d dann das E9 drin?--Sanandros (Diskussion) 22:13, 6. Jan. 2016 (CET)
2147483642d ist kurz für 2147483642d+00 und das ist zahlenmäßig gleich 2147483642E+00 oder 214748364.2E+01 oder … 2.147483642E+09 (Dezimalpunkt verschieben). --Rôtkæppchen₆₈ 23:00, 6. Jan. 2016 (CET)
Und was könnte schlimmes passieren wenn ich das d weglasse?--Sanandros (Diskussion) 23:10, 6. Jan. 2016 (CET)
Dann wird die Zahl als Ganzzahl interpretiert. Ich hab früher in C immer .0 dazugeschrieben, wenn es eine float- oder double-Konstante sein sollte: 2147483642.0 . --Rôtkæppchen₆₈ 00:04, 7. Jan. 2016 (CET)
OK ich habe es jetzt mit deiner Hilfe und mit der HIlfe dieser Page verstanden. Wenn ich double d = 1 / 2 rechne erhält d den Wert 0.0 und wenn ich doube d = 1.0 / 2.0 rechne erhält d den Wert 0.5. Thx. Jetzt wäre nur noch die Frage offen wie ich einen User dazu bringe einen double einzugeben ohne dass ich ihn darauf hinweise?
Also die Idiotensichere alternative zu:
Scanner user_input = new Scanner( System.in );
double user_double;
System.out.print("Enter double:")
user_double = user_input.next()

--Sanandros (Diskussion) 09:12, 7. Jan. 2016 (CET)

Wenn du den Eingabe-String als double parst und in eine double-Variable speicherst, kannst du ganz normal damit rechnen, ob es nun 2 oder 2.0815 ist. Die Zwei verhält sich dann wie eine 2.0d. --Eike (Diskussion) 10:50, 7. Jan. 2016 (CET)
Ah ok ich habe jetzt mal das Prog geschrieben und dabei kam alles am ende so raus wie es sollte also calc hatte den Wert 0.5:
//Scann first number like 1
Scanner user_input = new Scanner( System.in );
double user_double1;
System.out.print("Enter double:");
user_double1 = user_input.nextDouble();
	
//Scann second number like 2
double user_double2;
System.out.print("Enter double:");
user_double2 = user_input.nextDouble();

//Calculate 1 / 2 = 0.5
double calc = user_double1 / user_double2;
System.out.println(calc);
Vielen Dank fürs Helfen.--Sanandros (Diskussion) 12:45, 7. Jan. 2016 (CET)
Du musst bei Fließkommazahlen halt bedenken, da kommt nicht immer haargenau raus, was man sich denkt. Wichtig ist aufzupassen, dass sich Fehler nicht anhäufen. Wenn man immer wieder floats aufaddiert, kann aus kleinen Abweichungen ein großer Fehler werden. (Für ein fieses Float-Rechenbeispiel kannst du mal ausprobieren, was Janka hier gegen Ende beschreibt.) --Eike (Diskussion) 12:56, 7. Jan. 2016 (CET)
@Sanandros: Zur Frage, wie Du einen User dazu bringst, einen double einzugeben ohne dass Du ihn darauf hinweist: Bei manchen Programmiersprachen lässt sich der Eingabedatentyp vorher festlegen. Bei vielen BASIC-Dialekten schreibt man z.B. INPUT xy!, wenn man eine float-Variable eingeben will oder INPUT xy# wenn die Variable double sein soll. Analog INPUT xy% für Integer, INPUT xy& für long und INPUT xy$ für String. Es gab aber auch BASIC-Dialekte, die nur zwischen numerisch und String unterschieden. In C gibt man grundsätzlich char oder strings ein und aus. Umwandlungen werden hier mit printf, scanf, sprintf, sscanf, fprintf und fscanf gemacht, wobei der verpflichtende Formatstring das Aus- oder Eingabeformat vorgibt. --Rôtkæppchen₆₈ 15:28, 7. Jan. 2016 (CET)
Danke für den Hinweis. Ich werde das mal im Hinterkopf behalten. Bin gerade daran meine kleine Java Kenntise nach langer Zeit wieder zu nutzen.--Sanandros (Diskussion) 22:17, 7. Jan. 2016 (CET)
Archivierung dieses Abschnittes wurde gewünscht von: Speravir (Disk.) 01:32, 8. Jan. 2016 (CET)

Warum wird beim Casten in manchmal Java gerundet und manchmal nicht?

Warum wird wenn ich die Zahl 9.999999999 von double nach float caste auf 10.0 gerundet, jedoch wenn ich die Zahl 9.999999999 von double nach int caste nicht gerundet und stattdessen 9 ausgegeben?--Sanandros (Diskussion) 22:56, 6. Jan. 2016 (CET)

Weil beim nach Int casten generell nicht gerundet wird, sondern nach dem Komma abgeschnitten. Zumindest kenne ich keine Programmiersprache die das anders macht. --FGodard||± 23:05, 6. Jan. 2016 (CET)
Dass er, wenn er nach int castet, abschneidet finde ich ja ok aber ich wundere mich eher dass er, wenn er nach float castet, nicht einfach 9.999999 schreibt wundert mich.--Sanandros (Diskussion) 23:15, 6. Jan. 2016 (CET)
Ist 9.999999999 in float überhaupt von 10.0 unterscheidbar? --Rôtkæppchen₆₈ 23:24, 6. Jan. 2016 (CET) Es ist es nicht zumindest wenn man meinem Visual Studio glaubt. --Rôtkæppchen₆₈ 23:29, 6. Jan. 2016 (CET)
(BK)Die Typumwandlung zwischen float und double wird in IEEE 754 geregelt. Dabei muss exakt gerundet werden. Die Umwandlung zwischen float bzw double und ganzzahligen Typen wird meistens in der Definition der jeweiligen Programmiersprache geregelt. Oft werden dabei die Nachkommastellen einfach abgeschnitten. Positive Zahlen werden so abgerundet, negative Zahlen aufgerundet. Es können sich deswegen Abweichungen zwischen cast nach float oder double und cast nach einem Ganzzahltyp ergeben. --Rôtkæppchen₆₈ 23:23, 6. Jan. 2016 (CET)
OK gut jetzt aber eine Folgefrage, wenn ich als dobule 9.9999996 habe wird, wenn ich nach float caste, daraus 10.0, wenn ich jedoch als double 9.9999995 habe und ich nach float caste, erhalte ich 9.999999, müsste da nicht gemässe Rundung#Mathematisches_Runden die 3. Regel angewandet werden und auf die nächste gerade Zahl, also 10.0, gerundet werden?--Sanandros (Diskussion) 23:48, 6. Jan. 2016 (CET)
Nein, denn bei der Umwandlung double nach float wird auf Binärbrüche gerundet. Im Prinzip werden die untersten 28 Bits der Mantisse verworfen und anhand des nächsten Bits und des aktiven Rundungsmodus entschieden, ob auf- oder abgerundet wird. Die einzelnen Bits sind aber nicht mit den Dezimalstellen übereinstimmend, da zur Codierung einer Dezimalstelle krumme ld 10 = 3,32 Bits benötigt werden. --Rôtkæppchen₆₈ 23:58, 6. Jan. 2016 (CET)
OK thx.--Sanandros (Diskussion) 09:15, 7. Jan. 2016 (CET)
Archivierung dieses Abschnittes wurde gewünscht von: Speravir (Disk.) 01:30, 8. Jan. 2016 (CET)

Text als Kreisfläche

Das ist „Apfelform“; gesucht ist „Kreisform“.
Zum Vergleich: Du meinst sowas?

Gibt es eine einfache Methode, einen Text - horizontale Zeilen - in OpenOffice so anzuordnen, dass er danach etwa eine Kreisfläche ergibt?

Wenn nicht: Wie berechnet man die Anordnung (Zeichen per Zeile) von etwa 1211 Zeichen (Buchstaben + Leerzeichen + Satzzeichen), damit der Text danach etwa die Form einer Kreisfläche hat? Play It Again, SPAM (Diskussion) 23:21, 6. Jan. 2016 (CET)

Du brauchst im Prinzip zwei „negative“ Halbkreise links und rechts, zwischen die Du den Text setzt und dann den Text so darumherumlaufen lässt. Mit Microsoft Word lassen sich die Halbkreise mit der integrierten Malfunktion erzeugen, wahrscheinlich geht das bei OOo/LibO/AOO ähnlich. --Rôtkæppchen₆₈ 00:18, 7. Jan. 2016 (CET)
In LibreOffice geht es so:
  1. Zeichne einen Kreis (mit der Zeichenfunktion). Der ist zunächst blau, du kannst ihm natürlich auch jede andere Farbe oder gar keine geben.
  2. Klick einmal rein und schreib oder paste deinen Text rein.
  3. Klick den Kreis an, Kontextmenü: Text…, Haken bei „Text in Form umbrechen“.
--Kreuzschnabel 07:06, 7. Jan. 2016 (CET)
Im Prinzip: ja, aber die Wörter sollen nicht abgeschnitten (können aber mit Bindestrich umgebrochen) werden.
Der Kreis soll die sonst (hochkant) rechteckige Seitenformatierung ersetzen. Play It Again, SPAM (Diskussion) 08:09, 7. Jan. 2016 (CET)
ähm … genau das bekommst du doch mit „meiner“ Methode. --Kreuzschnabel 08:28, 7. Jan. 2016 (CET)
OK. Das versuche ich dann mal. Danke! Play It Again, SPAM (Diskussion) 08:34, 7. Jan. 2016 (CET)
Besser geht so was natürlich mit einer richtigen Layout-Software. Es muß nicht Adobe InDesign sein, da gibt’s auch freie Lösungen wie Scribus. --Kreuzschnabel 08:52, 7. Jan. 2016 (CET)
Der Vollständigkeit halber: In LaTeX wäre shapepar die bevorzugte Lösung. --132.230.195.196 09:19, 7. Jan. 2016 (CET)
Archivierung dieses Abschnittes wurde gewünscht von: Play It Again, SPAM (Diskussion) 08:34, 7. Jan. 2016 (CET)

Erzählung gesucht: Die Kirche ist nur für die Toten

Mir ist eben eine kurze Erzählung in den Sinn gekommen, in der ich meine ein Pfarrer einer anderen Person darlegt, dass das Geschäft der Kirche nicht misszuverstehen sei, es nämlich allein für die Toten ist. Es werden einzelne Aspekte der kirchlichen Tätigkeit durchgegangen (etwa, dass auch die Bänke in der Kirche für die Toten sind), bevor es zu einem bündigen Ende kommt. Ich meine, es war von einem einigermaßen bekannten Schriftsteller, ich finde es aber nicht mehr. Wer kennt den Text und kann ihn nennen? --Chricho ¹ ² ³ 00:52, 5. Jan. 2016 (CET)

Keine Erzählung, sondern Gedicht: Georg Trakl, Die tote Kirche (1909 ?) ??? mfg, GregorHelms (Diskussion) 08:25, 5. Jan. 2016 (CET)
Link - aber nur lesen, wenn man wirklich gut drauf ist! Play It Again, SPAM (Diskussion) 09:11, 5. Jan. 2016 (CET)
Nein, war definitiv Prosa, nicht Trakl, ein Gespräch. Ich hatte an Tucholsky gedacht, habe es auf textlog.de aber nicht gefunden. --Chricho ¹ ² ³ 13:38, 5. Jan. 2016 (CET)
Ein bißchen in die Thematik geht: Jean Paul, "Rede des toten Christus vom Weltgebäude herab, daß kein Gott sei" (Text hier, da kommt aber kein Pfarrer vor. - Es könnte aus einem Roman von Georges Bernanos (Monsieur Ouine) oder François Mauriac (Le Nœud de Vipères) stammen, vielleicht hat jemand die zur Hand. (Ergänzend sei noch Die Kirche ist tot verlinkt.) --Bremond (Diskussion) 19:39, 5. Jan. 2016 (CET)
Vllt. war es auch kein Pfarrer, sondern ein anderer Kirchenbediensteter. Jean Paul sicher nicht. Die anderen kenne ich nicht, glaube nicht daran. --Chricho ¹ ² ³ 23:17, 5. Jan. 2016 (CET)
Kannst du die Erzählung zeitlich etwas eingrenzen? Ich glaube, in einem moderneren englischen Krimi so etwas in die Richtung gelesen zu haben, aber ehe ich mich durch meine Brandlasten wühle ... Das kann ich mir allerdings gerade nicht verkneifen. --Alnilam (Diskussion) Heute schon gelobt? 23:33, 5. Jan. 2016 (CET)
20. Jhd., recht sicher im Original deutscher Sprache, eher nicht zu spätes 20. Jhd., da die Erzählung eine Ernsthaftigkeit hat, die nur aus einer Zeit zu stammen scheinen kann, als Gläubige noch nicht offen eingestanden haben, dass Frömmigkeit ihnen Lifestyle-Produkt ist, mit dem sie sich ggü. anderen profilieren können. Die Erzählung ist in sich geschlossen, maximal ein paar Seiten (dass sie irgendwie in einen Zyklus eingegliedert ist, ist indes nicht völlig ausgeschlossen), es soll nur ein einziger Punkt gemacht werden (vgl. etwa Kirche und Wolkenkratzer, bloß ist dort der Punkt ein anderer). Ungläubigen Fragen des Gesprächspartners, ob denn nicht … für die Lebenden ist, wird mit Erklärungen geantwortet, dass auch … letztlich nur den Toten dient. Hoffentlich irre ich mich nicht. Danke für den Link. --Chricho ¹ ² ³ 11:42, 6. Jan. 2016 (CET)

Vorurteile als Frage-Antwort Dialog sichtbar gemacht!

Wie lautet der vollständige Dialog, bei dem der Antwortgeber immer in etwa sagt: "Mal sehen, kann sein...? --5.56.225.44 13:21, 6. Jan. 2016 (CET)

Bundespressekonferenz. --195.36.120.125 13:47, 6. Jan. 2016 (CET)

Was bedeutet "xx"?

... hinter einer Facebook-Nachricht? Ich habe schon gegooglet, aber es bleibt mir etwas unklar. 90.184.23.200 18:30, 4. Jan. 2016 (CET)

Küsse? -- FriedhelmW (Diskussion) 18:51, 4. Jan. 2016 (CET)
Richtig, siehe: Liste_von_Abkürzungen_(Netzjargon)#X --Buchling (Diskussion) 19:32, 4. Jan. 2016 (CET)
Wobei man sich das nicht unbedingt als Knutscherei vorstellen muss, mein Bruder schickt mir auch mal "XOXO". --Eike (Diskussion) 20:19, 4. Jan. 2016 (CET)
XOXO ist der feste Begriff Hugs and Kisses, kann also auch als Bruderliebe gelten. So wie hier demonstriert. -- Janka (Diskussion) 02:31, 5. Jan. 2016 (CET)
Netzjargon? Ich kenne "XXX" mit der Bedeutung "Kisses" aus England seit den 1970er Jahren. -- Ian Dury Hit me  08:17, 5. Jan. 2016 (CET)
Ja - und früher (1937) hat man es auch auf Papier so gemacht. Wenn man heute im Web nach XXX sucht, kommt das irgendwie anders 'rüber.
Es wäre interessant, der Frage nachzugehen, warum es xxx ist - und nicht beispielsweise kkk ... Play It Again, SPAM (Diskussion) 09:27, 5. Jan. 2016 (CET)
Sprich xxx als "Exes". Ist phonetisch dichter dran an "Kisses" als "kays" (kkk). -- Ian Dury Hit me  11:32, 5. Jan. 2016 (CET)
Früher(tm) gab's mal eine Webseite freexxxpics.de, dort fand sich -wie zu erwarten- eine Sammlung kostenloser Bilder mit dem Inhalt XXX (und xxx), mit vielen vielen verschiedenen X'en. -- Janka (Diskussion) 11:28, 5. Jan. 2016 (CET)
Das wiederum hat mit den Küssen weniger zu tun, denke ich. Kommt eher von X-Rating. -- Ian Dury Hit me  11:32, 5. Jan. 2016 (CET)
Ich dachte immer, das komme daher, dass das X ein sich zuspitzender Kussmund darstellen soll ... Es gibt allerdings auch andere Theorien. --King Rk (Diskussion) 12:30, 5. Jan. 2016 (CET)
Ab spätestens dem dritten X sind vorherigen nur noch eine «Second Edition». Dabei geht es um zäh-fließenderes als Speichel, gelegentlich auch Küsse, die «nicht mehr auf Augenhöhe erfolgen», Arterhaltung oder so tun als ob. --Hans Haase (有问题吗) 12:57, 5. Jan. 2016 (CET)
Du meinst „zäher fließend“. Der Komparativ „zäh-fließender“ ist Unsinn, da man „fließend“ nicht logisch steigern kann. --Kreuzschnabel 07:38, 6. Jan. 2016 (CET)
Noch zäher-fliesender? Ein Kabarettist fragte sich, ob man Nomen steigern könnte.[13] --Hans Haase (有问题吗) 16:04, 6. Jan. 2016 (CET)

Pflege-Bahr Ablehnung möglich?

Beim Pflege-Bahr dürfen Versicherungen keine Gesundheitsfragen stellen. Wenn sie aber auf anderen Wegen von einem angeschlagenen Gesundheitszustand wissen, können Sie dann (gegebenenfalls mit einem anderen Grund oder unbegründet) einen Antragsteller ablehnen? --89.204.138.195 17:07, 6. Jan. 2016 (CET)

DVB-T-Zimmerantenne für UKW-Radioempfang geeignet?

Ich habe eine Technisat TT1 Zimmerantenne (Amazon), die ich mal für DVB-T-Empfang genutzt habe. Kann ich diese Antenne mit einem Koax-Adapter (2x Kupplung, da sowohl an Antenne als auch am Tuner Koax-Stecker) an meinen analogen Tuner anschließen, um den UKW-Empfang zu verbessern? Am (sehr alten) Tuner befindet sich am Antenneneingang die Beschriftung "75 Ohm FM". --2A00:C1A0:4705:4800:2547:5FD3:17BA:FD96 00:04, 7. Jan. 2016 (CET)

Da steht „Empfangsbereich: VHF Kanal 2 - 12, UHF Kanal 21 - 69“. Der UKW-Hörfunkbereich liegt zwischen Kanals 4 und 5, sollte also von dieser Antenne abgedeckt sein. Die zwei Antennenstäbe sind wahrscheinlich für VHF (Kanäle 2 bis 12, 47 bis 230 MHz) zuständig. UKW 87,5 bis 108 MHz liegt da mittendrin. Die Acht und das Sieb sind für UHF (Kanäle 21 bis 69, 470 bis 862 MHz) zuständig. --Rôtkæppchen₆₈ 00:12, 7. Jan. 2016 (CET)
Klingt gut, danke. Um die 75 Ohm muss ich mir keine Sorgen machen? Es gibt noch ein Rad an der Antenne, mit der ich einen Wert zwischen 0 und 36 dB einstellen kann. Ist das soetwas wie die Empfangsstärke? Ich stelle den Wert also so klein ein, dass die Empfangsqualität immer noch optimal ist, und spare damit dann Energie gegenüber den vollen 36dB? Oder hat das einen völlig anderen Hintergrund? --2A00:C1A0:4705:4800:2547:5FD3:17BA:FD96 00:17, 7. Jan. 2016 (CET)
Die 75 Ohm sind der Normwert für die Impedanz von Antennenanschluss, Kabel und Antenne. Bei Hörfunk sind sie weniger wichtig, bei Analogfernsehen konnten abweichende Werte zu Bildverschlechterung führen. Die Verstärkungsregelung kann nicht dazu beitragen Energie einzusparen, da die Ausgangsleistung einer Zimmerantenne so oder so sehr klein ist. Diesen Steller kannst Du zum Einstellen des Empfangspegels nehmen, bei dem die Empfangsgüte am besten und die Bitfehlerwahrscheinlichkeit am kleinsten ist, also der Punkt zwischen verrauscht und übersteuert. Für den UKW-Empfang musst Du die optimale Stellung bei einem oft gehörten Stereosender nach Gehör einstellen. --Rôtkæppchen₆₈ 00:24, 7. Jan. 2016 (CET)
Danke! Dann mach ich mich morgen auf den Weg und kaufe den genannten Adapter, um es auch in der Praxis zu probieren. 2A00:C1A0:4705:4800:2547:5FD3:17BA:FD96 00:26, 7. Jan. 2016 (CET)

Strafmaße etc. rund um die Kölner (und andere) Überfälle

Hallo!

Leider habe ich bei Facebook erst bei Lutz Bachmann und dann auch bei PEGIDA Schreibverbot bekommen (glaubt es oder nicht: Ich habe nichts Böses geschrieben, keinen Hass gesät - ich habe nachgefragt, was die Damen und Herren Demokraten dort ja immer propagieren, zuletzt bei einer Meldung mit falschem Kommentar zu einem Interview Biedenkopfs und zur Hymne). Daher kann ich dort nicht nach Meinungen forschen, deshalb hier, ohne etwas schönreden zu wollen (die Unterstellung würde mir dort wohl drohen): Wie prüfen Polizei oder Staatsanwaltschaft, ob wirklich alle Opfer der Sexuellen Übergriffe in der Silvesternacht 2015/16 Opfer waren?

Die Berichterstattung ist ja sehr schwierig, da die einen Offenheit erwarten, die anderen selbst bei Offenheit Betrug erwarten: Sprich: Dem einen sind die Opferzahlen zu hoch, dem anderen zu niedrig.

Gerade auch weil so etwas noch nie vorgekommen ist ist es auch schwer einzuschätzen, was da noch alles kommt.

Als Demagoge würde ich da ja doch auf die Idee kommen, ein paar meiner Schäfchen anzustacheln, um die Opferzahlen noch höher werden zu lassen (nicht falsch verstehen: Schon ein Opfer war eins zu viel). Die Polizei wird ja nun nicht prüfen, ob das meldende Opfer PEGIDA-Anhängerin ist. Und Lutz Bachmann oder seine Kumpels in Köln und den anderen Städten werden niemanden offen dazu anstiften.

Was droht jemandem, der bei solch einer Falschaussage erwischt wird? Was dem, der sie dazu anstiftet?

Und, andere Seite betrachtet: Wenn dort also jemand aus dieser Menge heraus eine Frau angefasst hat mit dem Ziel, sie abzulenken und ihr etwas zu stehlen: Was wird, sofern er erwischt wird, für das Strafmaß hinzugenommen: Nur der Raub an sich oder dann auch sexuelle Belästigung? (nicht signierter Beitrag von 217.9.49.1 (Diskussion) 14:03, 7. Jan. 2016 (CET))

Zu Opferzahlen und Strafmaßen lesen Sie das StGB und fragen Sie Ihren nächstgelegenen Polizisten oder Staatsanwalt. --Informationswiedergutmachung (Diskussion) 14:06, 7. Jan. 2016 (CET)

Eventuell kommt Schwerer Raub infrage ("Raub als Mitglied einer Bande, die sich zur fortgesetzten Begehung von Raub oder Diebstahl verbunden hat, unter Mitwirkung eines anderen Bandenmitglieds"). --Eike (Diskussion) 14:10, 7. Jan. 2016 (CET)
Siehe für den Bereich des Strafrechts (s.a. Strafrecht (Deutschland)) den Artikel Strafe, insbesondere aber bei mehreren erkannten Delikten auch Strafzumessung (Deutschland) und Gesamtstrafe (die aus den Strafen für mehrere Delikte gebildet wird, wenn diese gemeinsam verhandelt wurden). --2003:45:463D:4600:2D8F:BB30:ECA8:D017 14:23, 7. Jan. 2016 (CET)
Zur ersten Frage: Vortäuschen einer Straftat, zur zweiten Frage: Anstiftung zum Vortäuschen einer Straftat scheint mir nicht strafbar zu sein, zur dritten Frage: Tateinheit. --BlackEyedLion (Diskussion) 15:25, 7. Jan. 2016 (CET)
Wenn die Anstiftung gelungen ist, droht dem Anstifter die selbe Strafe wie dem Täter, § 26 StGB. Lediglich die versuchte Anstiftung wäre straffrei, weil das Vortäuschen einer Straftat "nur" ein Vergehen ist. -- Ian Dury Hit me  08:49, 8. Jan. 2016 (CET)

Als Demagoge würde ich auch auf die Idee kommen, dass die Opferzahlen viel zu niedrig sind, weil Frauen nach spurenloser sexueller Belästigung keine Lust auf Behördengänge haben, die am Ende wahrscheinlich sowieso ins Leere laufen.--Expressis verbis (Diskussion) 17:30, 7. Jan. 2016 (CET)

Ich biete mal die Wette an, daß kein einziger muslimischer (falls sich der eine oder andere Nichtmoslem beteiligt hat, hat er schlechtere Chancen) Täter wirklich bestraft wird. Schlimmstenfalls Geldstrafen, die sie nicht zahlen (können) oder Haftstrafen auf Bewährung. Die mE richtige Maßnahme bei fehlender Aufenthaltsberechtigung - sofortiger Abbruch des Asylverfahrens und Abschiebung - ist nicht einmal theoretisch vorgesehen, wie der EXPRESS am 2. 1. (damals wurde ja wie üblich von "Einzelfällen" berichtet) beruhigend berichtete.--80.129.155.135 13:17, 8. Jan. 2016 (CET)

Wieviel? Zeitrahmen? Wie ist dein richtiger Name, auf dass ich mein Geld dann auch bekomme? --Eike (Diskussion) 13:23, 8. Jan. 2016 (CET)
Öhm, die Bestrafung oder Nichtbestrafung dürfte wenig mit der formalen Religionszugehörigkeit zu tun haben. Das Grundproblem in solchen Fällen ist, dass bei Straftaten aus dem Schutz der Menge heraus der hinreichende Beweis der Täterschaft sehr schwierig, manchmal auch unmöglich ist. Hier noch verschärft durch den Umstand, dass zu wenig Polizei vor Ort war. Solche Ermittlungsprobleme gibt es auch bei Großveranstaltungen immer wieder. Man sollte daraus kein Desinteresse an einer Bestrafung ableiten. Abschiebungen sind bei klarer Beweislage unter Umständen durchaus möglich. Das hängt einerseits vom Status des Täters ab, andererseits vom Herkunftsland. Nach Syrien zum Beispiel wird aus nachvollziehbaren Gründen niemand abgeschoben. Rainer Z ... 19:18, 8. Jan. 2016 (CET)
Da muss man sich auch mal fragen, warum uns die Politiker - fast alle Juristen - immer dieses Märchen von der "Abschiebung von Straftätern" verkaufen wollen. Ja, Ausländer können abgeschoben werden, wenn sie zu mehr als drei Jahren Gefängnis verurteilt wurden, aber natürlich nicht in einem schwebenden Asylverfahren und nicht in das Land, in dem sie mit dem Tode bedroht wurden. Wohin aber wohl sonst?--Optimum (Diskussion) 19:38, 8. Jan. 2016 (CET)
Da käme z.B. ein sicherer Drittstaat in Frage, über den der Ausländer eingereist ist. --Rôtkæppchen₆₈ 21:32, 8. Jan. 2016 (CET)

Ich halte die Frage für so weitgehend wie möglich beantwortet. Wie u.a. aus den verlinkten Artikeln hervorgeht kann man hier nicht über konkrete Strafen sondern nur über mögliche Strafrahmen spekulieren. Alle weiteren und von der Frage wegführenden Diskussionen und Spekulationen sollten wie bei allen von unseren men in mission heißgeredeten Themen wie üblich lieber im Cafe stattfinden. Dies auch unter dem Eindruck des bedenkenswerten Kommentars von Christiane Florin: „Jetzt interessieren die Opfer nur, wenn sie verwertbar im Meinungskampf sind.“ (Christiane Florin: Wir sind alle Frau - Kommentar zu den Reaktionen auf die Ereignisse in Köln, Deutschlandfunk, Kultur heute, Sendung vom 8. Januar 2016, Audio). Ich setze also eine Erle. --2003:45:465A:9300:9C16:4AF5:EAC:E69 22:09, 8. Jan. 2016 (CET)

Archivierung dieses Abschnittes wurde gewünscht von: 2003:45:465A:9300:9C16:4AF5:EAC:E69 22:09, 8. Jan. 2016 (CET)

Akustische Rückkopplung bei Hörgeräten mit Antischall beheben?

Wäre es rein technisch (für Hersteller) möglich, das bei der Akustische Rückkopplung in Hörgeräten erzeugte laute Pfeifen durch Antischall technisch zu lösen (d.h. die Rückkopplung ist zwar da, aber der Hörgeräteträger hört das Pfeifen nicht). Kopfhörer mit Antischall gibt es ja schon, aber bei Hörgeräten habe ich dergleiche Technik noch nicht gefunden. --Ratzer (Diskussion) 14:46, 7. Jan. 2016 (CET)

Meine Tante hat so ein Ding mit angepriesener Unterdrückung von Rückkopplung, also müsste es technisch möglich sein, wenn auch die Stöpsel fast 5.000 Euro gekostet haben. Rückkopplung kommt aber nicht von Außen, wie der (Stör-)Schall, den man gegenfiltern kann, sondern tritt auf, wenn Lautsprecher und Mikrofon sich zu nahe kommen und zusammen mit dem Verstärker in Resonanz geraten. Das kann man dämpfen, hab ich auch schon mal gelesen, weiß aber nicht wie es funktioniert.--2003:75:AF0C:E00:5D00:F8A4:7985:916C 15:37, 7. Jan. 2016 (CET)
Hier setzt sich einer damit auseinander http://iem.kug.ac.at/fileadmin/media/iem/altdaten/projekte/dsp/detektion/petersen.pdf --2003:75:AF0C:E00:5D00:F8A4:7985:916C 16:16, 7. Jan. 2016 (CET)
Danke. Ist Deine Tante zufrieden mit der Pfeifunterdrückung? Um welchen Hersteller und welches Gerät handelt es sich?--Ratzer (Diskussion) 16:39, 7. Jan. 2016 (CET)
Die Tante war glücklich mich nun am Telefon zu verstehen, der Hund und die Uhr gingen ihr aber zwischenzeitlich auf die Nerven, ersterer mache laufend Geräusche an das Ticken wolle sie sich gewöhnen. Pfeifen hat sie noch nicht bemerkt, der Hersteller war für sie das Geschäft in der Hauptstraße unten im Ärztehaus. Der sagte mir, das sei von Siemens ein Nitro IT. Der bestätigte mir auch, dass ein Rückkopplungsfilter eingebaut ist.--2003:75:AF0C:E00:5D00:F8A4:7985:916C 17:32, 7. Jan. 2016 (CET)
Besten Dank, diesen Infos werde ich mal nachgehen.--Ratzer (Diskussion) 21:07, 7. Jan. 2016 (CET)
Archivierung dieses Abschnittes wurde gewünscht von: Speravir (Disk.) 00:50, 9. Jan. 2016 (CET)

Innere Medizin: Pylorus

in der Inneren Was bedeutet Pylorus --217.85.43.223 15:23, 7. Jan. 2016 (CET)

Hast du den Artikel Pylorus schon gelesen? --Wrongfilter ... 15:33, 7. Jan. 2016 (CET)
Archivierung dieses Abschnittes wurde gewünscht von: Speravir (Disk.) 18:11, 8. Jan. 2016 (CET)

endowment - Genuss oder Furcht: Selbstwert und antizipierter Nutzen

“The biggest issue is editor diversity,” says Wales. (dtsch. Der wichtigste Punkt ist die Vielfalt der Autoren.) Wie ist endowment in einer Arbeit über Selbstwertgefühl und antizipierter Nutzen, Identität, Moral und Tabus (engl. self-esteem (SE) or anticipatory utility (AU). doi=10.1093/qje/qjr002) zu verstehen? PDF 300 kB, Seite 813 Danke, --Edward Steintain (Diskussion) 20:40, 7. Jan. 2016 (CET)

Der volle Begriff ist hier initial endowment = Anfangsausstattung. --King Rk (Diskussion) 00:48, 8. Jan. 2016 (CET)
en:wikt:endowment und en:wikt:endow sind hier unvollständig. Ich würde endowment hier zwanglos mit Ausstattung übersetzen. --Rôtkæppchen₆₈ 00:55, 8. Jan. 2016 (CET)
Danke für die sehr guten Hinweise. --Edward Steintain (Diskussion) 07:56, 8. Jan. 2016 (CET)
Archivierung dieses Abschnittes wurde gewünscht von: Speravir (Disk.) 00:49, 9. Jan. 2016 (CET)

Weiß man eigentlich, wie sich Zugvogel-Schwärme zum Losflug ins Winter/Sommergebiet "absprechen"?

Weiß man eigentlich, wie sich Zugvogel-Schwärme zum Losflug ins Winter/Sommergibt "absprechen"? Damit meine ich: Wie wird das Signal gesetzt ("heute geht's los"), dass es jetzt wirklich los geht zum kontinuierlichen Flug in den Süden/Norden, und nicht nur eine kleine Runde bis zum nächsten Weiher? Das also nicht etwa die hälfte des Schwarms denkt: Ach ne, warten wir noch bis morgen. Oder kommt das vor? --Zulu55 (Diskussion) Unwissen 15:20, 6. Jan. 2016 (CET)

Ich glaube nicht, dass Vögel denken. Aber dumm sind sie auch wieder nicht. Wenn ich morgens meine drei Schippchen Erdnüsse ins Futterhäuschen kippe, ertönt ein schriller Pfiff und die Meute fliegt an. Und wenn ich die Meute der Spatzen beobachte, dann gibt es eine strenge Hierarchie, mit Ober- und Unterbossen. Solche Schauspiele wie Merkel und Seehofer gibt's da nicht da wird das gemacht, was der Chef pfeift.--2003:75:AF0A:E200:25C7:3AB:F209:EF3E 16:22, 6. Jan. 2016 (CET)
Knaller-Frage!
Nach 30 Min. Suche mit Variablen ("Bird migration", "triggered departure", "synchronized departure", "preflight signalling" und so weiter) liefert folgende Ergebnisse:
  • Unterschiedliche Spezies machen es in unterschiedlicher Weise - das überrascht uns nicht, oder?
  • Es gibt „übergeordnete Faktoren“ (Klima, Futter, Licht,...) und dann „Startfaktoren“.
  • Klima (Sonne, Wind etc.) spielen eine „übergeordnete Rolle“, da mit verändertem Klima ein verschobenes Wanderverhalten beobachtet wird - das überrascht uns nicht, oder?.
  • Einige Zugvögel ziehen nicht, wenn ein anderer „übergeordneter Faktor“ (z.B. weiterhin gutes Nahrungsangebot) eintritt (oder wenn der Jungvogel beim ersten Mal daran gehindert wird, abzufliegen).
  • Und hier ein lieblos hingeworfener ;-) Suchvorschlag => Sociable schedules: interplay between avian seasonal and social behaviour <= (kurz: => 2006AnimBehavHelm.pdf <=). Da wird "Active communication (signalling) of seasonal disposition involves complex systems of visual, vocal and possibly olfactory cues." diskutiert (mit Beispielen hier und da, aber "Es gibt nichts Generelles", ausser "dass es komplex ist".
Also: Obwohl die Dinger weniger Hirn haben als das Volumen einer Walnuss, kriegen sie es perfekt hin, als sozialer Schwarm ihr Überleben zu sichern.
Ich schick' jetzt mal den Artikel ans Europaparlament. Piep! Play It Again, SPAM (Diskussion) 19:48, 6. Jan. 2016 (CET)
Zugunruhe --Expressis verbis (Diskussion) 02:13, 7. Jan. 2016 (CET)
Danke an Play It Again, SPAM und Expressis verbis

Vertragsverletzungsverfahren

Deutschland habe aktuell 41 Vertragsverletzungsverfahren am Hals. Ist das ein Rekord, oder gibt es noch lausigere EU-Mitglieder? Ich finde dazu nichts in der Wikipedia. --62.202.181.176 15:49, 6. Jan. 2016 (CET)

müsste sich hieraus ergeben. Italien? --gdo 15:54, 6. Jan. 2016 (CET) Etwas älter hier - Mittelfeld. --gdo 15:56, 6. Jan. 2016 (CET)
Wir haben Vertragsverletzungsverfahren, hatten Rechtsschutz (EG), und würden uns über eine Liste der Vertragsverletzungsverfahren nach Land und Anlass bestimmt freuen. --Hans Haase (有问题吗) 15:57, 6. Jan. 2016 (CET)
Das ist ja wirklich interessant. Ich habe dem Thema mal hinterhergegraben. Hier gibt es eine Übersicht zum Thema Vertragsverletzungsverfahren. Unten auf der Seite gibt es einen weiterführenden Link zu den Entscheidungen der Kommission zu Vertragsverletzungen. Wenn man die Suchmaske leer läßt, dann erhält man eine Auflistung aller Entscheidungen zu Verletzungsverfahren und kann diese sogar nach Excel exportieren. Leider hab ich noch keinen Weg zu den original Dokumenten gefunden. --mw (Diskussion) 09:11, 7. Jan. 2016 (CET)

Flüssiger Isolierstoff für Kabel

Hallo Experten. Wer hat Erfahrung beim Isolieren von Stromkabel im Fahrzeugbau. Immer wieder gibt es Kontaktprobleme, wenn Feuchtigkeit dazu kommt. Gibt es hier einen flüssigen Stoff, mit dem man nach dem fertigen Anklemmen die Verteilerdose füllt. Dieser soll dann aushärten und auf Dauer isolieren. Die Frage wird bestimmt viele Nutzen beschäftigen. Ich dachte an metallfreien Flüssigkunststoff oder ähnliches. Gibst das? Viele Grüße und Danke für die Antwort. (nicht signierter Beitrag von Feilmeuer Josef (Diskussion | Beiträge) 14:47, 6.Jan 2015)

Vergussmasse. --Rôtkæppchen₆₈ 16:43, 6. Jan. 2016 (CET)
(BK) Das geht mit Gießharz wird z.B. bei Erdkabel-Verbindungen angewendet. Hier ist so eine Abzweigdose die zum füllen mit Kunstharz vorgesehen ist. Wie weit es sowas auch im KFZ-Bereich gibt kann ich nicht sagen. --Mauerquadrant (Diskussion) 16:47, 6. Jan. 2016 (CET)
Ich hab so etwas auch schon im Hochspannungs- und Medizintechnikbereich gesehen. Bei Feuchtigkeit dürfen aber keinerlei Ritzen o.ä. vorhanden sein, da die sonst die Feuchtigkeit aufsaugen und zum Dauerproblem werden. Ansonsten eben IP6K7. --Rôtkæppchen₆₈ 17:01, 6. Jan. 2016 (CET
BK Ich habe ein Auto einer namhaften Firma und Erfinder des Automobils, da hat sich der Kabelbaum infolge eines Konstruktionsfehlers durch Kapillarwirkung voll Motoröl gesaugt. Der Anwalt der Firma trug im Gegengutachten vor, dass dieses Öl im Kabelbaum durchaus auch positive Wirkung hätte, es verhindert Feuchtigkeitsschäden und bat das Gericht das im Urteil zu berücksichtigen. Hat nix geholfen, der Kabelbaum musste erneuert werden, will aber damit sagen, dass es spezielle Kriechöle gibt, die durchaus und lange einen Feuchtigkeitsschutz bilden, wenn man die Kontakte und Stecker damit einsprüht. Das Vergießen mit Kunststoff hat einen Nachteil bei notwendigen Reparaturen. --2003:75:AF0A:E200:25C7:3AB:F209:EF3E 17:03, 6. Jan. 2016 (CET)
Im billigsten Fall sprüht man die fraglichen Verbinder in trockenem Zustand satt mit WD40 ein. Das kriecht dann überall hin,. wo es hinkommt und verdrängt etwaige Feuchtigkeit. --Rôtkæppchen₆₈ 17:07, 6. Jan. 2016 (CET)
Möglicherweise verbindest Du zwei verschiedene Metalle und hast dadurch ein Thermoelement gebaut. Das entsteht wenn Wasserleitungsrohre aus Aluminium mit einem aus Kupfer verbunden werden. Im Kabelbaum hat dies stets unerwünschte elektrische Eigenschaften. Zum Schmiermittel: Motor- und vor allem Getriebeöle müssen einen Schmierfilm erhalten und sind auch so aufgebaut. Besonders bei schrägverzahnten Getrieben muss das Öl die Reibung auffangen. Sie haben keine Spezifikationen und Eigenschaften für den Erhalt des Kontaktes elektrischer Steck- oder Schleifkontakten. Dazu gehören der Schleifer eines Potentiometers oder der Schleifring zwischen Lenksäule und Lenkrad für den Kontakt der Hupe im Lenkrad. Öle für elektrische Kontakte haben hingegen nicht die Eigenschaften unter diesen Bewegungsgeschwindigkeiten, Temperaturen und Druckbelastungen den Schmierfilm aufrecht zu erhalten. Sie eigenen sich für eben diese Kontakte, wie sich auch Antrieb des Wischergetriebes vorkommen und den Punkt, an dem der Wischer aufgeräumt ist, durch einen Kontakt erkennen. Die einfachen Kabelschuhe sind jeglicher Witterung, Kondensation, Spritzwasser mit und ohne Straßensalz ausgesetzt. Seit den 1980er Jahren gibt es Stecker, die Silikongummidichtungen enthalten und die Kontakte vor diesen Einflüssen Schützen. Sie sind es, die die Beleuchtung bis auf den Ausfall von Glühlampen zuverlässig machen, aber sich beim Nachrüsten nicht durchsetzen, da die Crimpzangen – das Montagewerkzeug dafür – für gelegentliche Benutzung zu teuer erscheint. Wie die Batteriekontakte aus Blei, macht auch der Anteil an Blei oder Aluminium im Sockelkontakt der Glühlampen ein Problem mit Korrosion. Auch ermöglichen diese geschützten Steckverbindungen die gewagte Konstruktion des VW-Konzerns, Motorsteuergeräte im Wasserkasten des Fahrzeugs unterzubringen. Die zahlenmäßig meisten Steckverbindungen, die der Elektronikeinzelhandel anbietet, sind nicht für den Einsatz im Fahrzeug geeignet oder althergebrachte Technik wie die Kabelschuhe, die es seit Ewigkeiten gibt, aber wenn richtig montiert völlig rüttelfest sind. Daran würde auch das Gießharz im Kabelbaum scheitern und die Kabelzugentlastung sowie den Knickschutz beeinträchtigen. Ein Nachteil kommt stets mit, ob das ausgehärtete Harz steinhart wird oder gummiartig flexibel bleibt. Steinhart ist mit Knickschutz nicht vereinbar, Gummi nicht mit dem Vibrationsschutz von zu fixierenden Teilen. Schneidklemmen zerschneiden unter Vibration das in ihnen montierte Kabel. Selbst Blinkerrelais, die aufgrund der beweglichen Bauteile nicht vergossen werden, können nach Jahren ausfallen, weil die mechanische Last auf den Lötstellen einseitig beschichteter Leiterplatten durch Vibration brechen. Reparierbar sind sie durch nachlöten. --Hans Haase (有问题吗) 17:32, 6. Jan. 2016 (CET)
Mein Gott Haase, warum kannst du nicht kurz und prägnant auf den Inhalt der Frage eingehen und musst über weite und viele unpassende Randbereiche referieren. Damit ist der Thread versaut und für den Frager wertlos.--2003:75:AF0A:E200:BC88:BF56:461A:3164 23:13, 6. Jan. 2016 (CET)
Mit welcher der Antworten lässt sich das Problem des Fragestellers wohl dauerhaft beheben? --Hans Haase (有问题吗) 02:15, 7. Jan. 2016 (CET)
  • Wie Rotkäpchen oder eben Silikon gerade in Form von Silikonharz. Und apropo aushärten, dass ist in einem Fahrzug schlecht, wegen den Vribrationen. Da ist eine Dichtmassen die im geringen Masse weich bleiben sind besser, da sie weniger Riss anfällig sind. Das Problem ist hier eher die Verträglichkiet mit den übrigen Stoffen die im Auto vorkommen, wie Öle, Kühlmittel, Scheibenwischwasser und Treibstoffe. Der Dichtstoff muss das aushalten. --Bobo11 (Diskussion) 23:36, 6. Jan. 2016 (CET)
Im Prinzip wäre also Zweikomponenten-Polyurethan geeignet, weil das nach dem Aushärten elastisch bleibt. Damit hat man aber noch nicht das Ritzenproblem erschlagen. Das zu vergießende Gehäuse und die Kabeleinführungen müssen sich fest mit der Vergussmasse verbinden. Evtl ist also eine Kombination aus elastischer Vergussmasse, IP-Abdichtung bzw Kabelverschraubung und Kriechöl angeraten. --Rôtkæppchen₆₈ 01:06, 7. Jan. 2016 (CET)

Transport eines fremden Reisepasses

Liebe Wikipedianer, darf man eigentlich für jemand anderen dessen Reisepass ins Ausland transportieren? Zum Beispiel: Person A nimmt den Reisepass für Person B, die sich in einem anderen Land befindet, in dieses Land mit, überschreitet also die Grenze unter Verwendung des eigenen Reisepasses und mit einem zweiten, fremden Reisepass im Gepäck. Begeht eine der beiden Personen dabei eine Straftat? Vielen Dank für die Hilfe!--62.225.205.34 18:38, 6. Jan. 2016 (CET)

Sofern Du Dich damit nicht ausweist ist das keine Strafsache. Im Zweifelsfall ist es besser ein Schreiben des Eigentümers dabei zu haben, aus dem hervor geht, dass Du der Überbringer einer Fundsache bist. Z.B. Personalausweis auf Reise abhanden gekommen, Du bringst ihm jetzt den Reisepass vorbei.--Markoz (Diskussion) 18:41, 6. Jan. 2016 (CET)
Deutschland hat damit kein Problem, etliche Länder sehen das jedoch grundlegend anders, z. B. Staaten in Westafrika. Quelle: [14] --Pölkkyposkisolisti 18:50, 6. Jan. 2016 (CET)
wieso schwant mir, dass die Frage vielleicht nicht so ganz präzise gestellt ist...? In der Regel dürfte der Transport eines Passes unproblematisch sein, wobei es Länder geben mag, die das nicht erlauben (was ich mir kaum vorstellen kann, weil "Vollmachten" etc. nahezu überall existieren und die Abholung (und notwendigerweise der Transport) von Identitätsdokumenten durch Bevollmächtigte international -zumindest in Einzelfällen- möglich sein dürfte. Davon unabhängig gibt es aber sog. "Proxy-Pässe" (leider noch kein WP-Artikel, s. aber z.B. [15]), die ggf. nicht überall anerkannt werden und ziemlich regelmäßig auch urkundsrechtlich problematisch sind, wobei man dann ggf. schnell in einen strafbaren Bereich hineinkommt. --gdo 19:04, 6. Jan. 2016 (CET)
Um das nochmal präziser zu sagen: Meine "reine Wissensfrage" dreht sich um stinknormale, echte Reisepässe, die vom jeweilgen Land für den jeweiligen regulären Bürger ausgestellt wurden, nicht um Zweitpässe, vorläufige Exemplare oder gar Fälschungen etc. Gibt es dann auch Antworten ohne Konjunktiv? Vielen Dank!--62.225.205.34 19:21, 6. Jan. 2016 (CET)
die/as deutsche botschaft/konsulat oder eine entsprechende einrichtung eines anderen EU-staates können da gesicherte auskunft geben. im zweifelsfall kann dort ein international anerkanntes ersatzdokument erstellt werden, was den transport des dokuments erübrigt (das ist einer der jobs unserer auslandsvertretungen). und jetzt etwas konjunktiv: eventuell ist es möglich, den reisepass via diplomatenpost an die/as betreffende botschaft/konsulat zu senden, sodass der besitzer es dort abholen kann. --Dirk <°°> ID 30601 20:08, 6. Jan. 2016 (CET)

Jedes Land sein Passgesetz... Die Frage lässt sich so nicht beantworten. Zwei verschiedene Pässe machen dich aber überall auf der Welt verdächtig (sofern sie eben gefunden werden).--Antemister (Diskussion) 20:14, 6. Jan. 2016 (CET)

Ein zweiter, fremder Pass im Gepäck, ist verdächtig, gerade wenn er keinen sehr nahen Familienagehörigen gehört. Das alleine kann je nach Land sehr unangenehm werden (ganz egal ob strafbar oder nicht). Einer Ehefrau mag man noch glauben das sie den Pass ihres Mannes transportiert. Aber eben selbst da muss man aufpassen. In der EU, wo man mit der ID bzw. Personalausweise rumreisen darf, ist es ja noch verständlich, dass eien Familienangehörigen einem der Pass bringen muss, damit man in ein Drittland vereisen kann. Nicht aber in Ländern wo für die Person, der der Pass gehört eigentlich Passpflicht besteht (dieser Pass sich also bei der Person -zu denn du ihn bringen willst- befinden sollte). Es ist also neben der Frege bei wem, auch noch eine Frage nach dem in welchem Landgrenze es sich handelt. In der EU bzw. Schengenraum würde ich das wegen der generellen Strafbarkeit jetzt mal verneinen. Ob es schlau ist steht auf einem anderen Blatt. Bei Leuten die in der selben Haushaltung wohnene bzw. dirkte Blutsverwandte sind mag das noch gehen, mehr wird aber definitv schon kritisch. Und eine Vollmacht ist sicher nei falsch am Platz. Ansonsten gibt es ja auch noch die Post und eingeschriebene Briefe. Mir wäre nicht bekannt, dass es strafbar ist einen Pass zu verschiken.--Bobo11 (Diskussion) 20:57, 6. Jan. 2016 (CET)
Genauso kam es mir eben auch in den Sinn. Danke Boboll! Mir ist ähnliches passiert, Pass zu hause liegen gelassen. Bekannter brachte ihn dann mit. Allerdings noch vor der Grenze, die ich ohne nicht hätte überschreiten können.--Maresa63 Talk 21:16, 6. Jan. 2016 (CET)
Eben man muss hier schon grundsätzlich unterschieden über was für eine Grenze der Pass soll. Eine vollwertige Grenze mit Peronenkontrolle (Schengen Aussen Grenze) oder eine Grenze wo Personenfreizügikeit gilt (Innegrenze Schengenraum). Aber es ist eben auch wichtig, ob es für den Beamten nachvollziehbar ist das du legal in den Besitz des fremden Passes gekommen sein kannst. Da liegt der eigtliche Haken verborgen. Das der Sohn den Pass des Vaters bei sich hat, ist noch nachvollziehbar. Aber es ist verständlich wenn der Beamte hellhörig wird, wenn es sich der Pass bei einer fremde -soll heisen nicht verwannte- Person befindet. Wie schon geschrieben, auch innerhalb Europas würde ich persönlich es per Post machen, gerade wenn es "nur" ein Freundschaftsdiesnt sein soll und von keinem sehr nahen Verwanten. Wenn eilt halt per Express. Per Post ist zwar nicht günstig, aber eher auf der legalen Seite.--Bobo11 (Diskussion) 22:19, 6. Jan. 2016 (CET)

Mehrere Pässe sind nicht unüblich. Weltreisende brauchen das sogar zwingend und das wissen sowohl die ausstellenden Behörden im Heimatland als auch die Kontrollstellen weltweit. Ein offensichtlich fremder Paß, der nicht als Urkunde benutzt wird, bringt in manchen Ländern Ärger und führt mindestens zu Nachfragen. Kann aber im Extremfall auch als Spionage ausgelegt werden, kommt aufs Land an. In Israel, Saudi-Arabien oder den USA würde ich es nicht drauf ankommen lassen. --Pölkkyposkisolisti 00:58, 7. Jan. 2016 (CET)

Habe ich schon gehört, dass die deutsche Polizei Stress gemacht hat, weil sie bei jmd. eine Tasche eines andern mit dessen Ausweis gefunden haben. Das konnte geklärt werden, weil der Besitzer wieder dazu kam. Dass es kein Verbot gibt, heißt nicht, dass man nicht Probleme bekommt. --Chricho ¹ ² ³ 14:53, 7. Jan. 2016 (CET)

Handlungsfähigkeit bei Bevollmächtigten

Sachverhalt: eine Person startet ein Verwaltungsverfahren. Sie erteilt einer anderen Personen eine schriftliche Vollmacht, die diese Person dazu berechtigt, in seinem Namen das Verfahren zu betreiben. Nun erhält der Bevollmächtigte einen rechtlichen Betreuer. Wird er dadurch handlungsunfähig und kann keine Handlungen mehr vornehmen, oder gilt da etwas anderes, weil diese Person nicht Partei des Verfahrens ist, sondern lediglich Bevollmächtigter? -- Liliana 01:53, 7. Jan. 2016 (CET)

Im Zweifelsfall wird das der vorsitzende Richter des Verwaltungsverfahrens entscheiden, insofern es das Verfahren betrifft. Da so oder so einen Anwalt hinzugezogen werden sollte, wäre es das beste, solche Fragen mit dem zu besprechen. --178.4.107.253 02:21, 7. Jan. 2016 (CET)
Wenn ich das richtig verstehe, ist dem Prozessbevollmächtigten im Nachhinein ein Betreuer an die Seite gestellt worden? Grundsätzlich keine gute Voraussetzung für einen Prozess - passtder Richter wird nicht begeistert sein... Dennoch fordert das Gesetz hier lediglich die sog. "Prozessfähigkeit". Diese wird - im Gegensatz zum früheren Recht ("Entmündigung") - durch eine Betreuerbestellung nicht eingeschränkt. Allerdings geht aus dem Sachverhalt nicht hervor, aus welchem Grund eine Betreuung verfügt wurde und ob damit gleichzeitig eine mangelnde Geschäftsfähigkeit festgestellt wurde (§ 104 BGB, Geschäftsunfähigkeit wegen psychischer Beeinträchtigung oder Partielle Geschäftsunfähigkeit etc.). Sollte dies so sein, wäre dies allerdings ein zwingender Ausschlussgrund aus dem gerichtlichen Verfahren, da ansonsten die Klage als "unzulässig" abzuweisen wäre (BGH, Az.: III ZR 306/98; ist zwar ein Urteil im Zivilrecht, im VerwRecht wird dies jedoch nicht anders gesehen) --OnlyMe (Diskussion) 02:22, 7. Jan. 2016 (CET)
Nein, nicht dem „Prozessbevollmächtigten“, sondern dem Bevollmächtigten in einem Verwaltungsverfahren wurde ein Betreuer bestellt, wobei ich davon ausgehe, dass es sich um Liliana selbst handelt, die uns kürzlich von der gegen sie ergangenen Betreuungsanordnung in allen Angelegenheiten berichtete. Wie im Verwaltungs[gerichts-(Schreibfehler nachträgl. berichtigt --Vsop (Diskussion) 12:14, 7. Jan. 2016 (CET))]verfahren VwGO/62 beeinträchtigt eine Betreuung die Handlungsfähigkeit als Bevollmächtigter nur insoweit, als ein Einwilligungsvorbehalt an selbständigem Handeln ohne Zustimmung des Betreuers hindert; das gilt gleichermaßen für den Bevollmächtigten wie für die Partei. Siehe im übrigen auch BVwVfG/14. --Vsop (Diskussion) 04:14, 7. Jan. 2016 (CET)
Ein Verwaltungsverfahren ist nicht gleich ein Verwaltungsgerichtsverfahren; es gibt also auch nicht gleich einen Richter. Bestehende Rechtsverhältnisse werden durch die Anordnung einer Betreuung nicht ohne weiteres beendet; auch erteilte Vollmachten erlöschen nicht automatisch. --Snevern 07:01, 7. Jan. 2016 (CET)

BMW Welt

-->Was kann man und in welcher Reihenfolge in der BMW Welt online zwecks Erstellung einer schriftlichen Arbeit besichtigen? Ich muss nämlich ein Projekt über die BMW Welt machen, um sie anschließend in einer mündlichen Prüfung zu präsentieren.

--82.79.55.77 11:03, 7. Jan. 2016 (CET)

Wie wär's mit hingehen und angucken? --Heletz (Diskussion) 11:06, 7. Jan. 2016 (CET)
... und vorher BMW Welt - mit links/Links - lesen. Play It Again, SPAM (Diskussion) 11:10, 7. Jan. 2016 (CET)

Einen guten Rutsch

Einer nach der anderen!

--2.164.26.173 21:48, 6. Jan. 2016 (CET)

Guten Rutsch? --Buchling (Diskussion) 22:53, 6. Jan. 2016 (CET)
Oder wie man in China sagt: Guten Lutsch ins neue Jahl.--Optimum (Diskussion) 17:36, 7. Jan. 2016 (CET)

Kosten für Briefwahl aus dem Ausland

In knapp zwei Monaten ist es wieder soweit: Landtagswahl in drei Bundesländern. Angenommen, ein Deutscher, der sich für ein paar Monate im Ausland befindet, aber in Deutschland weiter gemeldet ist, möchte bei einer der Landtagswahlen per Briefwahl von seinem Wahlrecht Gebrauch machen (was in der heutigen Zeit leider selbst von körperlich anwesenden Personen nur zu selten in Anspruch genommen wird), wer trägt die Kosten für die Beförderung der Wahlunterlagen ins Ausland und vom Ausland zurück zur Gemeindebehörde (der Kurierdienst von Auslandsvertretungen kommt nicht in Frage)? Ich gehe in beiden Fällen davon aus, dass der Absender das Entgelt zahlt und damit bei der Rücksendung der Briefwähler. Vielleicht übernehmen die Verwaltungen aber auch Kosten für die Rücksendung aus dem EU-Ausland und für andere Staaten nicht? § 36 IV BWahlG, § 40 VI LWO BW und § 21 III LWahlG RLP sind dem Wortlaut nach nur für Beförderung im Inland bzw. besondere Versendungsformen einschlägig. Grüße, --Jakob Gottfried (Diskussion) 21:56, 6. Jan. 2016 (CET)

Es gibt Länder, da ist das Beteiligen an Briefwahlen aus dem Ausland ein Vergehen und Wahlunterlagen werden dort nicht weiterbefördert. Üblicherweise schickt man ansonsten soweit ich weiß solches Zeug nicht mit internationaler Post, sondern an das Konsulat oder die Botschaft im Ausland, dieses gilt jedenfalls für Bundestagswahlen. Die wären auf jeden Fall der Ansprechpartner für diese Frage.--Giftzwerg 88 (Diskussion) 23:22, 6. Jan. 2016 (CET)
Kostenlos verschickt gibt es wahrscheinlich höchstens einen Bußgeldbescheid wegen Verstoß gegen das Meldegesetz.--2.240.89.187 23:26, 6. Jan. 2016 (CET)
Meines Wissens ist bei ausländischen Briefwählern der Briefwähler für ausreichende Freimachung und rechtzeitige Einlieferung des Wahlbriefes zuständig. Es kann durchaus vorkommen, dass ausländische Postverwaltungen unfreie Briefe (Entgelt zahlt Empfänger) nicht weiterbefördern, deswegen ist Freimachung mehr als ratsam. Da der Briefwahlbezirk aber nicht für jedes denkbare Land Briefmarken besorgen kann, muss das der Briefwähler tun. Auch sind Internationale Antwortscheine hier ungeeignet, da diese nur Standardbriefe abdecken und ein Wahlbrief üblicherweise aus einem verschlossenen Umschlag mit den Stimmzetteln, einem Begleitschreiben und der eidesstattlichen Erklärung des Briefwählers besteht. Das passt nicht in eine Standardsendung. --Rôtkæppchen₆₈ 23:39, 6. Jan. 2016 (CET)
Ich finde nichts Substantielles und nehme an, dass sich die jeweiligen LWG und LWO an den Regelungen für Bundeswahlen orientieren. Die gesetzlichen Grundlagen hast du ja schon selbst gesichtet. Ich finde dazu noch einen Artikel in der WAZ. Weiterhin lese ich hier eine für die Landtagswahlen 2012 in Schleswig-Holstein gültige Regelung, die Briefwahlunterlagen auch im außereuropäischen Raum zuzusenden (§ 18 Abs. 5 Satz 3 LWO). Hier noch ein interessantes Gutachten von 2007 zur Portopflicht von Briefwählern. Im Zweifelsfall sollte man wohl bei dem zuständigen Bürgermeisteramt nachfragen. --91.44.73.83 00:23, 7. Jan. 2016 (CET)

Ich habe einmal meinen Briefwahlumschlag rechtzeitig in einer Deutschen Botschaft abgegeben. Das ging dann auf dem botschaftsüblichen Weg nach Deutschland. Ich nehme mal an das das AA zu Wahlzeiten darauf vorbereitet ist. --Eingangskontrolle (Diskussion) 11:18, 7. Jan. 2016 (CET)

Die Befoerderung der Unterlagen durch die Auslandsvertretung ist eine Moeglichkeit, ueber die man sich erkundigen kann (ob sie existiert bzw. bis wann man die Unterlagen abgeben muss), aber keine Verpflichtung. Ich habe aber immer den normalen Postweg gewaehlt mit Frankierung entsprechend dem Format und Gewicht. Nach China dauerte die Befoerderung der Unterlagen zur letzten Bundestagswahl allerdings so lange, dass ich nicht weiss, ob meine prompt zurueckgesandte Stimme noch rechtzeitig ankam, um gezaehlt zu werden. Fuer fest im Ausland ansaessige Deutsche (ich weiss, die Frage bezog sich nicht darauf, aber hier liest ja mancher mit) ist zu empfehlen, sich beim Elefand zu registrieren und anzugeben, dass man entsprechende Mitteilungen erhalten will. Dann erhalt man einige Monate vor der Wahl Informationen und erfaehrt man auch die Modalitaeten der Unterstuetzung durch die Botschaft beim Postverkehr. Der Antrag auf Registrierung zur Wahl bzw. auf den Wahlschein zur Briefwahl muss aber direkt an die zustaendige Gemeindeverwaltung in Deutschland (in dem Ort, an dem man zuletzt in Deutschland registriert war) richten. Die Gemeindeverwaltung verschickt die Unterlagen in jedem Falle kostenlos. -- 83.167.34.67 18:01, 7. Jan. 2016 (CET)

Roter Umschlag! Kosten aus dem Ausland trägst Du. Kleingedrucktes lesen: Falls Du als Briefwahl im Ausland nicht auffallen willst, den roten Umschlag in was auch immer stecken und zeitlich brauchbar versenden. Fragen? --80.187.96.226 17:20, 8. Jan. 2016 (CET)

wie lautet die Aussprache von (Pierre) BouleZ? boulé oder bouléZ ??

--87.173.233.114 12:44, 7. Jan. 2016 (CET)

Mit Summ-summ-s. Grüße Dumbox (Diskussion) 12:52, 7. Jan. 2016 (CET)
So sagt es auch Pierre Boulez: [ˈpjɛʀ buˈlɛz]. --Vsop (Diskussion) 13:45, 7. Jan. 2016 (CET)
Die Angabe von Silbenbetonungen bei franzoesischen Namen verstoert mich immer - das Franzoesische hat keine feststehende Silbenbetonung im Wort.([16]) Der englische Artikel z.B. verzichtet darauf. --Wrongfilter ... 14:35, 7. Jan. 2016 (CET)
Ich halte das Betonungszeichen bei [ˈpjɛʀ] für falsch, das bei [buˈlɛz] dagegen für richtig. „Pierre Boulez“ (ohne weiteren umgebenden Text) wird nun einmal auf der letzten Silbe betont. Dass ein Satz, in dem der Name vorkommt, anders betont wird, wird nicht behauptet. --BlackEyedLion (Diskussion) 15:34, 7. Jan. 2016 (CET)
Im Übrigen würde ich die IPA-Umschreibung nicht zwischen eckige Klammern, sondern zwischen Schrägstriche schreiben (WP:IPA#Unterschiede zwischen phonetischer und phonologischer Transkription). Schließlich geht es nicht darum, wie ein bestimmter Sprecher den Namen spricht, sondern wie er grundsätzlich so gesprochen wird, dass er erkannt wird. --BlackEyedLion (Diskussion) 15:39, 7. Jan. 2016 (CET)
Ich halte es schon fuer richtig mit den Betonungszeichen. Das "Pierre" wird als eigenstaendiges Wort genau so betont wie die zweite Silbe von "Boulez". Eine - . - Struktur ist etwas anderes als . . - 83.167.34.67 17:25, 7. Jan. 2016 (CET)
Man kann Pierre betonen, man kann es auch in die Gruppe Pierre Boulez ziehen und nicht betonen, wie es gerade passt. --Pp.paul.4 (Diskussion) 18:12, 7. Jan. 2016 (CET)
Das stimmt schon. In formaler Aussprache, z.B. der Name in Isolation als akustische Wiedergabe des Lemmas, wuerde ich eher zum betonten Pierre tendieren, zum unbetonten beispielsweise in Aufzaehlungen ("Reinhard Mey, Pierre Boulez und Karel Gott"). -- 83.167.34.67 18:23, 7. Jan. 2016 (CET)

Gesundheit/Herzkrankheiten

Hallo! Weiß jemand genaueres über die Herzkrankheit/Todesursache Herzmuskelentartung (1929), welche Herzkrankheit nach heutigem Begriff würde dem entsprechen? --Verena Mayrbäurl (Diskussion) 14:05, 9. Jan. 2016 (CET)

Das ist wohl ein Synonym für Myokardalteration. --Rôtkæppchen₆₈ 14:09, 9. Jan. 2016 (CET)
+1 So eine Art Sammelbegriff: Hier die genaueren Begriffe zu "Herzmuskelentartung": einmal Myodegeneratio cordis und dann noch Myocarditis chronica Play It Again, SPAM (Diskussion) 14:39, 9. Jan. 2016 (CET)

Spitze, vielen Dank! --Verena Mayrbäurl (Diskussion) 15:12, 9. Jan. 2016 (CET)

Archivierung dieses Abschnittes wurde gewünscht von: Play It Again, SPAM (Diskussion) 23:04, 9. Jan. 2016 (CET)

Lustig? Wikipedia ist der globale Archivar des Wissens

Unlustig – Wikipedia hat das ultimative enzyklopädische Wissen der Welt – immer einen Schritt hinterher. Wikipedia ist ein typischer Fall von (Wissen-)„Haben“ (vgl. SUV)) und nicht Wissen „machen“ → Unglück. Versagt die globale Gesellschaft und Wikipedia beim „Machen“? --Edward Steintain (Diskussion) 20:56, 9. Jan. 2016 (CET)

Nein. -- southpark 20:58, 9. Jan. 2016 (CET)
(BK) Wissen machen=Theoriefindung und die ist unerwünscht. --Mauerquadrant (Diskussion) 21:10, 9. Jan. 2016 (CET)
Archivierung dieses Abschnittes wurde gewünscht von: Frage abschließend geklärt|southpark 20:58, 9. Jan. 2016 (CET)
Bei „Lustig?“ geht es um eine Grundtendenz des Selbstwertgefühls – global und bei Wikipedia: Die freie Enzyklopädie. --Edward Steintain (Diskussion) 21:13, 9. Jan. 2016 (CET)
Mauerquadrant: "Wissen machen=Theoriefindung und die ist unerwünscht." Widerspruch: Das war eine Frage! --Edward Steintain (Diskussion) 21:18, 9. Jan. 2016 (CET)
Nein, das war keine Frage: Wikipedia ist ein typischer Fall von (...) nicht Wissen „machen“. "Wissen machen" ist hier nicht erwünscht - wie bei jeder anderen Enzyklopädie, deren Zwecke es ist, Wissen wiederzugeben. --Snevern 21:24, 9. Jan. 2016 (CET)
Das ist alles Quatsch. Wikipedia ist mit seiner Löschwut ein klarer Fall von Wissen „zerstören“. --Be11 (Diskussion) 21:26, 9. Jan. 2016 (CET)
Ja, das kann man so sehen. Muss man aber nicht. Und vor allem muss man es hier nicht diskutieren - das ist schon tausendmal an anderen Orten diskutiert worden. Hier jedenfalls gehört's nicht her. --Snevern 21:39, 9. Jan. 2016 (CET)
Selbstverständlich nicht. Wikipedia bildet vorhandenes Wissen (wp:Q) ab, schafft aber kein neues (wp:NOR). --Rôtkæppchen₆₈ 22:00, 9. Jan. 2016 (CET)
Es geht um WPIG, Wikipedia Information Generatating. Das ist keine Theoriefindung. Ich bedauere, wie mit Anfragen bei Wikipedia umgegangen wird. Ich neige dazu, mir selbst eine Beitragssperre (vgl. Meidung) von vorerst einer Woche aufzuerlegen: Ein genereller wikipediansicher Fehler gegangen von Mauerquadrant (Systemopfer) wird gemieden (geschannt), so der Vorschlag und die Theorie meines Selbstwerts und Selbstverständnisses. --Edward Steintain (Diskussion) 22:22, 9. Jan. 2016 (CET), in wikipedianischer Tradition der vielen wikipedianischen Apostiker
Weia.
Und warum eigentlich ist unter allen Internetseiten dieser Welt ausgerechnet die WP:AU der richtige Platz für diese Mitteilung? --Snevern 22:56, 9. Jan. 2016 (CET)
Rotkaeppchen68 sagt es, nur gibt es dabei den Filter der Relevanz, der von jedem so ausgelegt wird wie sein eigener Horizont reicht. Da einige sogenannte „WP:Wikifanten“ (ein Kofferwort aus Wikipedianer und einem schwergewichtigen Trampeltier) zuerst da waren und auch Wissenschaftler daran resignieren lassen, dass das frei ergooglebare Wissen nicht das tiefgründigste ist und einige selbsternannte „Nachsichter“ (nicht zu verwechseln mit Nachsicht) die Wikipedia auf den online Verfügbaren Müll reduzieren und sie dadurch «pressehörig» machen, statt belegte Tatsachen zusammenzutragen. Welche Gemeinsamkeit das mit Gleichschaltung haben mag, bliebe jedem für sich selbst zu urteilen. Das Problem dabei, es ist nicht genügende aktuelles Wissen vorhanden, um den Hass sachlich zu widerlegen. Da die Wikipedia Wissen zusammenträgt, unterliegt sie ebenso den Zeitungssterben. Wäre sie tagesaktuell, so würde sie der Sargnagel der Zeitungen selbst sein. Auch wird das unter WP:WWNI das beschriebene HowTo falsch verstanden. Während damit gemeint ist, Artikel allgemein gültig auf Basis von Grundlagenwissen zu schreiben und sich nicht aus der Sicht von Parteien oder Herstellern, ihren Produkten und Interessen, wird dies hingegen exklusionistisch als zu kompliziert oder irrelevant oder mit beliebigen Ausreden dahin kaputtdiskutiert bis nur noch unbrauchbares Wissen übrig bleibt. Mit der ursprünglichen Idee von Jimmy Wales, das Wissen der Welt frei verfügbar zu machen, hat das nicht mehr viel gemeinsam. Das Potential wird wohl erkannt und es wird kritisiert, warum Wikipedia als gemeinnützige Einrichtung im Geld schwimme. Die Wahrheit ist oft unliebsam und das Budget für Rechtsschutz ist untertrieben klein. Auch in Deutschland wurden Aussagen aus der Wikipedia gerichtlich herausgeklagt. Deutschland ist auf der Liste der Länder nach Pressefreiheit nicht an oberster Stelle, auch wird die deutsche Wikipedia nicht auf deutschen Webservern gehostet. Es wäre auch unverantwortlich als marktführende Enzyklopädie mit zu geringem Budget abhängig zu sein. Dies zeigen Phänomene wie Knöllchen-Horst, der erst weniger ernst genommen wurde, nachdem die Stadt Osterode 150.000 Euro mehr in der Stadtkasse hatte oder die Zahl der Justizopfer. Rotlink? Artikel gibt es nicht? Aha, denn so darf man das nicht sagen, denn schön gesagt sind sie: Opfer eines Justizirrtums. Andere Länder, in denen es Wikipedia gibt, haben derartige Artikel nicht. Es ist gerade die führende Rolle, die die Wikipedia spielt und die sie attraktiv macht für Bezahlschreiber, PR-Agenturen und Meinungsmacher, wie der WP:Pressespiegel nicht nur zuletzt wiedergeben kann. Mit letzerem Klientel der der Beitragenden Benutzer geht die deutsche Wikipedia einen anderen Weg als die US-amerikanische, die den angelsächsischen Sprachraum abdeckt. Auch die Diskussionen über den Pressespiegel zeigen, dass Wissen und Realität nicht identisch sind, besonders wenn es darum geht, Leser zu fragen wie das Wissen in die Wikipedia kommt. Das mag desillusionieren, zeigt aber auch auf, dass überall auf der Welt nur mit Wasser gekocht wird, von den strategischen Sicherheitslücken zum Erhalt ganz zu schweigen. Beim Verstehen dieser Zusammenhänge wird der einzelne reifen. Nicht die Wikipedia ist die Quelle des Wissens, sie trägt es nur zusammen. Sie ist nicht resistent gegen den Librizid. Hiervon bekommt sie regelrecht die Vergesslichkeit der Alzheimerdemenz. Gegen gekaufte Wissenschaft oder Studien mit Interessen ist Wikipedia ausgeliefert, unkritisch und einseitig. Ihre Wohltätigkeit am Leser ist das Verringern der Interaktionskosten beim Aufwand Wissen zu erlangen. --Hans Haase (有问题吗) 23:08, 9. Jan. 2016 (CET)
Das nennt man Weltschmerz pur, Haase, was hat man dir nur angetan.--2003:75:AF4E:F000:454:5978:C057:379B 15:07, 10. Jan. 2016 (CET)
Siehe Beiträge und Versionsgeschichten, der Rest ist verlinkt. --Hans Haase (有问题吗) 17:31, 10. Jan. 2016 (CET)

Verhalten von Pedelec-Akkus bei niedrigen Temperaturen

Ich habe bei meinem erst wenige Monate alten Transportpedelec festgestellt, daß die Akkus bei den derzeitigen moderaten Minusgraden ziemlich schnell schlappmachen. Wo finde ich ideologie- und geschäftsinteressefreie zuverlässige Informationen über das Verhalten von Lithium-Eisen-Phosphat-Zellen bei tiefen Temperaturen? (Mein Verdacht geht dahin, daß der Hersteller in die Akku-Elektronik lediglich eine Spannungsüberwachung eingebaut hat, die aber die Temperaturen nicht berücksichtigt, so daß die niedrigere Spannung wegen des erhöhten Innenwiderstands fälschlich als "leerer Akku" interpretiert wird, obwohl noch reichlich Ladung vorhanden ist.)

Zweite Frage: Wären NiMH-Akkus eine Alternative für tiefe Temperaturen? (Das Fahrzeug wird eigentlich gerade im Winter gebraucht, so daß es absurd ist, wenn es ausgerechnet bei Eis und Schnee ausfällt.)

„ideologie- und geschäftsinteressefreie zuverlässige Informationen über das Verhalten von Lithium-Eisen-Phosphat-Zellen“ findest Du im zugehörigen Datenblatt. --Rôtkæppchen₆₈ 19:02, 6. Jan. 2016 (CET)
Ansonsten vollkommen identische Räder vom gleichen Hersteller werden in Deutschland mit 250 Watt, in Finnland mit 400 Watt verkauft. --Pölkkyposkisolisti 19:09, 6. Jan. 2016 (CET)
Lithium-Eisenphosphat-Akkumulator gelten schon als relativ Kältefest. Wie wäre es sich einen zweiten Akku zuzulegen Und vor jeder Tour den geladenen zimmerwarmen Akku einzubauen und und den Kalten neu zu laden und aufzuwärmen. Dabei könnte noch eine extra Wärmeisolierung um das Akkugehäuse sinnvoll sein da mit der Akku nicht so schnell auskühlt.--Mauerquadrant (Diskussion) 20:01, 6. Jan. 2016 (CET)
Egal was für ein Akku oder Batterie, es ist sogar physikalisch begrümndbar, irgend wann mögen die Kälte einfach nicht mehr. Und Ja, meist ist der Akku ist dann noch nicht leer, sondern bringt nicht mehr das was dioe Elekrtronik von der Stromquelle erwartet. Aber es ist eben auch Physik, dass Kälte der Energieumwandlung von chemisch gebunderner Energie in elektrische Energie weg im Wege steht. Aus Erfahrung; Bei kalten Temperaturen ist zweit Akku eigentlich Pflicht. Beim Fotografieren gilt eigentlich immer mindestes 2 Akkus. Der eine im Gerät der andere in der Innentasche wo er mit Körperwärme "regeneriert" wird. Dann heist es halt schlicht weg regelmässig wechseln udn zwar bevor die Elektroik ihn als Leer anzeigt sondern schon dann wenn er "angeschlagen" ist (ao 1/2-1/4 anzeigt). Leider funktioniert das mit grossen Akkus nicht wirklich gut. Aber was eben schon viel hilft wenn man den Akku nicht Rad , soll heissen in der kalten Garage, ondern, im warmen Zimmer läd.--Bobo11 (Diskussion) 20:41, 6. Jan. 2016 (CET)
Also: Es ist ein Transportrad. Es hat standardmäßig schon zwei parallel betriebene Akkus 36 V/9 Ah. (Wobei "parallel" relativ ist: Die werden eben nicht parallel entladen, sondern nacheinander, was beim Nachladen unpraktisch ist, da man auf den leereren warten muß - der Ladestrom ist so unpraktisch niedrig, daß die Ladung 8-10 h dauert.) Die haben aber gar keine brauchbare Ladezustandsanzeige, sondern nur ein paar LED, die offenbar ziemlich unzuverlässig die Spannung indizieren, wozu man absteigen, hinter das Rad treten und ein Knöpfchen drücken muß. Und es gibt einen zweiten Akkusatz, der als "Gepäck" mitgeführt wird (womit ich dann netto an die 20 kg Akkus transportiere) und reduziert die Nutzlast, für die das Rad angeschafft wurde. Und trotzdem sinkt die nutzbare Kapazität beider Sätze zusammen auf ca. 1/4 der normalen Reichweite eines Akkusatzes unter normalen Bedingungen (was teilweise auch auf schwierigere Fahrbahnzustände zurückzuführen ist), und das ist offensichtlich Kernschrott und kostet nebenbei auch noch Geld durch entgangene Aufträge.
Und ich will jetzt nicht wissen, welche "Krücken" ich anwenden könnte, sondern belastbare Quellen über das grundsätzliche Verhalten dieses Akkutyps (z. B. Kennlinien usw.), und es interessiert mich einen Scheißdreck, welche Motorleistungen Pedelecs in Finnland haben, denn ich bin in Deutschland. Bei Kälte steigt offensichtlich bzw. vermutlich der Innenwiderstand der Zellen - schön (bzw. unschön), ist gut vorstellbar, aber kein prinzipielles Problem, da man eben "nur" genügend viele Zellen parallel zu schalten brauchte. Aber wie stark steigt denn der Innenwiderstand, wo kann ich sowas nachschlagen? Und was wären Alternativen?
NiMH ist einerseits billiger, andererseits sind die schwerer. Aber wie verhalten sie sich im Vergleich bei niedrigen Temperaturen? Und es kann doch eigentlich nicht sein, daß Akkus prinzipiell bei zweistelligen Minustemperaturen gar nicht mehr funktionieren und bei einstelligen nur sehr eingeschränkt - Kfz fahren schließlich auch bei arktischen Temperaturen.
Tipp für Antworten: Man spart Zeit, wenn man für den Fall, daß man zur Frage nichts beizutragen hat, auch einfach nichts schreibt.
Na klar, geschäftsinteressefrei im Datenblatt des Herstellers. Ganz bestimmt. (Zum Glück weiß ich nicht, wo ich das Datenblatt finde - BMZ rückt jedenfalls keine Informationen heraus, Heinzmann auch nicht, und der Fahrradhersteller (MIFA) hat keine Ahnung.) (nicht signierter Beitrag von 92.228.251.190 (Diskussion) 16:33, 7. Jan. 2016 (CET))

Ich frage mich wirklich, ob es dir neu sein kann, dass Batterien bei kalten Temperaturen grundsätzlich in die Knie gehen. Ist doch auch ein Klassiker bei Autobatterien. Und die werden nur zum Starten wirklich benötigt. Rainer Z ... 19:57, 7. Jan. 2016 (CET)

Eigentlich haben alle Batterien ein Kälteproblem, da die Leitfähigkeit des Elektrolyten und damit der Innenwiderstand der Zelle stark temperaturabhängig ist, siehe Elektrische Leitfähigkeit#Temperaturabhängigkeit letzter Absatz. Der Innenwiderstand der Batterie müsste also durch konstruktive Maßnahmen verringert werden. Da gibt es einige Möglichkeiten, die die Batterie aber leider entweder teurer, größer und schwerer oder aber weniger haltbar machen, sowohl was Selbstentladung, als auch was Zyklenhaltbarkeit betrifft. --Rôtkæppchen₆₈ 21:31, 7. Jan. 2016 (CET)
Die batterie könnte sich selbst heizen aber ich denke, da kommt unterm Strich nicht viel bei raus. --Pölkkyposkisolisti 21:47, 7. Jan. 2016 (CET)
Auf die Idee war ich auch schon mal gekommen. Nur wie sieht die Bilanz aus? Und könnte man das kombinieren mit einer guten Wärmeisolation? Rainer Z ... 12:54, 8. Jan. 2016 (CET)

Wenn DJs live auf großen Bühnen auflegen

Es gibt ja Konzerte auf denen DJs quasi live auflegen, eine große Bühne haben und richtig wie andere Musikerkünstler auch gefeiert werden. Aber anders wie diese findet ja eher kaum eine Performance statt. Der DJ legt seine Sachen auf, schwingt vielleicht noch ein wenig die freie Hand und gut (einfach ausgedrückt). Wohin gegen Sänger oftmals mehr Präsenz haben/zeigen, sich mehr bewegen, die Showelemente eine größere Bedeutung haben, etc. Und das würde ich gerne verstehen: warum bekommen auf manchen Festivals/Konzerten eine so große Bühne obwohl man sie dann hinter ihrem Pult eh kaum sieht, wie manch andere Musikkünstler, der quasi ein "richtiger Musiker" (in Anführungszeichen!) ist? Gerade da würde sich ja auch ein Playback anbieten. Bzw. was spräche da eigentlich dagegen. Bei einem "richtigen Sänger" wäre das denke ich das Erleben der Live-Stimme. Bei einem DJ ist es ja eigentlich maximal das er da irgendwelche Fehler reinmachen kann (die aber quasi gleich klingen als würde er seine CD abspielen). Oder variieren DJs bei Live-Auftritten auch ihre Songs? Ich würde das gerne verstehen und freue mich über informative Antworten. --87.140.192.144 21:42, 6. Jan. 2016 (CET)

Klar tun wir das... o.o... wäre schlimm, wenn nicht... stell Dir vor, Du gehst in nen Club und da spielt an jedem Abend der gleiche Mix... DJing ist weit mehr als nur Lieder aneinanderreihen.. gute DJs mischen zwei, drei, drölfzig Lieder ineinander und nehmen Dich mit auf eine Reise... was komplett anders als Dein WinAMP oder meinetwegen Virtual DJ-Auto DJ zu Hause macht ist. --Odeesi talk to me rate me 21:50, 6. Jan. 2016 (CET)
Fällt mir grad noch so ein. Klar gibt es DJs, sogar populäre, die während einem Auftritt Pre-Mixed CDs abspielen (Beispiel 1), bei manchen erschrickt man sich, dass sie richtig krasse Fehler bzgl. Basics machen Beispiel 2), andere machen sich aber einfach nur nen Spass (Beispiel 3)... aber das DJs keine Musiker sind.. uhm.. ja... Guetta is keiner, der hier allerdings schon, die Beiden sowieso. --Odeesi talk to me rate me 00:50, 7. Jan. 2016 (CET)
Es wird gemacht, weil Menschen das so wollen (/dafür bezahlen). Muss man nicht verstehen, ist aber so.--Wikiseidank (Diskussion) 08:21, 7. Jan. 2016 (CET)
Ja, in Clubs, das ist ja was anderes. Da ist ein DJ nötig. Die machen ihre Arbeit, manchmal sogar gute und das alles eben an ihrem Arbeitsplatz. Aber, ich weiß nicht, habe aber den Eindruck, in den letzten Jahren, da kommen DJs auch immer mehr als alleiniger Part auf große Bühnen und spielen ihre Songs. Die spielen da nicht fremde Songs ab, sondern die eigenen. Und das verstehe ich da eben nicht. Sie singen nicht, sie tanzen nicht wirklich, sondern legen quasi ihre eigene CD auf (und mixen natürlich auch die Übergänge oder Effekte rein oder keine Ahnung). Aber wie konnte das so ein großes Ding werden, dass sie das auf riesigen Bühnen mit tausenden Zuhörern auf irgendwelchen riesigen Open-Air-Festivals machen? Was ist deren Verdienst (ohne deren Arbeit jetzt klein reden zu wollen, mir gefallen solche Lieder ja auch)? So ganz verstehe ich das eben nicht... --87.140.195.3 19:44, 7. Jan. 2016 (CET)
Ja einige/mehr werdende "DJs", dann sind es keine DJs - siehe Paul Kalkbrenner, spielen eigene Sachen. Das dann aber nicht von CD (20. Jahrhundert?) oder MP3, sondern direkt aus der DAW, also aus den zur Verfügung stehenden Tongeneratoren: Instrumenten und Effekten.--Wikiseidank (Diskussion) 08:04, 8. Jan. 2016 (CET)
Was DJs auf Bühnen machen, erlebt man als Zuschauer leider kaum mit. Zunächst hat der DJ, der seine eigenen Lieder spielt, nicht auf CD, die er einfach nur zusammen mischt, das Lied wird mit den Synthesizern tatsächlich live gespielt, dafür werden einzelne Tonspuren ein- und wieder ausgeblendet, Effekte verschiedenster Art benutzt. Das Lied erkennt man, aber es wird jedesmal neu interpretiert. Im Gegensatz gibt es dazu dann tatsächlich auch DJs, die dann auch wenn sie live auf der großen Bühne stehen fertige Lieder mischen und sich dann darauf spezialisieren, diese möglichst optimal ineinerander fließen zu lassen. Ich hab 2014 mehrere Festivals dokumentiert und auch manche DJs bei ihrer Arbeit beobachten dürfen. Wer mir besonders in Erinnerung geblieben ist, ist Kid Simius, der nicht nur tausend Arme zu haben schien, so viele Knöpfe und Tasten drückte er gleichzeitig, sondern auch noch eine absolut mitreißende Show und Bühnenpräsenz ablieferte, außerdem noch viele Instrumente spielte. Ein Youtube-Video, wo man manches sehen kann, findest du hier. (Mir ist aufgefallen, auf dem Konzert wo ich Kid Simius 2014 sah, hatte er noch ein Theremin gespielt, das hab ich in dem Video nicht entdeckt) Ein Gegenbeispiel war 2014 beim Juicy Beats, wo als Hauptact Boys Noize auftrat. Der hatte bei mir den Eindruck hinterlassen, in die zweite Kategorie zu fallen. Live gespielt hat der, wenn dann nicht viel, was trotzdem gut war. Die Lieder waren perfekt hintereinander gereiht, er hat dafür mit umwerfenden Effekten das Publikum begeistert. Gedanken macht sich jeder DJ, welches Lied in welcher Reihenfolge er spielen wird, aber das ist bei Bands nicht anders. Wenn du aus Berlin kommst und auf elektronische Musik stehst, geh nächstes Wochenende zum 3000 Grad Festival im Ritter Butzke, das ist ziemlich klein und dort gibt es massig DJs, die dir nach ihrem Konzert sicherlich Fragen beantworten können. Grüße--188.109.148.111 14:59, 8. Jan. 2016 (CET)
Der Begriff Live-Act ist noch nicht genannt worden? Auf Veranstaltungsplakaten steht meist explizit, welcher DJ die Musik live macht. Mit gewöhnlichem Auflegen von Vinyl-Schallplatten oder digitaler Musik, Übergänge machen und mal die Bässe raus- und reindrehen, hat ein Live-Act nichts zu tun. Wurde oben aber schon ganz gut beschrieben, wie das aussehen kann. --Kapitän Turnstange ¯\_(ツ)_/¯ 15:30, 8. Jan. 2016 (CET)

Gerät DVB-T kompatibel machen

Gibt es eine Möglichkeit, sowas DVB-T tauglich zu machen? Danke


--Emilsinclair2.0 (Diskussion) 18:32, 7. Jan. 2016 (CET)

Wenn's einen HF-Eingang hat: Einen DVB-T-Receiver mit HF-Ausgang dazukaufen. Grüße Dumbox (Diskussion) 18:40, 7. Jan. 2016 (CET)
DVB-T ist bald schon wieder Geschichte. Demnächst gibt es dann DVB-T2 oder vielleicht auch gar nix mehr, wer weiß. Terrestrisch empfängt ja kaum noch einer. Ein Elektronikbastler könnte einen dieser ganz kleinen Scart-DVB-T-Empfänger besorgen und versuchen, die Schaltung in das Gerät reinzufummeln. Wenn's mechanisch passt, kann man die RGB-Ausgänge an den Monitorteil des Fernis anklemmen und den Original-Empfänger totlegen.
Ich halte das allerdings für Unsinn, da bei diesem Erhaltungszustand das Gerät einen Sammlerwert hat. -- Janka (Diskussion) 20:05, 7. Jan. 2016 (CET)
Ohne Eingriff in das denkmalgeschützte Gerät: Eine Set-Top-Box an einen VHS-Recorder und diesen an das denkmalgeschützte Gerät anschließen. Oder einen passenden Tablet vorne auf die Röhre kleben. --Rôtkæppchen₆₈ 20:12, 7. Jan. 2016 (CET)
So wie es ausschaut, ist das ein einfacher japanischer Schwarzweißfernseher. Ein Bekannter hat mal einen Videorecorder an einen derartigen Fernseher angeschlossen, indem er ihn an der Videodemodulatordiode für das Bild und am FM-Demodulatorausgang für den Ton angezapft hat. Wegen der unterschiedlichen Polarität von Basisband-FBAS-Signal (schwarz = 0V, weiß = 1V, ultraschwarz=−0,3V) und demoduliertem TV-Signal (z.B. ultraschwarz=1V, weiß=0,1V) muss da noch ein invertierender Verstärker mit DC-Versatz dazwischengeschaltet werden. --Rôtkæppchen₆₈ 09:42, 8. Jan. 2016 (CET)

Einsatzkräfte in Köln

Wieviel Prozent der Einsatzkräfte waren Frauen? --2A02:908:F358:46C0:EDC1:C172:934D:978E 08:53, 8. Jan. 2016 (CET)

Müssten ca. 15% sein (Quelle,wenn Frage auch nicht Berlin) Reichts nicht langsam mit der aktuellen "Sau"?--Wikiseidank (Diskussion) 09:13, 8. Jan. 2016 (CET)

Bild in Kaminzimmer

Das Titelbild eines Kriminalstückes [17] zeigt eine Spielszene mit zwei Personen in einem stilvoll eingerichteten Kaminzimmer, wobei die Personen womöglich nachträglich hineinkopiert wurden. Das auffällige Bild über dem Kamin, vermutlich entweder ein Gemälde oder ein glänzendes Metalrelief, ist m.E. eine stilisierte Version des Motives von Exekias' Vasenmalerei Achilles und Ajax beim Brettspiel File:Akhilleus Aias MGEt 16757.jpg.

Das hier ist ein "Standardwappen", in das jeder Wappenhalter sein Wappen einsetzen kann: Symmetrisch, unten "drei Eier", Mitte "Kessel", am Rand "geschweifte Dinge" - sieht aus, wie das im S/W-Bild, oder?

Gibt es das Kaminzimmer oder das Bild über dem Kamin in der Realität? Rosenkohl (Diskussion) 13:22, 8. Jan. 2016 (CET)

Warum nicht? Schottische Hotels haben solch grossen Kamine - würde aber doch eher da ein Wappen darüber erwarten ( ... ehemaliges Herrenhaus oder Schloss...? Bildersuche mit => "fire place" scotland <= liefert fast ausschliesslich Wappen über den Kaminen {Was wäre die Verbindung zw. griechischer Mythologie und Winterabenden in Schottland?}). Steht im Buch nicht, wer den Umschlag entworfebn hat? Man könnte auch nach dem Kamin + Dachartiger Aufsatz suchen und nachsehen, ob GBS etwas Brauchbares ausspuckt. Play It Again, SPAM (Diskussion) 13:42, 8. Jan. 2016 (CET)

Frage Produktekalkulation

Eine Frage zur Produktekalkulation, für die wir einfach keine Lösung finden: Verkaufserlös minus Einstandspreis = Bruttogewinn, Bruttogewinn minus Gemeinkosten = Reingewinn. Sei der Bruttogewinn 400 Euro, der Gemeinkostenzuschlag 25% vom Einstandspreis, der Reingewinnzuschlag 10% von (Einstandspreis plus Gemeinkosten). Wie hoch ist der Verkaufserlös in Euro? --62.202.181.162 10:03, 8. Jan. 2016 (CET)

V-L=B, B-G=R, B=400, G=L*25/100, R=(L+G)*10/100 führt zu V~~1466.7 and R~~133.33 and L~~1066.7 and G~~266.67 (mit WolframAlpha)... *kicher* --Heimschützenzentrum (?) 10:37, 8. Jan. 2016 (CET)
Danke. Mit L meinst du wahrscheinlich den Einstandspreis. Warum kicherst du? --62.202.181.162 11:03, 8. Jan. 2016 (CET)
1. ja genau... L soll der Einstandspreis sein... 2. das Kichern kommt von den krummen Zahlen... normal sorgt der Lehrer doch dafür, dass im Ergebnis alles natürliche Zahlen sind... oda? --Heimschützenzentrum (?) 11:47, 8. Jan. 2016 (CET)
ach so: 3. zum Kichern ist vllt auch, dass ich das Wörtchen „-zuschlag“ jeweils weggelassen habe... sonst wusste ich nich was gemeint war... --Heimschützenzentrum (?) 13:08, 8. Jan. 2016 (CET)
Lösung von Hand: zunächst das Gleichungssystem ordentlich aufschreiben:
 1 B +     1 E +    0 G + 0 R + -1 V =   0
-1 B +     0 E +    1 G + 1 R +  0 V =   0
 1 B +     0 E +    0 G + 0 R +  0 V = 400
 0 B + -0,25 E +    1 G + 0 R +  0 V =   0
 0 B +  -0,1 E + -0,1 G + 1 R +  0 V =   0
Dann das Gauß-Verfahren anwenden, also von oben nach unten die Gleichungen so addieren, dass die Koeffizienten von links nach rechts verschwinden. Am Ende steht 1V=x da. (Wenn nur eine Größe gesucht wird, ist es hilfreich, die als rechtestes zu schreiben. Dann ergibt sich ihr Wert als erstes. Die anderen Werte müssen durch Einsetzen von unten nach oben bestimmt werden.) Wer mit Matrizen rechnen kann, kann das Gleichungssystem als Matrixgleichung schreiben.
Eingabe in WolframAlpha für dieses Lösungsverfahren: row reduce {{1, 1, 0,0,-1, 0},{1,0,-1,-1,0,0},{1,0,0,0,0,400},{0,-0.25,1,0,0,0},{0,-0.1,-0.1,1,0,0}} --BlackEyedLion (Diskussion) 13:39, 8. Jan. 2016 (CET)
Das geht sicher einfacher. Wir benutzen im Unterricht keine Gleichungssysteme, Gauß-Verfahren und Wolfram-Alpha. --62.202.181.162 13:57, 8. Jan. 2016 (CET)
der „einfache“ Weg ist: man löst eine Gleichungen nach X auf und ersetzt in den übrigen Gleichungen dann X... so reduziert sich das Gleichungssystem immer weiter... das lernt man noch vor der 11. Klassenstufe, oder? --Heimschützenzentrum (?) 14:08, 8. Jan. 2016 (CET)
Wir benutzen im Unterricht keine Gleichungssysteme. --62.202.181.162 14:13, 8. Jan. 2016 (CET)
Nimm User:Homer Landskirtys vorletzte Gleichung G=L*25/100, setze sie in die letzte Gleichung R=(L+G)*10/100 ein. Das Ergebnis und die vorletzte Gleichung G=L*25/100 setzt Du in die zweite Gleichung B-G=R ein. Löse nach L auf. Setze die erhaltene Gleichung für L in die erste Gleichung V-L=B ein und löse nach V auf. --Rôtkæppchen₆₈ 18:13, 8. Jan. 2016 (CET) Zur Kontrolle: Ich komme auf , was mit User:Homer Landskirtys Wert übereinstimmt. --Rôtkæppchen₆₈ 18:19, 8. Jan. 2016 (CET)
Du schreibst dein Kalkulationsschema auf und dann rechts davon in der Reihenfolge (A) bis (F) die Spalten mit den in der Aufgabenstellung enthaltenen Informationen bzw. den daraus abgeleiteten Größen:
(A) (B) (C) (D) (E) (F)
Verkaufserlös 1467
- Einstandspreis 100 % 80 % 1067
= Bruttogewinn 400
- Gemeinkosten 25 % 20 % 267
= Reingewinn 10 % 133
Ganz ohne (lineares) Gleichungssystem (mit 5 Gleichungen [das ist das was in der Aufgabenstellung steht] und 5 Variablen [das ist das was in der Spalte ganz links steht], weswegen du erwarten kannst dass es genau eine Lösung gibt), sondern mit dem Pragmatismus eines (angehenden) Kaufmannes. --Timurtrupp (Diskussion) 18:31, 8. Jan. 2016 (CET)

Google Play Games – Spielfortschritt/Erfolge/etc. ohne App einsehen

Mit der Google Play Games App können Erfolge, Missionen, Erfahrungspunkte, Highscores usw. unter Android eingesehen werden – sofern ein Spiel diese Möglichkeiten nutzt. Da alle diese Informationen mit dem persönlichen Google-Account verknüpft sind und damit auch zentral auf den Google-Servern verwaltet werden, sollte es doch auch eine Möglichkeit geben diese Daten auch ohne eine App einsehen zu können.

Kennt jemand eine Möglichkeit (z. B. eine Webseite) mit der man mit dem Desktop-Rechner auf diese Google-Play-Games-Daten zugreifen kann? -- ζ 10:12, 8. Jan. 2016 (CET)

Auf play.google.com gibt es links ein Menü mit einem Menüpunkt "Meine Play-Aktivitäten". Man muss nur mit seinem Google-Account angemeldet sein, um darauf zugreifen zu können. --TETRIS L 15:30, 8. Jan. 2016 (CET)
P.S.: Ich korrigiere: Offenbar sind dort nur die Dinge sichtbar, die man geteilt hat. --TETRIS L 15:35, 8. Jan. 2016 (CET)

Mühlenkopfschanze

Ich habe eine E-Mail an den SC Willingen geschrieben, ob ich vom ersten bis zum heutigen alles zum Schanzenrekord der Mühlenkopfschanze kommen kann? Dann haben sie geschrieben das ihr gucken soll: ---> www.weltcupschanze-willingen.de. Ich finde dort keine komplette Liste. Außerdem habe ich schon bei Google nichts gefunden, außer die hier ---> Mühlenkopfschanze auf Skisprungschanzen.com aber die ist nicht vollständig. Kann mir jemand helfen? --Auto1234 (Diskussion) 18:37, 8. Jan. 2016 (CET)

Ich finde nur entsprechende Informationen in der Unterseite Media bzw. einer dort verlinkten PDF-Datei (die aber keine weiteren Infos zu bieten scheint). Mehr gibt es dann wohl nicht. --Speravir (Disk.) 00:45, 9. Jan. 2016 (CET)

Wie viele Leute arbeiten im polnischen Bergbau?

Hallo Liebe Leute, ich möchte mich erkundigen wie viele Menschen in Polen im Bergbau arbeiten. Ich habe bereits aufrichtig recherschiert , bin jedoch nur auf eine sehr alte Zahl gestossen, was englische und deutsche Quellen angeht. Hat jemand von euch gegebenfalls Informationen darüber?--93.237.47.14 01:11, 9. Jan. 2016 (CET)

2009 waren es 183.400, davon 142.200 öffentlich und 41.200 privat.[18] Seite 257 --Rôtkæppchen₆₈ 01:24, 9. Jan. 2016 (CET)
2014 waren es 160.200.[19] Seite 242 --Rôtkæppchen₆₈ 01:30, 9. Jan. 2016 (CET)

Wow besten Dank , echt Lieb das du das zu so später Stunde für mich herausgefunden hast. Sollte dich je ein Wolf fressen werde ich alles tun um dich zu retten. (nicht signierter Beitrag von 93.237.45.142 (Diskussion) 02:16, 9. Jan. 2016 (CET))

Tabulator – Tab-Zeichen und Tab-Taste oder Tastatur-Makro?

Ich habe das Problem, dass beim Einfügen in eine Tabelle stehts die Tab-Zeichen in den Text eingefügt werden, statt mit dem Tab zum nächsten Feld zu springen. Google verwechselt mir die ganzen Begriffe wegen der Doppeldeutigkeit. Gibt es ein Programm, mit dem die Zwischenablage in Tastatureingaben einer virtuellen Tastatur verwandelt? Was mache ich bei LibreOffice bzw. OpenOffice falsch? Details: Es geht um LibreOffice Base, bei LibreOffice Calc funktioniert alles bestens. --Hans Haase (有问题吗) 22:20, 8. Jan. 2016 (CET)

Ins Blaue hinein, weil ich mit dem Programm nicht arbeite: Schon mal Shift+Tab, Ctrl+Tab oder Cmd+Tab probiert? --Jossi (Diskussion) 21:30, 9. Jan. 2016 (CET)
Ja, alles schon versucht, auch \t gehen \n vor dem Einfügen ersetzt, da Enter auch zum nächsten Feld springt. Es wird alles in den Text des Textfeldes umgelenkt. ein SmileysymbolVorlage:Smiley/Wartung/:-(  --Hans Haase (有问题吗) 23:29, 9. Jan. 2016 (CET)
Mit dem Umweg über die Tabellenkalkulation Calc geht es. Die CSV oder Tab-getrennte Datei dort einfügen. Die Tabs werden dort erkannt. Die Tabelle doer der markierte Teil kann in die Tabellenliste der Datenbank in Base per drag and drop rübergezogen werden. Die Datentypen sind dabei noch nicht festgelegt. Den Weg beschreibt dieses Tutorial.[20] Auch lassen sich Tabellen der Tabellenkalkulaiotn von der Datenbank aus als Datenquelle verknüpften. Schöner und logischer für die Formular und Tabellen wäre aber die Möglichkeit, den CSV-Text direkt einzufügen, zumal die Datentypen damit schon vordefiniert werden könnten und ab dem ersten Fehler abgebrochen werden würde. --Hans Haase (有问题吗) 19:18, 10. Jan. 2016 (CET)
Archivierung dieses Abschnittes wurde gewünscht von: --Hans Haase (有问题吗) 19:18, 10. Jan. 2016 (CET)

Domplatte und Kölner Bahnhofsvorplatz

Wer kann es einschätzen oder hat gar amtliche Zahlen, wieviele Menschen passen darauf? Um die 140.000 Menschen?--91.10.41.13 00:21, 7. Jan. 2016 (CET)

Für die Domplatte finde ich 7.000 m², das entspricht 28.000 Straßenbahnstehplätzen. Für den Bahnhofsvorplatz finde ich auf die Schnelle nichts. --Rôtkæppchen₆₈ 00:42, 7. Jan. 2016 (CET)
Der Bahnhofsvorplatz ist grob geschätzt 6.000 m² groß, was 24.000 Straßenbahnstehplätzen entspricht. Zusammen sind das also 52.000 Straßenbahnstehplätze. --Rôtkæppchen₆₈ 00:46, 7. Jan. 2016 (CET)
Jein. Schon im Neufert nachzulesen, daß es so nicht klappt ;) Das würde Tumult bedeuten, die wahre Zahl liegt deutlich darunter. Fehlende optische Fluchtwege würden sofort zur Entfernung von Massen führen. Maximal 1/4, also 13.000 würden sich freiwillig dort kurzzeitig aufhalten. Daß man sehr viel mehr da "einsperren" könnte, steht auf einem anderen Blatt. --Pölkkyposkisolisti 00:51, 7. Jan. 2016 (CET)
Vielen Dank für diese Zahlen. Nu gut... also selbst 10.000 Menschen sind zu Silvester dort nicht gewesen. Davon können doch keine 1.000 antanzende Taschendiebe und Vergewaltiger gewesen sein. Oder? Landesvorsitzender Gewerkschaftspolitik Arnold Plickert haute die Zahl 1.500 im ZDF raus. Wie kann die Polizei 1000 potenzielle Straftäter registriert haben? Liebe Leute, mir ist ja schnell Angst einzuflößen, 60 Anzeigen, davon 15 Sexualdelikte an einem Abend? --91.10.41.13 01:49, 7. Jan. 2016 (CET)
Nicht ablenken lassen vom polizeilichen Geschwafel rund um ihr Totalversagen. Nachträglich von 1500 Tatverdächtigen zu reden ist leicht. Fakt sind 60 Anzeigen (Dunkelziffer natürlich im Hinterkopf). Zahlen über die polizeilichen Aktivitäten wären interessanter, die genannten Delikte müssen Polizeibeamte auch ohne Anzeige verfolgen. Aber da ist eher wenig von GdP und DPolG zu hören. Zuviele Mitglieder keine Zeit gehabt?--2.240.89.187 02:35, 7. Jan. 2016 (CET)

Laut Pressemitteilung waren es 1000 Leute aber nicht 1000 Täter. Unter den 1000 Leuten waren die Täter, das ist ein Unterschied. de Maziere hat ausdrücklich betont, daß es nicht 1000 Täter waren. --Pölkkyposkisolisti 02:38, 7. Jan. 2016 (CET)

Und es können die gleichen Täter mehrmals Täter sein, so dass mehrere Anzeigen auf die selbe Person oder Personengruppe deutet. Möglich ist auch, dass es bei der selben Tat gleichzeitig mehrere Opfer gab, die unabhängig voneinander Anzeige erstattet haben (z. B. schon weil sie sich nicht kennen oder an unterschiedlichen Orten wohnen). Weiterhin gibt es ganz allgemein bei Sexualdelikten eine hohe Dunkelziffer, weil die Opfer sich nicht zu erkennen geben. Es gab also wahrscheinlich deutlich mehr Opfer, als nur die, die Anzeige erstattet haben. Bei Taschendiebstahl sieht es ähnlich aus, da kann man oft nur zur Polizei gehen und sagen: Der Dieb hat mein Geld gestohlen, ich weiß nicht genau wieviel Geld es war und ich habe den Dieb nicht gesehen, auf einmal war alles weg. Unter den Umständen kann man sich eine Anzeige sparen, es kommt eh nichts raus.--Giftzwerg 88 (Diskussion) 13:43, 7. Jan. 2016 (CET)
Aus eigener Erfahrung: Drei kooperierende Taschendiebe im einem Ladengeschäft reichen aus, um dem prospektiven Opfer den Eindruck zu verschaffen, es sei derückend überfüllt ... IMO reichte es schon aus, wenn unter den 1000 bis 1500 Menschen nur ca. ein gutes Dutzend mbis 20 kooperierende Räuber waren, die in der Deckung der passiven Menge die Frauen (vor allem von hinten) bedrängten und beklauten. dann auseinander gingen und sich schnell zu immer neuen Gruppen umformierten. Ob das so geplant war? Oder kam nach den ersten Versuchen die Erkenntnis: Das klappt ja wunderbar, das machen wir mal weiter? Die Angriffe mit Feuerwerk sind jedenfalls nicht so originell, schon vor 40 Jahren habe ich Alkis mit Sylvesterraketen auf fahrende Autos schießen sehen. -- Zerolevel (Diskussion) 17:54, 7. Jan. 2016 (CET)
Zurück zur Ausgangsfrage: Bei einer organisierten Veranstaltung - z.B. einem Konzert - würden die Behörden bei der Berechnung der maximal zulässigen Besucherzahl von 2 Personen pro m² ausgehen. Weitere Daten finden sich im Atikel Stehplatz. Wer es etwas theoretischer mag, kann sich das Dokument Statische und dynamische Personendichte bei Großveranstaltungen ansehen. --84.152.3.96 22:46, 7. Jan. 2016 (CET)
"amtliche Zahlen", zu wieviele Menschen waren zu Silvester auf der Domplatte und dem Kölner Bahnhofsvorplatz, habe ich bisher nicht gelesen. Da es nicht mehr als 10.000 Menschen gewesen sein dürften, sind dann 1000 schon sehr viel!
"In der Neujahrsnacht hatten sich aus einer Gruppe von rund 1000 Männern kleinere Gruppen gelöst, die vor allem Frauen umzingelt, begrapscht und bestohlen haben sollen." "Vorrangig ging es den meist arabischen Tätern um die Sexualstraftaten oder, um es aus ihrem Blickwinkel zu sagen, um ihr sexuelles Amüsement. Ein Gruppe von Männern umkreist ein weibliches Opfer, schließt es ein und vergreift sich an der Frau."
Und es kommt nach "der Kontrollaufgabe an den Grenzen" dem "Kontrollverlust im Inneren" nah. --91.10.17.138 01:29, 8. Jan. 2016 (CET)
Ich glaube, du hast den Text, den du zitierst, missverstanden. "hatten sich aus einer Gruppe von rund 1000 Männern kleinere Gruppen gelöst, die ..." heißt nicht, dass es 1000 Täter gewesen wären, sondern, dass es kleine Gruppen von Tätern waren. --Eike (Diskussion) 09:41, 8. Jan. 2016 (CET)
@91.10.17.138: Was die Behauptung zum "Schweigen der Medien" bei deinem zweiten Link betrifft: Mir scheint das dort als Beispiel genommene Zitat von Zeit Online duchaus objektiv zu sein:
Dabei ist bislang nichts über die Täter bekannt. Es gibt Hinweise, dass es sich um organisierten Taschendiebstahl handeln könnte. Es gibt aber auch Zeugenaussagen, die eine gezielte Erniedrigung von Frauen nahelegen. Die Polizei steht am Anfang der Ermittlungen.
Es werden nur die verschiedenen Aussagen beschrieben. Es wird nicht behauptet, dass jetzt das Eine oder das Andere stimmt. Leider erwartet man heute von den Medien oft, dass sie sofort konkret wissen, was genau passiert ist, das ist aber oft nicht möglich, leider werden dann oft Gerüchte als Wahrheit hingestellt und daran hat man sich anscheinend schon so gewöhnt, dass teilweise von "Schweigen" die Rede ist, wenn es mal nicht so ist. Generell darf man natürlich keinem Medienbericht trauen, was wirklich passiert ist wissen nur die, die dabei waren... --MrBurns (Diskussion) 10:11, 9. Jan. 2016 (CET)

Wie kommt die Luft in den unteren Hohlraum?

Oder herrscht dort absolutes Vakuumm? [21] --95.115.157.123 20:04, 7. Jan. 2016 (CET)

Da wird normale Luft drin sein, und es wird irgendwo einen Riss nach aussen gegeben haben. Nur weil du auf dem Schnittbild keinen siehst, heist das nicht das es keine gibt. Nach dem das Teil aufgeschnitten wurde hat sich die Frage nach den Vakuum eh erledigt. Es wird bei Entstungung wird darin sicher kein absolutes Vakuum geherrscht haben, dagegen spricht einiges in Sachen Physik. Aber es gibt auch noch logische Erklärungen, für die Bildung von Hohlräumen ohne Aussenverbindung. Denn der Hohlraum kann auch durch gasbildende Prozesse gefüllt worden sein. Bei so einem Einschlag eines Projektiils entsteht nicht gerade wenig Wärme. --Bobo11 (Diskussion) 20:31, 7. Jan. 2016 (CET)
Bei der Form des Hohlraums hätte er mit Vakuum keine Chance. Der Außenluftdruck würde ihn sofort wieder zusammendrücken. --Rôtkæppchen₆₈ 21:05, 7. Jan. 2016 (CET)
Ich zögere schon seit Stunden mit einem Widerspruch, weil ich es nicht besser weiß, aber ich habe schon das Gefühl, dass die hohen Impuls-/Energieerhaltungskräfte die Schicht nach außen getrieben haben und da tatsächlich ein Vakuum entstand. Sonst hätte das nicht die Linsenform angenommen und dick und tragfähig geformt wäre die Beule schon um dem Außendruck auch stand zu halten.--2003:75:AF0C:E00:79DC:B71F:78B6:FC38 23:51, 7. Jan. 2016 (CET)
Leider ist da kein Größenmaßstab beim Foto dabei. Es käme hier also auf die Wandstärke der Beule an. Man weiß außerdem nicht, ob das Metallteil vor dem Beschuss gasgefüllte Mikrolunker oder Poren hatte, die ein Vakuum ebenfalls verhindert hätten. --Rôtkæppchen₆₈ 00:00, 8. Jan. 2016 (CET)
Poren oder Vorschädigung im Material würde ich ausschließen, dieser Querschnitt mit einem Hohlraum ist typisch bei Beschuss, besonders wenn der Prüfling (ab einer gewissen Dicke) eingespannt ist und nicht ausweichen kann. Bei Beton führt das z.B. dazu, dass ohne Durchschuss innen ebenfalls ein Krater abplatzt. Die Panzerplatten z.B. sind am wirksamsten, wenn sie locker angebracht werden und die Energie sich dadurch zusätzlich ohne Beule abbauen kann.--2003:75:AF0A:4600:30A2:4742:67DD:B245 08:16, 8. Jan. 2016 (CET)
"Poren/Vorschaedigung ausschliessen": wuerde ich nicht. Das kommt drauf an, wie der Block/die Platte hergestellt wurde: gegossen/extruiert/gewalzt/gezogen... jenachdem ist das Materialgefuege unterschiedlich, und auf die atmosphaerischen Bedinungen waehrend des Herstellungsprozesses (kalt/heiss/Schutzgas/Vakuum...). Was da genau in der Blase drin ist haengt dann noch vom Material ab. Bei so einem Einschlag werden lokal gut und gerne 1000 Grad erreicht. Da geht ordentlich Chemie ab. Ich wuerde erwarten, dass da ein kleiner Burchteil eines Bars verschiedener Gase drin ist. "Drueckt der Luftdruck sofort wieder zusammen": auch das wuerde ich ausschliessen. Die Blase aussen wurde ordentlich tiefgezogen, die drueckt so schnell nix wieder zusammen. "Risse nach aussen" - noe, glaube ich auch nicht, das wird schon einigermassen gasdicht sein, das Material ist waehrend des Einschlages geflossen. --Nurmalschnell (Diskussion) 12:28, 8. Jan. 2016 (CET)
Der Punkt ist hier schlicht wer der das wir nur ein Foto vom Schnitt haben, udn erst noch eines auf dem man nicht mal die Kristalstruktur sieht. Für eine brauchbarer Erklärung wie der Hohlraum enstschatden ist, müssten wir aber das ganze Stück sehen können, oder eben einen Kristallschliff. So ist es bischen Stochern im Nebel, da es mehrere Varianten gibt. Ich habs schon gesehen, dass eine Beschussprobe seitlich aufgerissen ist. Das da auf der Rückseite keine runde Beule sondern eine mehr eine Welle drin war. Der Hohlraum bis zu einer Seite des Prüfstückes reichte. Das innere Trennen kann es dir auch bei anderen Biegebeaspruchungen geben, dass du im Inneren einen Längsriss kriegst (Laiennhaft ausgedrückt das sich das Blech imm innern in zwei dünnere Bleche auffteilt). Bei einem Beschuss hat man es eben auch mit Biegebeanspruchungen zu tun, und eben auch noch mit Wärme. So das eben die kalte Ausseinseite anders reagiert als der warme Kern. Dann reicht eben auch ein kleine Schädigung und die Teile trennen sich, und federn unter Umständen unterschiedlich zurück. Noch bisschen Gasbildung in der Schwachstelle und die ansich luftdichte Beule bleibt nach dem abkühlen stabil. Denn eine gute Panzerplatte ist aussen auf Härte und innen auf Zähigkeit getrimmt (beides zusammen geht fast nicht, denn es gilt hart=spröde, zäh=weich), damit gibt es in einer Panzerplatte auch unterschiedliche Gefüge, und in der Übergangszohne kann es bei genügend grosser Krafteinwirkung zu einer Abtrennung kommen. --Bobo11 (Diskussion) 12:00, 9. Jan. 2016 (CET)

Apfelschnecke schlau genug?

Ich habe in mein neuem (das Kleinere ersetzende) Aquarium eine große Wurzel stehen. Diese geht bis ca. 5cm unter die Wasseroberfläche. Nun habe ich auch Apfelschnecken, die ja atmen müssen. Zweimal habe ich jetzt gesehen, wie jeweils eine Schnecke minutenlang mit ausgefahrenem Atemrohr auf dem oberen Ende der Wurzel saß. Natürlich ohne Chance dort an Luft zu kommen. Ich hab sie dann beide dort weg geholt und auf Höhe der Wasseroberfläche an die Scheibe "geklebt". Hat jemand Erfahrung, wie schlau so eine Schnecke ist?! Soll heißen: muss ich mir Sorgen machen, dass sie da oben ertrinkt oder kommt sie rechtzeitig auf die Idee es wo anders zu versuchen?!?!


--92.210.152.101 20:46, 7. Jan. 2016 (CET)

Meine waren immer "schlau" genug; d.h., wenn's irgendwo nicht geklappt hat, haben sie innerhalb einer, zwei Stunden einen anderen Platz gefunden. Immer geradeaus, bis sie sich an der Scheibe gestoßen haben, dann hinauf. Leider bekomme ich keine neuen, nachdem sie an Altersschwäche gestorben sind. Faszinierende Viecher. 84.153.83.131 21:04, 7. Jan. 2016 (CET)
Es gibt eine einfache Lösung für die Frage des Schlauseins von Apfelschnecken im und unter Wasser (als Maß nehmen wir jetzt mal den Menschen):
  • Google => "Apfelschnecke ertrunken" <=
und dann
  • google => "Mann ertrunken" <=
Sie sind gut 80 Millionen Jahre alt und in dieser Zeit perfekte Überlebenskünstler geworden (Lungen + Kiemen ... und sie rauchen auch nicht! Man suche auch mal => Taiwan Apfelschnecke <= Hossah!).
Wenn die Parameter (pH, Temp. etc.) deines Aquariums stimmen, brauchst du dir keine Sorgen zu machen. Play It Again, SPAM (Diskussion) 08:27, 8. Jan. 2016 (CET)
Apfelschnecken sind so schlau, dass sie sowohl Kiemen als auch Lungen entwickelt haben, also schlau genug. --Pp.paul.4 (Diskussion) 09:51, 8. Jan. 2016 (CET)
ausgewachsene Apfelschnecken fressen schlafende Kleinfische wie Guppys , Platys- habe einer Freundin mal eine dicke Gelbe geschenkt, nach der ersten Nacht waren fast alle Fische verschwunden.....die war echt sauer, nicht die Schnecke, nur meine Freundin--Markoz (Diskussion) 19:07, 8. Jan. 2016 (CET)
Das wäre nicht die erste paarspezifische apfelartige Übergabe, die Probleme erzeugte. Bananenspinnen oder Birnen-Zitterrochen hingegen sind nicht mythologisch belastet... scnr Play It Again, SPAM (Diskussion) 10:30, 9. Jan. 2016 (CET)

Schadensersatzpflicht

Macht sich jemand schadensersatzpflichtig, wenn er als Gast und Experte in einer Servicesendung des öffentlich-rechtlichen Fernsehens wie z.B. "Alle Wetter" vom Hessischen Rundfunk statt fachliche Aussagen zu machen, den Rücktritt der Bundeskanzlerin wegen der Kölner Geschehnisse fordert und damit das Konzept der Livesendung über den Haufen wirft ?

--89.12.247.144 18:59, 8. Jan. 2016 (CET)

Nö hier gibt es Meinungsfreiheit--Markoz (Diskussion) 19:00, 8. Jan. 2016 (CET)
Für Schadenersatz müsste m. E. ein bezifferbarer finanzieller Schaden entstanden sein - welcher sollte das sein? --Eike (Diskussion) 19:38, 8. Jan. 2016 (CET)
Sinnvollerweise sollte ein Vertrag geschlossen sein, sodass eine Vertragsstrafe fällig wird. --BlackEyedLion (Diskussion) 23:30, 8. Jan. 2016 (CET)
Das hat weniger mit Schadensersatz zu tun, sondern ist in erster Linie ein Strafrechtsthema. Muttis Rücktritt gefordert? Im Fernsehen? Das ist mindestens Alternativlosigkeitsleugnung in Tateinheit mit Majestätsbeleidigung. Hat er möglicherweise sogar "provoziert" oder gar - "geschürt"? Ich dachte, so was gibt es allenfalls noch in der Anonymität des Internets, aber doch nicht im Fernsehen. Das erinnert uns Ältere an die düstere Vergangenheit, vor der Überwindung und Abschaffung von Opposition. --Gottbegnadeter (Diskussion) 23:47, 8. Jan. 2016 (CET)
Opposition? Haben wir eine Opposition? Im Bundestag???--80.129.151.196 13:29, 9. Jan. 2016 (CET)
Natürlich nicht, keine Angst. Aber wer in den finsteren 80er Jahren schon gelebt hat, der kann sich noch daran erinnern, dass es mal wie selbstverständlich Opposition gab. Im Bundestag und sogar in den Medien. Jeder Hinz und Kunz konnte einfach so Zweifel an der Kanzlerpolitik äußern in aller Öffentlichkeit. Ohne dass das staatsanwaltschaftliche Ermittlungen, Schadensersatzdiskussion, Medienaufschrei und gesellschaftliche Ächtung als Nazi oder gar als Rechtspopulist zur Folge hatte. In den 70ern war es noch krasser. Damals wurden Kanzler nicht nur kritisiert, sondern mussten sogar zurücktreten (Brandt) wenn sie sich haben ausspionieren lassen und somit erpressbar geworden sind. Zum Glück für Mutti und ihre US- Strippenzieher sind solche Zeiten vorbei . --Gottbegnadeter (Diskussion) 16:01, 9. Jan. 2016 (CET)

A. E. Thiemann

Es gibt einen "A. E. Thiemann" der in den 1920/30/40er Jahren ziemlich viel zum Thema Motoren und Kraftstoffe publiziert hat und der vermutlich auch enzyklopädisch relevant wäre. Ich kriege jedoch nicht mal die Vornamen raus. :-( Findet jemand genügend Daten, damit es zumindest für einen Rotlink in Thiemann reicht?--Mabschaaf 19:48, 8. Jan. 2016 (CET)

August Ernst. --Timurtrupp (Diskussion) 19:53, 8. Jan. 2016 (CET)
(BK) Das macht die DNB für dich: [22]. Grüße Dumbox (Diskussion) 19:54, 8. Jan. 2016 (CET)
Vielen Dank soweit. Findet auch noch jemand Lebensdaten und Beruf?--Mabschaaf 22:49, 8. Jan. 2016 (CET)
Wenns brennt (...Wortspiel...) würde ich so vorgehen:
  • Bei der WP-BIBA nach der Einleitung nachfragen zu dem Buch:
    • "Fahrzeug-Dieselmotoren" Das Schriftstück mit der Betitelung Fahrzeug-Dieselmotoren wurde im Jahr 1929 in Berlin über den Verleger R. C. Schmidt & Co. verbreitet. Der Autor ist August Ernst Thiemann und hat einen Titel publiziert.
Ich habe ein paar Publikationen aus dieser Zeit. Da haben die Autoren meist ein sehr persönliche Einleitung geschrieben (wie sie z.B. zu diesem Thama fanden). Evtl. hat man da eine Chance. "Ingenieur A. E. Thiemann" oder "Chemiker A. E. Thiemann" oder "Mechaniker A. E. Thiemann" liefern keine Hits. Gruss Play It Again, SPAM (Diskussion) 11:25, 9. Jan. 2016 (CET)
Verfrühte Initialzündung: Er war Ingenieur (2. und 4. Fundstück). Bleiben noch die Lebensdaten (da verweise ich wieder aufs Buch)=. Play It Again, SPAM (Diskussion) 11:31, 9. Jan. 2016 (CET)
Vielen Dank auch Dir für die Mühe. Inzwischen bin ich mir nicht mehr sicher, ob der Aufwand lohnt. Ein Eintrag in einer BKS sollte ja für mein dafürhalten zumindest irgendwann zu einem Blaulink führen können. Wenn die Quellenlage aber so dünn ist, wird es dafür nie reichen :-( Mein Thema ist es sowieso nicht, hier einen Personenartikel zu schreiben. Falls das also jemand adoptieren möchte: gerne! Sonst wird das hier (leider) einschlafen.--Mabschaaf 11:32, 9. Jan. 2016 (CET)

Dissertationspreis

Was ist ein Dissertationspreis? Ist das besser als ein summa cum laude oder ein Synonym? Wieviel Prozent aller Dissertationen (in Deutschland jährlich) bekommen in etwa durchschnittlich (ganz grob) so einen Preis? 0,001% 0,01% oder 0,1% oder 1,0% oder 10% oder gar noch mehr? vielen dank--Emilsinclair2.0 (Diskussion) 21:42, 8. Jan. 2016 (CET)

Ein Dissertationspreis ist keine Note, sondern unabhängig davon ein Preis für die wissenschaftliche Arbeit. Er wird nicht notwendigerweise von der Universität vergeben, sondern von einer Stiftung, einem Unternehmen oder irgendeiner andere Institution, die daran interessiert ist, wissenschaftlichen Nachwuchs zu fördern. --Snevern 22:53, 8. Jan. 2016 (CET)
+1 Um eine Idee zu bekommen, wieviele es gibt, hier links bei "Preise und Auszeichnungen" klicken; dann auf der nächsten Seite mit "Preis" suchen. Jeder kann sich bewerben, aber nicht jeder bekommt einen Preis. Play It Again, SPAM (Diskussion) 11:05, 9. Jan. 2016 (CET)
Eine Dissertation kostet um die 3.000, € . Für eine Diss. mit der Notenbezeichnungen "summa cum laude" (mit Auszeichnung /ausgezeichnet; oberes sehr gut (1+)) gibts für ein paar hundert Euro mehr. Im Fach Psychologie stieg der Anteil in den letzten Jahren auf 25 Prozent. LOL --91.10.6.194 01:25, 9. Jan. 2016 (CET)
Häh? Kostet? Seit wann kostet eine Promotion? Summa cum laude ist, je nach Bundesland, einfach nur die beste Note. 90.184.23.200 13:57, 9. Jan. 2016 (CET)
Er meint vermutlich für Leute, die ihre Promotionsarbeit von anderen schreiben lassen. --Eike (Diskussion) 14:36, 9. Jan. 2016 (CET)

Um einen Dissertationspreis bewirbt man sich typischerweise nachdem man die Promotion erfolgreich abgeschlossen hat. Je nachdem wer den Preis vergibt, kriegt man dann eine Urkunde oder Geldpreise (manchmal 10.000 Euro). Viele vergessen das, dabei stehen die Chancen oft recht gut und man kann die Bewerbungsunterlagen meist in leicht abgewandelter Form für mehrere Preise verwenden. Eine Google-Recherche im eigenen Fachgebiet wird meist viele Treffer ergeben. Und selbst wenn man nur in den Kreis der besten fünf Bewerber kommt, kann man das schon in seinem Lebenslauf gut verkaufen. Es lohnt sich also. 90.184.23.200 13:55, 9. Jan. 2016 (CET)

Nachtisch: Wer mit der Meinung von Leonard Mlodinow bezüglich Preisen und Ehrungen vertraut ist, wird ein besonderes Fest haben, wenn er erfährt, dass sich auch unter mit Dissertationspreisen ausgezeichneten Dissertationen Plagiate befinden. Das nennt man dann "doppelt gut". Hehehehe! Play It Again, SPAM (Diskussion) 15:06, 9. Jan. 2016 (CET)

Unionpedia

Eine weitere Pedia, die auf der Wikipedia aufbaut, scheint die Unionpedia zu sein. Eine Personenabfrage zur Person Henning Haßmann ergab dort folgendes Ergebnis.

Offensichtlich werden alle Wikipedia Links im Personenartikel in der Wikipedia dort aufgelistet. Ist diese Pedia bekannt, ist sie der Nachnutzung bekannt, falls dies sein muss? --AxelHH (Diskussion) 23:45, 8. Jan. 2016 (CET)

Unionpedia ist ein Konzept Karte oder semantische Netzwerk organisiert wie ein Lexikon oder Wörterbuch – gibt’s das auch auf Deutsch? Kann ich nicht so ganz ernst nehmen, das Projekt. --Kreuzschnabel 08:36, 9. Jan. 2016 (CET)
Kommt aus dem Englischen und schein botgeneriert zu sein - auch in den Ergebnissen.
Hat nichts mit der CDU zu tun, obwohl es Kauderwelsch ist. Play It Again, SPAM (Diskussion) 11:00, 9. Jan. 2016 (CET)
"Botgeneriert" etc. scheint neuerdings die Vokabel zum Schönreden sprachlicher Unmöglichkleiten zu sein. --84.135.139.5 11:16, 9. Jan. 2016 (CET)
Hihi, den hab ich kapiert! ;) Interessant, kein Impressum, kein Garnix, und alles Suchen führt - klar! - zu Unionpedia. Eine Domainabfrage liefert einen jungen Herrn in Buenos Aires, Fotograf und Software-Entwickler, dessen Leidenschaft es ist zu denken. Grüße Dumbox (Diskussion) 11:18, 9. Jan. 2016 (CET)
Erhältlich in Deutsch, Englisch, Französisch, Spanisch, Russisch, Ukrainisch, Schwedisch, Polieren, Niederländisch, Portugiesisch, Italienisch, Arabisch, Japanisch und Chinesisch. [Hervorhebung von mir] --132.230.195.196 11:32, 9. Jan. 2016 (CET)
Verflixte Homographe! Dumbox (Diskussion) 11:42, 9. Jan. 2016 (CET)

senkrecht begraben

In welchen Kulturen oder zu welchen Gelegenheiten oder Anlässen wurden Leichen senkrecht begraben? --93.133.156.73 11:48, 9. Jan. 2016 (CET)

Krieger ... und in Österreich in den 1970er Jahren ... also der Club der Senkrecht-Begrabenen
Auch der dann schweigende Till wurde stehend begraben.
Mehr mit Google => "buried upright"<= oder => "Vertical burials" <=
Modernes Projekt in Israel. {Unsinn} Play It Again, SPAM (Diskussion) 12:12, 9. Jan. 2016 (CET)
„multi-tier vertical burial system [...] to accommodate multiple vertical graves“ im Yarkon cemetery in Petach Tikwa heißt aber nicht stehend begraben, sondern bloß, dass sich die einzelnen Grabstätten in mehrstöckigen Gebäuden befinden. --Vsop (Diskussion) 15:43, 9. Jan. 2016 (CET)

Feinstrumpf nur kurz für Feinstrickstrumpf?

Getrickt
Fingestrickt?

Zum Abschnitt Strumpfhose#Feinstrumpfhosen gibt es folgende Bemerkung:

Zitat "Genau genommen ist die Unterteilung in Feinstrumpfhosen und Strickstrumpfhosen nicht ganz korrekt, da auch Feinstrumpfhosen gestrickt (und nicht etwa gewebt) sind"

Nun ist mir Aufgefalle das der dm Markt in seiner Produktübersicht Bezeichnung "Feinstrick" verwendet.[23] Ist das irgend eine neue Wortkreation oder ein seltener aber korrekter Überbegriff wie z.B. Schenkelstrümpfe für Halterlose- und Straps-Strümpfe? Zudem gibt es noch Strümpfe mit echten Nähten die wohl gewebt sind. Webstrumpf? --Tobias "ToMar" Maier (Diskussion) 15:50, 9. Jan. 2016 (CET)

Hopla. Ich merke gerade das der Begriff bei der DM Quelle verändert wurde und der Snapshot bei Wayback zu jung ist. :(--Tobias "ToMar" Maier (Diskussion) 16:01, 9. Jan. 2016 (CET)

Akku mit zu wenig Ampere laden gefährlich?

Hallo, hab schon Google & Co gefragt, aber nicht wirklich ne Antwort gefunden. Ich habe zu Weihnachten eine neue E-Dampfe geschenkt bekommen und bis dato über USB 3.0 geladen. Nun hab ich aber gelesen, dass das meinen Computer zerstören kann. Von meiner alten habe ich jedoch noch einen Wandstecker, der 5.0V bei 0.5A raushaut. Da USB 3.0 allerdings 0.9A ausgibt, frage ich mich, ob ich trotzdem mit dem Wandstecker, der per USB lädt, meine E-Dampfe laden kann, oder ist das zu gefährlich für den Akku (ist der hier [24]) --Odeesi talk to me rate me 20:52, 6. Jan. 2016 (CET)

Das Problem krigt hier nicht der Akku, sondern das Ladegerät. Wenn deine E-Dampfe mehr Amper zieht als das Ladegerät liefern kann, überhitzt das Ladegerät (oder eben diesen Computerstromkrie des USB 3 versorgt). Du soltest also immer darauf schauen, dass das Ladegerät mehr Amper liefern kann als deine E-Dmapfe beziehen will. --Bobo11 (Diskussion) 22:06, 6. Jan. 2016 (CET)
Wie soll ein Verbraucher mehr Strom ziehen als das Ladegerät liefern kann? Kannst mir das mal technisch erklären? Er kann möglicherweise mehr Strom ziehen, als die Quelle liefern sollte (wenn sie so blöd ist, das zu tun), aber nicht mehr, als sie liefern kann. Wo soll der denn herkommen? Und dafür, daß eine Stromquelle nicht mehr Strom abgibt, als für sie gut ist, sollte einklich deren Hersteller konstruktiv sorgen (Strombegrenzung). --Kreuzschnabel 07:16, 7. Jan. 2016 (CET)
Mit dem kann ist logischerweise die Strommenge gemeint, die das Ladegerät auf immer und ewig lieferen kann 8Udn nicht der kurzfristig lieferbare). Also auf die Stromstärke, auf die das Ladegerät auch ausgelegt ist. Wenn die E-Dampfe 1.2 A beziehen will, dann sollte das Ladegerät mindestens 1.2A ausgeben können, ohne dabei überlastet zu werden. Sprich auf dem Typenschild des Ladegerät sollte in diesem Fall mindesten 1.2 A stehen. --Bobo11 (Diskussion) 09:40, 7. Jan. 2016 (CET)
Der Akku darf da nicht ungeregelt dran. Batterycontroller bzw. Ladeschaltungen verhindern Überstrom, Überladen usw… Der USB sollte mit einer PTC-Sicherung – einer selbstrückstellenden Sicherung oder einem FET (Feldeffekttransistor, der selbst bei Überstom Schließt bzw. abregelt oder abgeregelt wird) ausgestattet sein. Aus Kostengründen ist er das oft nicht und es brennt ein filterndes Bauteil durch. Nur die alten Nickel-Cadmium-Akkus durften wegen des Memory-Effektes nur in vollständigen Zyklen und nicht im Schneckentempo geladen werden. --Hans Haase (有问题吗) 22:37, 6. Jan. 2016 (CET)
USB-Netzteile mit 3 Ampere gibt es für wenig Geld beim Elektronikhändler. Da lohnt es nicht, zuerst das alte Ladegerät zu ruinieren. Hier kommt es nämlich auf die Ladeschaltung der E-Dampfe an. Ist diese Ladeschaltung hinreichend intelligent, mit dem Ladegerät auszumachen, wieviel Strom sie ziehen darf, lädt sie auch an einem 500-mA-Ladegerät, nur aber sechsmalzweimal so lang. Ist die Ladeschaltung der E-Dampfe doof, ziehst sie ohne nachzufragen 0,9, 2,1 oder 3 1 Ampere und überhitzt oder zerstört so das Ladegerät. Für 3 Ampere ist eigentlich USB 3.1 Typ C zuständig, der kann und darf das. PCs haben aber üblicherweise USB 3.0 Typ A, der nur bis 1,5 Ampere spezifiziert ist. Je nach genauer Spezifikation der USB-3.0-Anschlüsse sind die Steckverbinder am PC elektrisch bis 5 A belastbar, der PC kann und darf aber vorher abregeln (oder kaputtgehen). --Rôtkæppchen₆₈ 22:38, 6. Jan. 2016 (CET)
Naja, die Dampfe scheint schon was intelligenter zu sein. Is ne Wismec Presa TC75W, auch wenn ich den Akku niemals so auslasten würde... da wäre der Dampf mir dann zu heiss. Aber technisch beruht sie wohl auf der joytech eVic VTC Mini. --Odeesi talk to me rate me 23:18, 6. Jan. 2016 (CET)
Eine Last im elektrischen Stromkreis kann nicht mehr Strom „ziehen“, als die Quelle abgibt. Die E-Dampfe kann also nicht mit Gewalt 0,9 A aus einem USB-Netzteil ziehen, das sich weigert, selbst im extremsten Fall, also dem Kurzschluß, mehr als 0,5 A zu liefern. Ich weiß nicht, wie das Schaltungskonzept heutiger USB-Anschlüsse in dieser Hinsicht aussieht, kann mir aber nicht so recht vorstellen, daß da auf Versorgungsseite groß was kaputtgehen soll. Am ehesten kann ich mir denken, daß der Last die 0,5 A, die sie bekommt, nicht reichen und der Akku deshalb gar nicht oder nicht richtig geladen wird. Inwieweit das der Fall ist, sollte dir einklich die Anleitung sagen, aber bei der Qualität heutiger Anleitungen hab ich da meine 2fel. --Kreuzschnabel 07:12, 7. Jan. 2016 (CET)
Eine Quelle kann durchaus mehr Strom abgeben, als ihr guttut. Dann überhitzt die Stromquelle, geht kaputt oder die Sicherung brennt durch. Die oben verlinkte Dampfe hat einen Anschluss Mikro-USB 2.0 Typ AB, darf also maximal 1 Ampere ziehen. USB 3.0 Typ A darf mit Datenverkehr bis 0,9 und ohne bis mindestens 1,5 Ampere belastet werden. Meines Erachtens sollte die Dampfe also problemlos an einem USB 3.0 Typ A geladen werden können. --Rôtkæppchen₆₈ 09:33, 7. Jan. 2016 (CET)

Kann mir mal jemand erklären, was eine E-Dampfe ist? Die Website des Herstellers hilft mir da auch nicht wirklich weiter. --Jossi (Diskussion) 11:23, 8. Jan. 2016 (CET)

E-Zigarette, vermute ich. --Eike (Diskussion) 11:45, 8. Jan. 2016 (CET)
So sehen die Dinger aber nicht aus. --Jossi (Diskussion) 21:21, 9. Jan. 2016 (CET)

Einen Akku mit zuwenig Ampere zu laden, kann sogar lebensgefährlich sein. Nehmen wir an, du lädst den 12-Volt-Akku eines Autos mit 1 Mikroampere. Du wirst dabei sterben. --62.202.181.162 11:31, 8. Jan. 2016 (CET)

Nein, die Selbstentladung der Starterbatterie ist höher. Komm wieder runter, gefährlich ist das nicht. --Hans Haase (有问题吗) 23:37, 9. Jan. 2016 (CET)
Das war ein Witz, oder sollte jedenfalls einer sein. Sogar wenn du einen Metallstab in die Luft hältst, kriegst du einen Strom ab, der höher ist, wei das Ding als Antenne wirkt.--Giftzwerg 88 (Diskussion) 23:44, 9. Jan. 2016 (CET)

Dämme im Mittelalter

Hallo,

ich möchte für einen Fluss gaaanz grob abschätzen (nicht baurelevant!), wie ein Damm in diesem Fluss beschaffen sein müsste. Dazu habe ich die Abflussmenge des Flusses mit seiner Durchschnittsgeschwindigkeit multipliziert und die Kraft erhalten, die auf jeden hypothetischen Querschnitt des Flusses einwirkt.

Es geht um eine Wette mit einem Kameraden aus dem Geschichtsunterricht, ob man im Mittelalter nicht mit Dämmen die an Flüssen liegende Bevolkerung mit Wasser und einer Kanalisation hätte versorgen können. Anders ausgedrückt, der Damm soll auf mittelalterliche Weise errichtet worden sein. Meine Idee war jetzt einfach, da ich momentan die sicher komplizierten physikalischen Verhältnisse (im Flussbett steckende Baumstämme u. ä.) nicht miteinbeziehen kann, einfach zu kalkulieren, wie viel Baumaterial man aufschütten müsste, damit die ausgerechnete Kraft nicht ausreicht, um die Masse des Baumaterials zu bewegen und dazu noch eine Hochwasser- / Stauungseffekt- / Sicherheitsmarge von 200 % aufzuschlagen.

Darum meine Frage: 1.) Welches Haupt-Baumaterial wurde für Dämme im Mittelalter verwendet? 2.) Wie wurde damals ein Damm konstruiert (rein interessehalber)?

Ich weiß, dass diese "Berechnung" modellhaft-abstrakt ist und nichts mit der damaligen Realität zu tun hat und allein schon ungenaue Datenwerte das Ergebnis verhageln. Aber ich bin einfach neugierig. Vielen Dank im Voraus, --217.237.164.210 20:02, 8. Jan. 2016 (CET)

Nicht gerade Mittelalter, aber siehe den Artikel Cornalvo-Talsperre. Laut en:WP gab es in Persien im Mittelalter auch reihenweise Stauseen mit Wasserrädern zur Kultivierung der Agrargebiete; suche nach = "dam". Siehe aber auch Liste römischer Talsperren und See von Homs. Da die Erfindung des römischen Betons dann in Vergessenheit geriet mag dann eine lange Pause bis in die beginnende Neuzeit eingetreten sein. --Cookatoo.ergo.ZooM (Diskussion) 21:19, 8. Jan. 2016 (CET)
Vielleicht hilft das weiter. --Niki.L (Diskussion) 22:10, 8. Jan. 2016 (CET)
  • Bitte definiere Mittelalter genauer, am besten mit einer 50-100 Jahre Periode. Denn im dem was normalerweise als Mittelalter bezeichnet wird, gab es eigtlich noch kein Bedarf an "moderen" Dämme. Was Otto Normalbürger gern noch als Mittelalter versteht ist aber gern mal Spätmittelalter oder schon Frühe Neuzeit (ab 15. jahrhundert). Dann richtige Flussverbauungen usw. gab es erst vereinzelt im Spätmittelarter, richtig verbreitet eigentlich erst in der frühe Neuzeit. Das dieses Zeitproblem auch ein Problem bei der Fragestellung sein kann, ist hoffentlich verständlich.--Bobo11 (Diskussion) 22:59, 8. Jan. 2016 (CET)
  • Zum Thema selber (ohne Zeitbezug); In der Regle Holz udn was sonst vor Ort vorhanden war. Denn der Transport war bevor es ausgebaute Strassen gab ein generelles Problem. Die "guten" Strassen, also die Chaussee somit die ersten echten Kuststrassen sind aber ganz klar schon Neuzeit (Selbst 1836 rechnete der Kanton Bern für Strassentransporte noch wie folgt. Die Annahme war, dass der effektive Gestehungspreis beim Ankauf von einem m³ Stein zu 2 alten Franken, sich bei Transport über 6 (sechs!) Kilometer verdoppelt. Der Transport über 6 Kilomter ebenfalls 2 alte Franken kostet würde). Davor war es schlichweg unrealistisch mit nicht vor Ort greifbaren Baumaterialen zu bauen. Das heisst man musste sich in der Umgebung bedinen können, oder die Baustoffe mussten auf dem Fluss von oben heran geführt werden können (und das ging am einfachsten beim Holz). Am üblichsten waren Faschinen Verbauungen. Mit dennen griff man eben auch in die Strömmung und denn Geschiebtransport ein. Die Dämme wurden dann mit abgelagerten Kies von Hand geschüttet, oder man brach das Rohmaterial eben in der Nähe. Das waren auch keien Bauarbeiter die diesen Ausbau machten, sondern die Bevölkerung vor Ort erledigte die Sache als Gemeinwerk. Gern mal als Fronharbeit, oft aber auch als bezahlte Taglöhner-Arbeit. Dann aber eben in der Form der Beschäftigung der Armen in der Zeit wo keine Feldarbeit anfiel (eine frühe Form der Fürsorge). --Bobo11 (Diskussion) 23:14, 8. Jan. 2016 (CET)
Zum Thema Holztransport muss man auch die kleinen Staudämme nennen, die das Wasser von Bächen soweit aufgestaut haben, dass auch dort mit einer Welle die Flösse transportiert werden konnten. Die Staudämme bestanden z.T. am Rand aus Kies/Erde/Stein, im Wasser dann aus Holzpalisaden. --Hachinger62 (Diskussion) 19:20, 9. Jan. 2016 (CET)

Durch Schädelbasisbruch gerruchsinn verloren was kann ich tun

--91.44.152.185 12:27, 9. Jan. 2016 (CET)

Notarzt rufen? --Simon-Martin (Diskussion) 12:30, 9. Jan. 2016 (CET)
Anosmie lesen und den behandelnden Arzt fragen. Play It Again, SPAM (Diskussion) 12:38, 9. Jan. 2016 (CET)


Kommt entweder von selbst wieder oder auch nicht. Behandlung gibt es keine, nur eine Menge Scharlatane. -- Janka (Diskussion) 16:55, 9. Jan. 2016 (CET)

Ja, da hat Janka recht, entweder erholen sich die Nerven von selbst, oder sie bleiben für immer geschädigt. Aber du kannst es auch positiv sehne, denn nicht jeder überlebt einen Schädelbasisbruch. Sei glücklich das du überlebt hat, denn eine positive Lebenseistellung hilft auch dann, wenn der Geruchsinn beeinträchtigt bleibt. --Bobo11 (Diskussion) 17:21, 9. Jan. 2016 (CET)
Mach doch aus Deiner Not eine berufliche Existenz. Ein Freund von mir verlor bei einem epileptischen Anfall Geruch und GeschmacKsinn...ein Arzt machte ihn auf eine sehr gute Verdienstmöglichkeit aufmerksam. Der ist Tatortreiniger geworden. Der arbeitet 5- maximal 10 Tage im Monat und verdient wirklich gut. Ist nichts für zartbesaitete Seelen. Aber die Leichen sind immer schon weg, in der Regel handelt es sich auch nicht um Morde sondern verweste Vereinsamte. Das ist sehr geruchsintensive Entrümpelung die sehr, sehr gut bezahlt wird. Einmal hatte der einen dreifach Messermord, da war der echt fertig nach, habe die ganze Nacht mit dem gesoffen um den aufzumuntern, aber das war einmal in 10 Jahren.--Markoz (Diskussion) 00:15, 10. Jan. 2016 (CET)

Wer kann diese Signatur / Unterschrift auf dem Gemälde entziffern?

Gemälde Unterschrift

Habe das Bild auf dem Flohmarkt erstanden. Es ist ein Stillleben mit Obst (Wein und Äpfel) und einem Krug, alles auf einer weißen Tischdecke. Wer kann die Signatur entziffern?

--Piadora (Diskussion) 17:04, 9. Jan. 2016 (CET)

Karl-Heinz, Nachbar, Hobby-Maler --87.148.75.22 17:29, 9. Jan. 2016 (CET)
So wird das nichts, weil man nach Signaturen auf Stillleben nicht nur nach den Signaturen suchen kann. Bitte das ganze Bild zeigen. Wahrscheinlich wird es schwiereig, weil Bilder von wirklich berühmten Malern nur selten auf Flohmärkten auftauchen. Play It Again, SPAM (Diskussion) 23:09, 9. Jan. 2016 (CET)
Die werden aber hier und da anderen Artikeln beigelegt oder auch direkt verkauft. --2003:76:E4C:F1AD:8530:90E9:242D:23FF 23:51, 9. Jan. 2016 (CET)

Abkürzung in einem Kirchenbuch?

Moin, mal wieder eine Frage an die versammelten Experten: In einem Kirchenbuch findet sich ein Eintrag »geboren am 24. November igoi oder Jahrgang igoi Nr. 38«, das Jahr wurde mir leider nicht übermittelt, wird wahrscheinlich 17. bis 19. Jh. sein. Was bedeutet dieses »igoi«. Wahrscheinlich eine Abkürzung die ich meinen Nachschlagewerken nicht finden. Was Google ausspuckt, bezieht sich eher auf Börsengeschehen in unserer Zeit, soweit ich das ausreichend überblicken kann. Freue mich über Antwort. --Gwexter (Diskussion) 11:37, 7. Jan. 2016 (CET)

Sicher, dass da nicht 1901 steht? Grüße Dumbox (Diskussion) 11:40, 7. Jan. 2016 (CET)
Was mich betrifft: Brett vorm Kopf? Ich frage noch mal ... --Gwexter (Diskussion) 11:58, 7. Jan. 2016 (CET) Sorry, meine Bemerkung war wohl eher doppeldeutig ...
Schau dir mal Mediävalziffer an. Ich wette, Dumbox hat recht. Rainer Z ... 12:08, 7. Jan. 2016 (CET)
Möglicherweise hat den Leser (nicht ich) auch ein jeweiliges Pünktchen über den handschriftlichen Ziffern irritiert, war ja lange Zeit üblich. Wie gepostet, ich habe nochmal zurückgefragt. --Gwexter (Diskussion) 12:48, 7. Jan. 2016 (CET)
Wenn es ein Kirchenbuch ist, müsste sich schon aus dem Gesamtzusammenhang (übergreifender Titel, alle anderen Einträge etc.) ergeben, ob prinzipiell die Lesart "1901" in Frage kommt oder ob das abwegig ist. Wenn das "igoi" nur an einigen Stellen auftaucht, denke ich, dass es eine andere Bedeutung hat. Das Ganze ist doch eine Frage zur "Historik" als Wissenschaft. Gibt es hier in WP keine Historiker, die das lösen können? Ich dachte doch, die Historiker lernen den Umgang mit Quellen. Dazu gehört doch zu allererst das Lesen selbiger. Vielleicht kann man auf dem Geschichtsportal helfen? --84.135.153.125 13:32, 7. Jan. 2016 (CET)
Schimpf doch nicht gleich. So ein Projekt zur Lokalgeschichte hat meist viele Freiwillige, die Quellen lesen und transkribieren, und sicher nicht alle sind ausgebildete Historiker. Da kann so eine Fehldeutung von Ziffern für Buchstaben leicht mal vorkommen, und in den beiden genannten Kontexten ist die Konjektur nun halt ausgesprochen naheliegend. Gwexter sprach ja selbstironisch von einem Brett, und oft genug passiert es einem ja, dass man den Wald vor lauter Bäumen nicht sieht. Grüße Dumbox (Diskussion) 13:44, 7. Jan. 2016 (CET)
Danke Dumbox. @IP: Zur Erläuterung: Ich habe die Anfrage wie o. a. bekommen, kenne das Original also nicht. Um solche Dinge zu klären, muss man übrigens kein Historiker sein. Ich selbst bin auch keiner, schlage mich aber seit einem Vierteljahrhundert mit alten Schriften herum, wobei ich zurückschauend bis ins 17. Jahrhundert Texte recht gut lesen kann. Und wenn man mal was nicht weiß (oder vom rechten Weg abirrt), darf man ja wohl noch fragen dürfen, auch wenn da keine hochkarätige wissenschaftliche Fakultät drauf wartet. Zur Qualität der Leute, die in diesen Bereich der WP reinschauen, mag ich mich nur lobend äußern. LG --Gwexter (Diskussion) 14:06, 7. Jan. 2016 (CET)
Übrigens liefert mir Google-Buchsuche allein für November igoi angeblich 688 (geschätzt 82, angezeigt 93) Ergebnisse, die auch 1801 beinhalten (nämlich gleich das erste mir angezeigte Ergebnis). --Pp.paul.4 (Diskussion) 14:30, 7. Jan. 2016 (CET)
Wie in diesem Falle: https://books.google.de/books?id=lH2WpQwZYRkC&pg=PA492&&dq=Dezember+igoi eine falsche Übertragung per Texterkennung. Suchbegriff hier »Dezember igoi«. Da muss man wohl erst drauf kommen. --Gwexter (Diskussion) 14:42, 7. Jan. 2016 (CET)

"Texterkennung"???? --84.135.141.169 11:03, 8. Jan. 2016 (CET)

Um deinem Fragesteller helfen zu können, wäre ein Faksimile (Scan, Foto) des Kirchenbucheintrags hilfreich. Die Möglichkeit eines Lesefehlers ist sonst einfach zu hoch. --Jossi (Diskussion) 11:33, 8. Jan. 2016 (CET)
@IP: In beiden Beispielen (in meinem noch etwas deutlicher) ist klar erkennbar, das beim Erfassen von »I80I«, was im Original im Kontext deutlich als »1801« anzusehen ist, ein Fehler unterlief. So etwas ist typisch für Programme, die Grafik-Textdateien (wie beispielsweise bei diesen Google-Scan-Interpretierungen) in weiter verarbeitbare Zeichen umstricken, also wirklich simple Lesefehler. Weil ich selbst viel mit Texterkennung arbeite, hätte ich das wissen sollen, aber ich kenne das Original zu meinem Anliegen noch nicht. Ich nehme jatzt an, dass es sich um ein pdf-Dokument handelt, das auf diese Art und Weise entstanden ist. Ein Lesefehler ist allerdings auch bei einer Handfschrift nicht auszuschließen. Ich hatte das schon erwähnt, dass die Ziffer 1 in älternen Schriften (wegen des Pünktchens darüber, dass die Ziffer 1 kennzeichnen soll) im Eifer des Gefechts schon mal als »i« oder »j« interpretiert wird. --Gwexter (Diskussion) 22:21, 8. Jan. 2016 (CET)

Vielleicht ein missgelesenes ejsd. [ejusdem] oder ibid. [ibidem]? --Miebner (Diskussion) 22:33, 8. Jan. 2016 (CET)

Moin, ich habe Rückmeldung bekommen: Gemeint ist sowas wie »igio« oder »ig io«, wie oben benannt also falsch übermittelt. Es geht um einen Geburtseintrag mit Taufe, wobei diese Zeichenkombination sich wohl auf den Taufeintrag bezieht. Die genaue Jahresangabe habe ich jetzt auch mit 1901, Zufall. An das Original komme ich leider nicht ran und muss mich auf meinem Informanden verlassen. Lesefehler per TE trifft also nicht zu, aber in meinen schlauen Büchern finde ich auch nichts. Ich denke aber, dass diese mögliche Abkürzung nichts anderes aussagen könnte als »getauft im gleichen Jahr« oder so ähnlich. LG --Gwexter (Diskussion) 19:33, 9. Jan. 2016 (CET)
Archivierung dieses Abschnittes wurde gewünscht von: Gwexter (Diskussion) 11:58, 11. Jan. 2016 (CET)

Nouvelle-Manche

In der fr:wp wird für die Völkerschlacht bei Leipzig ein teilnehmendes „1er régiment de cavalerie Landwehr Nouvelle-Manche“ genannt. Nouvelle-Manche habe ich in solch einem Zusammenhang noch nie gehört. Es handelt sich um ein preußisches Regiment. Kann da jemand etwas mit anfangen? --Centenier (Diskussion) 18:22, 10. Jan. 2016 (CET)

Ich konnte nix finden mit Suchfunktion im Artikel, also selber suchen UND: Dragoner-Marschregiment

(Régiment des Dragons de Nouvelle Marche) fiel mir als erstes ins Auge. Das dann folgende 1er régiment de cavalerie Landwehr Nouvelle-Manche ist dann ein Tipp oder Lesefehler. Werde ich gleich korrigieren. --G-Michel-Hürth (Diskussion) 18:49, 10. Jan. 2016 (CET)

Google Books => prussie "Nouvelle-Manche" <=
Dann ebenso => Neumark "Nouvelle-Manche" <= Auch Neupreußische Heeresorganisation (Landwehr-Kavallerie Neumark...)
Hilft das ? Play It Again, SPAM (Diskussion) 18:51, 10. Jan. 2016 (CET)
Ich tippe auf Schreibfehler bei den Franzosen: Nouvelle Marche = Neumark (Brandenburg)--Hinnerk11 (Diskussion) 18:55, 10. Jan. 2016 (CET)
Bei den meisten zitierten Stellen steht auch bei Eingabe von manche im Text marche. Laut Lexikon gibt manche in diesen Zusammenhängen auch keinen Sinn. --G-Michel-Hürth (Diskussion) 23:04, 10. Jan. 2016 (CET)
Natürlich - Neumark bzw. „1. Neumärkisches Landwehr-Kavallerieregiment“ wir dumm von mir. Man dankt! -- Centenier (Diskussion) 08:09, 11. Jan. 2016 (CET)
Archivierung dieses Abschnittes wurde gewünscht von: Centenier (Diskussion) 08:09, 11. Jan. 2016 (CET)

Akutalisierung Firefox

Hallo, ich arbeite unter Vista mit Firefox, derzeit Version 43.0.1 . Angeblich sind Updates verfügbar (entspricht auch dem Stand im Artikel Firefox). Nur: ich komme an diese Updates nicht ran, auch nicht über die Seite [25] - da heißt es "Herzlichen Glückwunsch. Sie verwenden die neueste Firefox-Version." Unter [26] German Windows kann ich auch nur die 43.0.1 laden. Andererseits bekomme ich regelmäßig die Meldung, dass eine neue Version zur Verfügung stünde, die mangels entsprechender Rechte nicht aktualisiert werden könne. O. k. hier im Netz bin ich ohne Adminrechte unterwegs, aber die Situation ist mit einem Adminkonto auch keine andere. Ratlos! Was habe ich übersehen? --91.89.10.94 23:41, 10. Jan. 2016 (CET)

Mit diesem Link hat es funktioniert. Herzlichen Dank! --91.89.10.94 00:12, 11. Jan. 2016 (CET)
Bittesehr. Achte aber darauf, dass die kommende Version nun automatisch Updated oder spätestens, wenn Du im Menü→Hilfe→Über Firefox auswählst, den Download beginnt, sonst rate ich zur De- und Neuinstallation nach vorheriger Sicherung des Benutzerprofils. --Hans Haase (有问题吗) 00:17, 11. Jan. 2016 (CET)
Archivierung dieses Abschnittes wurde gewünscht von: --Hans Haase (有问题吗) 12:20, 11. Jan. 2016 (CET)

Betriebssysteme in Java?

Hallo. Die folgende Frage ist vielleicht dämlich, aber welchen Vorteil bieten den Nutzern eigentlich Java-Betriebssysteme? Können sie auf ein laufendes OS gestartet werden oder wie läuft das mit der Kommunikation mit der Hardware? (nicht signierter Beitrag von 94.222.214.17 (Diskussion) 09:21, 8. Jan. 2016 (CET)) --(nicht signierter Beitrag von 94.222.214.17 (Diskussion) 2016-01-08T08:21:26‎ (UTC))

es gibt auch Java-Prozessoren... :) --Heimschützenzentrum (?) 09:25, 8. Jan. 2016 (CET)
Keinen, sie sind zu langsam. Auch wenn immer wieder erwähnt wird, dass man auch mit Java performant programmieren könnte, passiert das in der Realität nie. Genau so wie die Plattformunabhängigkeit von Java die zwar theoretisch möglich ist in der Praxis aber praktisch nicht vorkommt. --2003:66:8954:937A:2CEF:7AE2:7372:22A8 11:16, 8. Jan. 2016 (CET)
Das letztere ist falsch. Die weltweit millionenfach eingesetzten Programmpakete Apache OpenOffice und LibreOffice sind dank Java plattformunabhängig. --FGodard||± 11:54, 8. Jan. 2016 (CET)
Das wäre neu. Tatsache ist, dass sich zumindest LibreOffice der Altlast Java, die aus StarOffice übernommen wurde, entledigen will und schon den größten Teil des java-abhängigen Codes ohne Java neu geschrieben hat. Es verbleibt nur noch ein kleiner Teil java-abhängiger Code, der aber auch irgendwann mal weg soll. Siehe LibreOffice#4.x-Versionen--Rôtkæppchen₆₈ 13:12, 8. Jan. 2016 (CET)
Da muss man aber auch sagen, dass es heute nur bedingt um Performance geht. Was bringt mir der schnellste Code wenn ihn keiner schreiben kann oder es zu lange dauert ihn zu schreiben (übertrieben ja, schon klar). Und wenn selbst Optimierung nichts bringt und Teile zeitkritisch sind, dann werden diese eben in C oder so ausgelagert. Java-Code ist nun nicht allzu schwer, wird an vielen Hochschulen gelehrt, plattformunabhängig. Trotzdem verstehe auch ich nicht so recht, wie Java angeblich die populärste Programmiersprache sein kann. --87.140.192.3 21:35, 8. Jan. 2016 (CET)
Wenn Du den Artikel zu Ende gelesen hättest, dann hättest Du auch erfahren, dass die Methodologie dieses Index fragwürdig ist. Von einer in der verwendeten Form nicht wirklich gestellten Suchmaschinenanfrage auf die Popularität dieser Sprache zu schließen, halte ich für nicht zielführend. --Rôtkæppchen₆₈ 05:00, 10. Jan. 2016 (CET)

Sonnenaufgangszeiten in Abhängigkeit von der geografischen Höhe/Höhendifferenz

Ich bin gestern von Tromsø nach Oslo geflogen. In Tromsø ist derzeit Polarnacht. Die dauert noch bis 15. Januar (siehe Artikel Tromsø). Als das Flugzeug um 10:45 Uhr etwa 50 km südlich von Tromsø war und eine Höhe von etwa 4800 m erreicht hatte (alles per GPS ermittelt), ging zu meiner Überraschung im Süden die Sonne auf. Ich hätte erwartet, dass man dazu deutlich südlicher sein muss. Nun zur Frage: wie kann man die Sonnenaufgangszeiten bei gegebener geografischer Breite und Länge über die Bezugshöhe korrigieren? Im Web habe ich dazu nichts gefunden. Mit zunehmendem Breitengrad sollte der Einfluss der Bezugshöhe immer größer werden. Oder als konkrete Rechnung: Wie hoch muss in Tromsø ein Ballon fliegen, damit man im Dezember zur Sonnenwende die Sonne zur Mittagszeit gerade noch sieht? Gruß --Kuebi [ · Δ] 18:54, 8. Jan. 2016 (CET)

Wenn man annimmt, dass das Sonnenlicht den Polarkreis an Mittwinter tangential streift, ist das eine einfache trigonometrische Aufgabe. Bei einem Winkelunterschied Tromsö-Polarkreis von 3,08 Grad und einem Erdradius von 6366,2km komme ich auf 9,2km Höhe. Natürlich auch von den örtlichen Gegebenheiten abhängig. Wer hat was anderes? --Optimum (Diskussion) 19:19, 8. Jan. 2016 (CET)
Cool! Die 3,08° hätte ich ja selbst aus den Breitengraden (Tromsø und Polarkreis) errechnen können. Auf die Idee kam ich nicht. Aber interessant, dass man bei den flachen Winkeln gar nicht so hoch hinaus muss, um einen deutlichen Effekt zu erzielen. Gruß und Dank --Kuebi [ · Δ] 19:26, 8. Jan. 2016 (CET)
Wenn du nicht die Gradzahl sondern die Entfernung zum Polarkreis hast, geht es mit den Formeln in Sichtweite#Geometrische_Sichtweite. --Engie 19:40, 8. Jan. 2016 (CET)

Das erinnert ein bisschen an die Frage von neulich: Wie weit ist es bis zum Horizont. 90.184.23.200 15:07, 9. Jan. 2016 (CET)

Faktisch ist der Polarkreis dann alles andere als ein Kreis, sondern speziell im Gebirge eine Vielzahl von Linien, die im Norden auf den Gipfeln ihre maximale Auslenkung haben. Ein Kreis wäre es nur, wenn man es auf Seehöhe bezieht.--Kuebi [ · Δ] 08:19, 10. Jan. 2016 (CET)
Nicht mal dann. Ein Kreis ist der Polarkreis nur, wenn man von einem Rotationsellipsoid oder einer Sphäre ausgeht. Das gilt aber für jeden Breitenkreis. --Digamma (Diskussion) 10:49, 10. Jan. 2016 (CET)
In der Praxis ist natürlich wieder alles viel komplizierter als in der Theorie, siehe z.B. Erdfigur. Der Polarkreis ist nur eine gedachte Linie. Für die Sichtbarkeit der Sonne muss man dann auch noch das Geoid und die Astronomische Refraktion berücksichtigen.--Optimum (Diskussion) 12:09, 10. Jan. 2016 (CET)

Suche ein Mailprgramm das zu mir passt.

...und es sollte folgendes können: Ich hätte gerne eine Tabelle, bei der links runter alle meine 150 Adressen stehen und oben dann Spalten sind für meine verschiedenen Verteiler. Dann möchte ich ein X setzen können bei denen, die im Verteiler sind und kein X bei denen die keine Lust auf die eine oder andere Mail haben. Habt Ihr Tipps, egal ob kostenpflichtig oder gratis. Rolz-reus (Diskussion) 17:51, 9. Jan. 2016 (CET)

Mal eine Rückfrage: Welches Mailprogramm verwendest Du bisher? --Rôtkæppchen₆₈ 18:51, 9. Jan. 2016 (CET)

Outlook rolz-reus (nicht signierter Beitrag von 84.166.159.83 (Diskussion) 08:40, 10. Jan. 2016 (CET))

Lateinamerika Katalog des Bibliotheksnetzwerks?

Wie finde ich den gemeinsamen Katalog des lateinamerikanischen Bibliotheksnetzwerks? Danke --C.Koltzenburg (Diskussion) 22:04, 9. Jan. 2016 (CET)

Meinst Du den Bibliothekskatalog des Instituto Cervantes? --Rôtkæppchen₆₈ 00:05, 10. Jan. 2016 (CET)
Was genau meinst du mit "lateinamerikanischen Bibliotheksnetzwerks" Wenn es Iberialiber, der ersten GoogleTreffer ist, dann kannst du ja den KVK verwenden, aber bzgl. Deutschlnad dürfte die Bibliotek des IAI dem doch überlegen sein.--Antemister (Diskussion) 11:30, 10. Jan. 2016 (CET)

Bürgermeister von Dülken

Hi! Kriegt jemand raus, wer im Jahre 1936 in Dülken Bürgermeister war und vielleicht auch, von wann bis wann? Danke vielmals, – Doc TaxonDiskussionWiki-MUCWikiliebe?! 14:13, 10. Jan. 2016 (CET)

Mit zitternden Händen setze ich 5 Millionen Reichsmark auf Ludwig Simon als Bürgermeister von Viersen-Dülken. Play It Again, SPAM (Diskussion) 14:33, 10. Jan. 2016 (CET)
Könnte aber auch Dr. Gustav Mertens gewesen sein... Play It Again, SPAM (Diskussion) 14:59, 10. Jan. 2016 (CET)
... und was machen wir hieraus? Play It Again, SPAM (Diskussion) 15:01, 10. Jan. 2016 (CET)
Nee, nee, nee! Hier ist Karlheinz Fischer-Fürwentsches bestätigt, also Simon wars. Play It Again, SPAM (Diskussion) 15:05, 10. Jan. 2016 (CET)

Namensbestandteil "-enn"

Reine Neugierde auf den Namen, nicht auf die Vita der Person: Wie kommt das klein geschriebene "-enn" im Namen der Kriminologin Rita Steffes-enn zustande. --Maasikaru (Diskussion) 09:05, 6. Jan. 2016 (CET)

GoogleBooks Suche => Steffes-lai, Steffes-mics (von miis — Bartholomäus herkommend) <= Play It Again, SPAM (Diskussion) 09:25, 6. Jan. 2016 (CET)
Diese Antwort würde mich als Fragesteller eher verstören als weiterbringen. --Mauerquadrant (Diskussion) 10:04, 6. Jan. 2016 (CET)
Hat es zunächst durchaus:-> Ich konnte aber dann über diese Hilfe ermitteln, dass es sich offenbar um eine lokale Sonderform aus Laubach (Eifel) handelt. Der Googlebook-Schnipsel gehört zu einem Aufsatz: Peter Wimmert: Die Eigennamen des Dorfes Laubach, Kr. Cochem : Ein Beitrag zur Kenntnis der Eifeler Mundart. in: Zeitschrift für Deutsche Mundarten, 6. Jahrg. (1911), S. 36-40, erhältlich über JSTOR. Was "-enn" bedeutet, weiss ich zwar immer noch nicht. Meine Neugier ist aber gestillt. Deshalb danke. --Maasikaru (Diskussion) 10:08, 6. Jan. 2016 (CET)
Und wie kommt man bei der suche nach Steffes-enn auf die Idee nach Steffes-lai oder Steffes-mics zu suchen? Ohne Erklärung dieses Zwischenschrittes ist das eine lieblos hingeklatschte Antwort. --Mauerquadrant (Diskussion) 10:35, 6. Jan. 2016 (CET)
Ich habe versucht, die Ref. zu verlinken. Wenn man das aber macht, wird genau dort abgeschitten, wo es interessant wird. Dann habe ich herumprobiert und gefunden, dass mit der vorgegebenen Suche "am meisten sichtbar ist". Zu meiner tiefen Befriedigung hat der Frager den Faden aufgenommen - und daran gezogen. Play It Again, SPAM (Diskussion) 10:49, 6. Jan. 2016 (CET)

„Now he discovered that secret from which one never recovers, that even in the most perfect love one person loves less profoundly than the other.“

Thornton Wilder in The Bridge of San Luis Rey
Ich würde mal in Richtung -senn denken. Der Steffen, der Senn(er) war als die Namen vergeben wurden.--2003:75:AF0A:E200:7161:9489:2EF4:BA32 10:40, 6. Jan. 2016 (CET)

Senner in der Eifel? „So findet man: Steffes-lai, Steffes-mies (von miis — Bartholomäus herkommend), Steffes- tun {tun = Anton), Steffes-ollig, Steffes-enn, Stoffes -hoff und. Steffes- Holländer.“ Nichts davon führt Wimmert auf eine Berufstätigkeit zurück. Auch heute noch bleiben manche Rätsel ungelöst. --Vsop (Diskussion) 10:59, 6. Jan. 2016 (CET)

Fahr mal in die Eifel. Berge wie im Allgäu.--2003:75:AF0A:E200:25C7:3AB:F209:EF3E 13:37, 6. Jan. 2016 (CET)
So isses! Vielleicht käme man durch Genealogie dieser Familien weiter (anfängliche Doppelbenennung, dann Verkürzung).
Es hat (auch) nichts mit dem bretonischen Suffix "-enn" zu tun, aber die oben im Link angegebenen Beispiele machen klar, dass es wohl irgendeine Abkürzung eines Namenssuffixes (Heiligenname, topographischer Zusatz, ... whatever) ist, der erlaubte, die Vielfalt der Steffes (Stephans?) auseinanderzuhalten. Play It Again, SPAM (Diskussion) 11:02, 6. Jan. 2016 (CET)
Besser verlinkt. Hört sich für mich an wie Änn, Koseform von Anna, ich bin aber etwas weiter südlich zuhause. --Pp.paul.4 (Diskussion) 12:33, 6. Jan. 2016 (CET)
Teufel auch!
TF: Da steht etwas von französischen Einwanderern - und hier haben Zugezogene dieser Regionen den Zunamen "von der Bretagne"... Vielleicht hat es doch etwas mit dem bretonischen "enn" zu tun? Die "Steffes-in" ? "in-Anhängung" - gabs ja im Deutschen auch. Play It Again, SPAM (Diskussion) 13:11, 6. Jan. 2016 (CET)
Finnische Nachnamen wie z.B. Kimi Räikkönen? --Hans Haase (有问题吗) 13:26, 6. Jan. 2016 (CET)

gibt es zu dieser namens-auffälligkeit (wie nennt man das phänomen? gibts da ein fachbegriff?) einen wiki-artikel? --Dirk <°°> ID 30601 20:20, 6. Jan. 2016 (CET)

Genanntname beschreibt ein ähnliches Phänomen. Und Benutzer:Hans Haase sei herzlich gratuliert zu der nicht zu bremsenden Schamlosigkeit, mit der er sich wieder mal mit Blödsinn hervortut. --Vsop (Diskussion) 13:22, 7. Jan. 2016 (CET)

Ist das vielleicht nur ein Druckfehler für eigentlich "-Enn", also ein gewöhnlicher Doppelname? Ansonsten könnte man ja bei wirklichem Interesse die Person selbst fragen, da sie offenbar noch lebt; Für Historiker z.B. ist so ein Umstand immer ein Glücksfall. --84.135.153.125 13:45, 7. Jan. 2016 (CET)

Nein, das gehört so. Ich habe mal in der Moselgegend gelebt und kannte jemanden dieses Namens. Der wusste aber auch nur, dass es eben so ist. Grüße Dumbox (Diskussion) 13:53, 7. Jan. 2016 (CET)
Der Artikel Patronym brachte mir Suchbegriffe, die zu einem Herrn aus Österreich führten.[27] Man könnte ihn fragen. --Hans Haase (有问题吗) 03:50, 8. Jan. 2016 (CET)
Weitere Recherche deutet vage auf Färöer hin. --Hans Haase (有问题吗) 19:44, 10. Jan. 2016 (CET)
Bitte, Hans! Warst du schon mal in der Eifel? Die österreichischen und ganz besonders die färöerischen Einflüsse dort halten sich in Grenzen. Grüße Dumbox (Diskussion) 19:53, 10. Jan. 2016 (CET)

Wofür war der Milliardenkredit an die DDR (Strauß/Schalck-Golodkowski)?

Erhalt der DDR? Zweckgebunden in der BRD auszugeben, damit Wirtschaftsförderung für die BRD Industrie?--Wikiseidank (Diskussion) 15:44, 6. Jan. 2016 (CET)

Ende des ersten Absatzes im Artikel Alexander Schalck-Golodkowski und dem ref folgen. --mw (Diskussion) 15:49, 6. Jan. 2016 (CET)
(Schlecht geschriebene Prosa) Durfte die DDR das Geld frei verwenden? Musste dies in die BRD (für Warenkäufe) zurückfließen?--Wikiseidank (Diskussion) 16:15, 6. Jan. 2016 (CET)
Die DDR hat auch öfters Kredite ausgehandelt und sie nicht abgerufen, auch später nach 1983 noch. Es sollte immer nur die Liquidität des Staatshaushaltes gewährleistet sein. Irgendwie war ja Wirtschaft und Staat eine Suppe. Ein Staatsbankrott der DDR war zum damaligen Zeitpunkt nicht im Interesse des Westens, wegen der unbekannten Reaktion der damaligen UDSSR. Also hat der Westen sich mit Bedingungen zurückgehalten, wohl hoffend, dass die sich zu einem passenderen Zeitpunkt überschulden. --2003:75:AF0A:E200:25C7:3AB:F209:EF3E 16:43, 6. Jan. 2016 (CET)
Der Staatsbankrott der DDR ist eine Zeitungsente. --Pölkkyposkisolisti 16:44, 6. Jan. 2016 (CET)
Da gebe ich Dir recht, die Wende kam aus politischen Gründen, der Bankrott stand lediglich vor der Tür.--2003:75:AF0A:E200:25C7:3AB:F209:EF3E 17:16, 6. Jan. 2016 (CET)
Achwas? Natürlich eine unbequeme Meinung. Hier aber möglich. Sie sollte nur belegt werden. Grüße --KleinerTimmy (Diskussion) 18:12, 6. Jan. 2016 (CET)
Wie kann ein Kredit in DM dem DDR Staatshaushalt in MDN Liquidität verschaffen? Kann man mir meine Vermutung ausreden, dass dieser Kredit auch nichts anderes war, als "Entwicklungshilfe", also Kredite mit der Verpflichtung davon im Kreditgeberland einzukaufen (und gewisse politische Zusagen einzuhalten)?--Wikiseidank (Diskussion) 21:08, 6. Jan. 2016 (CET)
Was hat die DDR denn mit den Einnahmen aus den Intershops und dem Zwangsumtausch gemacht, wenn sie mit Westgeld gar nichts anfangen konnte? --Optimum (Diskussion) 22:20, 6. Jan. 2016 (CET)
Der Kollege will nur etwas bestimmtes hören, trotzdem noch eine Antwort. Die DDR musste für ihre Wirtschaft um den bekannt hohen Qualitätsstandard zu halten viel Material im Westen gegen Devisen einkaufen. Alles gab es nicht im Ostblock und wenn, dann nicht zur Genüge oder nicht in gewünschter Qualität. Selbst der Rum und die Bananen aus Cuba kosteten Devisen. Probleme machten auch die (Zwangs-)Lieferungen in die Bruderländer, die nur wertloses Geld einbrachten.--2003:75:AF0A:E200:BC88:BF56:461A:3164 23:06, 6. Jan. 2016 (CET)
Diese Darstellug ist stark vereinfachend. Die Wahrheit liegt dazwischen. Wohl war die DDR-Volkswirtschaft auf Gestattungsproduktion und Westexporte angewiesen, dennoch hätte ein Nichtzustandekommen von Milliardenkredit und Erdgas-Röhren-Geschäft nicht sicher einen DDR-Staatsbankrott ausgelöst. --Rôtkæppchen₆₈ 02:51, 7. Jan. 2016 (CET)
Der Kollege will nichts bestimmtes hören, sondern ist auf der Suche nach Gegenargumenten zu seiner Vermutung, die dadurch entsteht, da es nirgendwo genauer beschrieben ist. Vielleicht können es nicht mal die Beteiligte der DDR erklären, weil sie es eigentlich auch nicht verstanden haben - siehe Zins-Swap-Derivat Geschäfte, die diverse Kommunen in den 2000ern abgeschlossen haben, ohne diese zu verstehen--Wikiseidank (Diskussion) 08:25, 7. Jan. 2016 (CET)

Nebenziel war wohl auch das Kassieren der üblichen Vermittlerprovision (man munkelt konkret von 8,75 Mio DM). Ob sich Strauß und Schalck-Golodkowski das Geld geteilt haben oder schwarze Parteikassen gefüllt wurden ist Gegenstand zahlreicher Spekulationen. Rentiert haben dürfte es sich vermutlich im persönlichen Bereich der Beteiligten. Benutzerkennung: 43067 08:28, 7. Jan. 2016 (CET)

Die DDR benötigte für Geschäfte auf dem Weltmarkt Devisen. Da die eigene Mark für solcherlei Handel nämlich nichts wert war, waren Valutamark (Westgeld) hilfreich, denn mit Hilfe der DM konnte gehandelt werden. Ich verweise als Veranschaulichung immer wieder gerne auf die Kaffeekrise in der DDR, denn Kaffee konnte die DDR eben nur mit Dollar (oder eben DM) erwerben. Problematisch für die DDR Anfang der 1980er waren folgende Entwicklungen: durch die von Honecker durchgesetzte Einheit von Wirtschafts- und Sozialpolitik erhöhten sich die Schulden (subventionierte Miet-, Lebensmittel-, Schuhpreise und dgl. mehr, für deren Herstellung eben z. T. auch Waren eingeführt werden mußten > höherer Devisenbedarf) zum anderen traf die DDR - wenn auch verspätet - die Ölkrise, da die UdSSR Anfang der 1980er Jahre die Preise an den Weltmark anglich. Um das zu verdeutlichen: Offiziell gab es in der DDR keine Preiserhöhungen. Wenn nun also ein Brötchen 5 Pf. kostete, dann sind darin ja auch Produktionskosten enthalten. Diese änderten sich aber im Laufe der Jahre (z. B. steigende Erdölpreise > Verteuerung des Diesels > höhere Produktionskosten). Der Staat mußte also immer stärker subventionieren. Je mehr die Bürger kauften, desto mehr mußte hergestellt werden, desto höhere Kosten. Kredite in Valutamark halfen da ungemein.--IP-Los (Diskussion) 14:51, 8. Jan. 2016 (CET)
Hier wird zuviel durcheinander geschmissen. Erdöl aus der UdSSR wurde bestimmt nicht in DM gehandelt. Alternativen dazu konnte man sich auch nicht für DM auf dem Weltmarkt kaufen. Der "Weltmarkt" war auch bei eventuellem Vorhandensein von DM für die DDR (und RGW) nicht frei, siehe Embargos. Die "Kaffekrise" war in den 1980ern nicht mehr spürbar und mit Vietnam (habe jetzt keine Links, einfach googln) wurde die Möglichkeit geschaffen, Kaffee im sozialistischen Wirtschaftsgebiet zu beschaffen. (Noch heute profitiert der Weltmarkt von der Pionierarbeit der DDR im vietnamesischen Kaffeeanbau.). Zutreffend ist, dass die UdSSR durch die Veränderungen (Perestroika) die Wirtschaftsbeziehungen innerhalb des RGW und damit auch für die DDR verschlechterte. Die hier in Rede stehende 1 Mrd. DM hätte das langfristig jedoch nicht ausgleichen können und die Ökonomen der DDR (die sich natürlich erst mal gegen Parteibeschlüsse durchsetzen mussten) hatten ein langfristiges Denken (und kein Quartalsorientiertes, betriebswirtschaftliches)--Wikiseidank (Diskussion) 08:50, 9. Jan. 2016 (CET)
Das Durcheinanderschmeißen sehe ich auch. Die Mark der DDR war eine reine Binnenwährung, nicht konvertibel und damit für den Handel außerhalb des RGW überhaupt nicht zu gebrauchen. Zwischen RGW-Ländern wurde mit Transferrubeln "bezahlt". Beliebt waren auch Kompensationsgeschäfte, wie dieses. Die Zahlungsunfähigkeit der DDR war wohl nicht nur eine Zeitungsente, sondern durchaus zu befürchten. Der Binnen"markt" im RGW-Bereich wäre nie ein Problem gewesen, aber die DDR musste viele Dinge im NSW-Gebiet kaufen und hatte da ein Außenhandelsdefizit, dazu den Mangel an Devisen, also konvertibler Währung, so dass ein Kredit in einer harten konvertiblen Währung schon half, auch um weitere "Konsumgüter" zu kaufen, um Bedürfnisse der Bevölkerung zu befriedigen. --Hic et nunc disk WP:RM 09:37, 9. Jan. 2016 (CET)

Hier wird zuviel durcheinander geschmissen. In der Tat. Die "Kaffekrise" war in den 1980ern nicht mehr spürbar und mit Vietnam (habe jetzt keine Links, einfach googln) wurde die Möglichkeit geschaffen, Kaffee im sozialistischen Wirtschaftsgebiet zu beschaffen. Ja, man begann mit ersten Projektierungen Ende der 70iger/Anfang der 80iger. Da Pflanzen aber auch Erntereife benötigen, rechnete man erst Ende der 80iger mit ersten kleinen Ernten. Die DDR hatte letzten Endes vom vietnamesischen Kaffee genau nichts. Soviel dazu. M. W. ging es um die Kreditfähigkeit/Bonität der DDR auf dem internationalen Bankenmarkt. Wie auch heute viele Staaten, wurde die DDR damals von Ratingagenturen schlecht bewertet. Der Kredit half, den Buchstaben und das Plus nach dem Buchstaben wieder auf ordentliches Niveau zu heben. Es geht also nicht darum, ob mit dem Kredit bestimmte Waren gekauft wurden, sondern es war eine geldpolitische Entscheidung. Warum ausgerechnet Kommunistenfresser Strauß zur weiteren Stabilität der DDR beitrug ist eine andere Geschichte. Hier ist sicher auch ein Blick auf das damalige Verhältnis CDU-CSU hilfreich.--scif (Diskussion) 10:35, 9. Jan. 2016 (CET)

(BK) Hier wird nichts durcheinandergeworfen: Die UdSSR erhöhte in den 1970er Jahren die Preise für Erdöl stetig, um sie an das Weltmarktniveau anzupassen (zumal die DDR vorher durch den Wiederverkauf Gewinne erzielt hatte). Da die DDR dafür andere Güter lieferte, mußte sie mehr liefern. Die DDR mußte also mehr Leistungen für dieselbe Menge Öl aufbringen. Außerdem fehlten diese Güter für den Handel mit dem sogenannten Nichtsozialistischen Wirtschaftsgebiet.
Die Kaffeekrise habe ich nur als Veranschaulichung für die Wichtigkeit von Devisen gewählt. Aber: Die Beschaffung von Kaffee war auch danach immer noch ein Problem, soll heißen, die DDR mußte dafür immer mehr Geld ausgeben. Die DDR-Führung hatte registrieren müssen, daß die Ersatzlösungen Unmut unter der Bevölkerung auslösten. Man mußte also weiterhin (aus Sicht der Planungskommission) teuren Kaffee importieren. Natürlich wurde nun auch Vietnam gefördert, aber profitieren konnte die DDR davon nicht mehr, denn der massive Wachstum dieses Sektors setzte in Vietnam erst in den 1990er Jahren ein, da gab es die DDR nicht mehr. Die DDR war also bis 1990 von den Weltmarktpreisen abhängig, Vietnam bot da wenig Entlastung. Schaut man sich die Kaffeepreise in den 1980er Jahren an, dann zehrte das an den Devisenausgaben. Dieses Problem hatte die DDR natürlich auch mit anderen Waren.
Was den Kredit betrifft: Die DDR hatte 1982 laut Bundesbank 25,1 Mrd. DM Schulden und damit ein Liquiditätsproblem, da kaum noch Banken DDR-Kredite finanzieren wollten. Der Kredit half, diese Schulden abzubauen, um so auch wieder stärkere Unterstützung durch Banken erlangen zu können.--IP-Los (Diskussion) 11:22, 9. Jan. 2016 (CET)
Konsumgüter hat die DDR (erst mal, siehe unten) nicht im westlichen Ausland gegen DM gekauft. Das musste die DDR Produktion bzw. der RGW schon selbst schaffen. Anders könnte das bei Schlüsseltechnologien (bspw. IT) ausgesehen haben, aber dagegen stand ja bspw. COCOM. Zur Geldbeschaffung (Eigenkapitalnachweis) bei westlichen Banken ist mit Sicherheit absolut abwegig (wie sollten die zurückgezahlt werden?). Was in den 1980ern aufgefallen ist, dass Konsumgüter aus der BRD in der DDR (zumindest Berlin;o) verfügbar wurden, obwohl es ausreichende (wenn auch nicht so attraktive) Alternativen aus der DDR Konsumgüterproduktion gab, bspw.: Adidas Sportschuhe, Levis 501, VW Golf, HiFi, GEMA Rechte für westliche Popmusik usw.. Daher die Vermutung(!) die DDR konnte/musste mit dem Milliardenkredit westliche Konsumgüter beschaffen. Die BRD gab der DDR Geld, die vollständig in die BRD Wirtschaft zurückflossen = "Wirtschaftsförderung" nach dem Modell Entwicklungshilfe (oder Griechenland), Geldgeben mit der Verbindung von Bedingungen und gleichzeitigem vollständigen Rückfluss.--Wikiseidank (Diskussion) 11:29, 10. Jan. 2016 (CET)
Die Produkte, die du meinst, wurden zumeist in der DDR hergestellt, erhältlich waren sie überwiegend per Genex.
Konsumgüter hat die DDR (erst mal, siehe unten) nicht im westlichen Ausland gegen DM gekauft. Kaffee konnte man in der DDR nicht anbauen, die RGW konnte ihn auch nicht beschaffen, also mußte der im NSW erworben werden. Das galt auch für andere Rohstoffe. Dafür benötigte man harte Devisen (Dollar, DM).
Zur Geldbeschaffung (Eigenkapitalnachweis) bei westlichen Banken ist mit Sicherheit absolut abwegig (wie sollten die zurückgezahlt werden?). So wie das auch heute noch geschieht - durch Handelsüberschüsse. Deshalb hat man vieles in den Westen exportiert. Die DDR arbeitete eng mit Westunternehmen zusammen (VW, Quelle, Aldi usw.) und lieferte die Waren dann in die Bundesrepublik. Problematisch war dann, daß bestimmte exportierte Waren dann im Inland fehlten (Beispiele: Radeberger, Wernersgrüner - im Osten gab es diese Biersorten nur begrenzt, z. B. in der Mitropa zu entsprechenden Preisen, Grabower Küßchen usw.) oder es gab sie nur in minderwertiger Qualität (Ausschußware).--IP-Los (Diskussion) 15:57, 10. Jan. 2016 (CET)

Hab ich bzgl. Kaffe und Liquidität was anderes behauptet? Es ist nur eine höchst abenteuerliche Vorstlleung, das mancher glaubt, das irgendwelche Westwaren konkret mit den Strauß-Milliarden bezahlt worden sind. Erstens halte ich es für kaum vorstellbar, das man da konkrete Ausgaben konkreten Waren zuordnen kann, zum anderen ist das Auftauchen westlicher Konsumgüter kein Indiz für die Straußmilliarden. Speziell beim so beliebten Thema Kaffe setzte die Staatliche Plankommission zu einem gewissen Prozentsatz auf Oma Erna, die Kaffee aus dem Westen mitbrachte bzw. auf die Westpakete. Bei anderen westlichen Konsumgütern fallen mir die Namen Delikat, Exquisit und Inter-Shop ein. Und wenn wir schon bei Sportschuhen sind: m. W. ließ eine Marke auch in der DDR herstellen, zum anderen lief da viel über den Leistungssport. Speziell im Fußball wurde in den 80igern z. T. bis in die Bezirksliga mit adidas gespielt.--scif (Diskussion) 19:15, 10. Jan. 2016 (CET)

Es gab aber auch eine Adidas-Gestattungsproduktion im Ostblock. Somit waren Adidas-Schuhe auch für Transferrubel erhältlich. Auch hat die DDR Elektro-Hausgeräte und Unterhaltungselektronik auch nach Westdeutschland exportiert. Die Gestattungsproduktion wurde bereits genannt. Von den Praktica-Kameras wurde der Ausschuss in der DDR verkauft, die fehlerfreie Ware in Westdeutschland. Der westdeutsche Anbieter Foto-Quelle hat Orwo-Filme und -Fotopapiere (Vorkriegs-„Agfacolor Neu“ von 1936) unter der Eigenmarke Revue verkauft. --Rôtkæppchen₆₈ 19:25, 10. Jan. 2016 (CET)
@scif Dich habe ich ja auch gar nicht gemeint, sondern Deinen Punkt sogar noch weiter ausgeführt. Die Strauß-Milliarden waren nicht dazu da, einfach mal Produkte zu erwerben, sondern die Liquiditätsprobleme zu beheben.
@Rotkäppchen Da gab es ja eine Riesenpalette, z. B. auch Möbel, Strumpfhosen usw. Teilweise entfernte man dann im Westen den Hinweis auf das Herkunftsland.--IP-Los (Diskussion) 19:53, 10. Jan. 2016 (CET)
Ich habe beides gesehen: Im Westen verkaufte Möbel und Geschirr mit „Hergestellt in der DDR“ bzw „Made in GDR“ auf der Rückseite und ein DDR-Radio mit Philips vorne- und „Made in Germany“ hintendrauf und auf einigen Bauteilen. Der Netzstecker des Radios hatte dieselbe Form wie der einer von meinem Großvater aus der DDR mitgebrachten Tischleuchte. --Rôtkæppchen₆₈ 22:25, 10. Jan. 2016 (CET)

Panieren: durchgegartes Fleisch ohne "schwarze" Panierung?

(Hat ja auch was von Physik und Chemie.) Wenn man die Panierung nicht verbrannt haben möchte, aber sicher sein will, dass das Fleisch durchgegart ist (auch bei Verwendung von Tiefkühlkost), sollte man es dann vorher anbraten und anschließend mit der Panierung durchbraten?--Wikiseidank (Diskussion) 11:35, 10. Jan. 2016 (CET)

Nein. Die Panierung ist ja nicht nur optisch begründet sondern sie hält auch den Saft im Fleisch. Die Regulierung der Garstufen erfolgt über die Fleischdicke. max 1/2 cm dann ist das Fleisch durch und die Panade nicht verbrannt. Aber nicht vergssen "Schnitzel muss schwimmen" TK im übrigen bitte nicht und wenn, nach Packungsanweisung.--Graf Umarov (Diskussion) 11:53, 10. Jan. 2016 (CET)
Wenn das Fleisch noch nicht gar, die Panierung aber schon angebrannt ist, war die Pfanne zu heiß. Du musst paniertes Fleisch in tiefem Fett, z. B. Butterschmalz, so mittelheiß braten bis es schön goldbraun ist. Dan ist es auch gar. Vorausgesetzt natürlich, es ist kein zu dickes Stück. Rainer Z ... 11:58, 10. Jan. 2016 (CET)
Wenn die Panade schwarz wird, ist das Fett/Öl zu heiss. Meist benutzt man auch zuwenig davon, die Stücke müssen eigentlich darin schwimmen können (Soll heissen das Fleisch sollte nicht auf dem Boden der Pfanne aufliegen). Das überschüssige Fett kriegt man danach weg, in dem man das gegarte Stück kurz auf Kuchenkrepp/-papier legt. --Bobo11 (Diskussion) 12:07, 10. Jan. 2016 (CET)
Ja so ist es, das Fett leitet die Wärme besser als Luft und bringt sie durch die Panade Panierung ins Fleisch. Diese Öko-Fettvermeider und Sparer am falschen Platz predigen immer wenig Öl. Da brennt die Panade Panierung an, ist dennoch voller Öl oder Fett, das das Paniermehl aufgesaugt hat und das Fleisch ist nicht nicht gegart. Ob panierter Camembert, Chicken Nuggets oder andere Hühnerteile bei KFC, alles wird in der Panade Panierung frittiert. In der chinesischen Küche gibt es in an die Panade Panierung angelehnten ähnlichen Teig auch Frittiertes. Da darf das Schnitzel hinterher auch abtropfen, wenn es aus der Pfanne kommt. Die Temperaturschwankungen schluckt die Fritteuse mit der Masse der Fettmenge besser als eine Pfanne. --Hans Haase (有问题吗) 12:18, 10. Jan. 2016 (CET)
Öko-Fettvermeider? Was hat "Fettvermeidung" mit "öko(logischem Anbau)" zu tun? Man kann auch mit viel ökologischem Öl sein Schnitzel aus einer Biolandwirtschaft schwimmen lassen. Mir geht die Diskreditierung ökologischer Lebensweisen langsam ziemlich auf den Sack. 90.184.23.200 14:00, 10. Jan. 2016 (CET)
Du siehst wo das Fett dennoch hingelangt, ob viel oder zu wenig. Es wird mitgegessen, nur hat es seinen Zweck verfehlt wenn nicht genügend davon benutzt wird. Diese Vermeidungszwänge sind eine deutsche Krankheit, eine Missliebigkeit ohne nachweisbaren gegenwert, wenn bereits nachgewiesen ist, was außer Acrylamid da noch so alles entsteht. Die USA beschäftigen rund die Hälfte der deutschen Wissenschaftler, weil es diese sinnfreien Sparzwänge dort nicht gibt. Wer sie sucht, kann ja irgendwelchen Sekten beitreten. Ein weiteres Beispiel ist Adolf Hitler, der sich aus Prinzip falsch ernäherte und seine der Parkinson-Krankheit gleichenden Symptome des Zitterns der Hände, das auch in der Sprache zu hören war mit hoher Wahrscheinlichkeit erst hervorrief. Hätte er das abgestellt, wäre er vielleicht auf normale Gedanken gekommen. Stattdessen beute er einen Verwaltungswasserkopf zur Vernichtung seiner Sündenböcke auf und folgte damit allenfalls dem Parkinsonschen Gesetz. Wer so spart, spart zuerst am Verstand, dann am Wissen und gewinnt damit allenfalls den Darwin Award. Der Geisteskranke aus Braunau nahm noch Millionen seiner Spezies mit in den Tod. Das angebrannte Essen ist kein Garant alt zu werden, aber nicht ausreichend sicher, die Rentenkasse zu entlasten. Spare am richtigen Platz, und du kommst persönlich viel weiter. Unser Duales Ausbildungssystem reduziert sich grade wieder auf auf die Produktion von Pappenheimern. Es erfindet neue Einwegsqualifikationen, die den Auszubildenden nicht bis zur Rente reicht. Das Handweg klagt über Nachwuchssorgen, die Ansprüche der Schulbildung werden heruntergeschraubt, die Presse vermeidet es, die unangenehme Wahrheit auszusprechen, ohne Studium bist Du kein anständiger Mensch. Die Politik verkündet, das ihr duales Ausbildungssystem weltweit Nachahmung finden würde und redet bei Flüchtlingen von ausgebildeten Fachkräften, während sie bisher der Qualifikation der eigenen Bevölkerung Steine in den Weg legte. Selbst der «Terrorfreund» Gadaffi zahlte den Studierenden das Studium und vergab das Startkapital zur Eröffnung von Betrieben. In Deutschland ist Sozialhilfe mit den Studium unvereinbar. Man beschäftigte sich stattdessen damit, dass Maler und Stukkateure, die bisher ihre Gerüste selbst aufbauten, den neu erfundenen Beruf des Gerüstbauers erlernen mussten. Dasselbe Märchen durften KFZ-Mechaniker anhören, die nun Mechatroniker werden mussten aber von Elektronik keine Ahnung hatten. Den Heizungsbauern, die an der Steuerung modernden Anlagen verzweifelten, genehmigte man dafür das Verlegen von Abwasserleitungen. Arbeitgeber und Politiker lamentierten über den Frachkräftemangel, den sie bis dahin selbst verursacht hatten. Auch ist den Kultusministerien die Gedichtsinterpretation wichtiger als Grundlagenwissen, kaufmännische Allgemeinbildung und Betriebswirtschaft. Danach erzählt man der arbeitenden Bevölkerung, dass sie Verantwortung für ihren Nachwuchs zu übernehmen hätten als ob sie das nicht von selbst tun würden und stellt sie nicht nur auf sich selbst, sondern zockt sie noch ab. Die Abgaben sind da vielseitig und die Steuern absichtlich kompliziert. Die Politik religiös betrachtet: Es grenzt an Gotteslästerung, das von Gott gegebene Hirn nicht richtig einzusetzen. Daher bleibt zusagen: Nicht drüber aufregen, sondern draus lernen und nicht den Binsenweisheiten folgen. --Hans Haase (有问题吗) 17:29, 10. Jan. 2016 (CET)
Die ökologische Lebensweisen propagierenden Protagonisten sorgen doch selbst für dieses Image, indem sie ab und zu „vorschlagen“ irgendwelche in ihren Augen unökologische Dinge gesetzlich zu regeln und dann nach einem gesellschaftlichen Shitstorm kleinlaut zurückrudern. Wer ist denn auf die Idee gekommen, an einem Tag in der Woche Fleischkonsum zu verbieten? Wer ist denn auf die Idee gekommen, die Strompreise durch Zwangsvermarktung nachhaltiger Energien dauerhaft zu erhöhen? Wer ist denn auf die Idee gekommen, man könne durch Alkoholverkaufsverbote Jugendliche davon abhalten, sich zu betrinken? Wer ist denn auf die Idee gekommen, Fleischkonsum sei böse und propagiert die ideologisch motivierte Irrlehre, eine Ernährung ohne tierische Bestandteile sei nachhaltig, umweltfreundlich und gesund? --Rôtkæppchen₆₈ 14:31, 10. Jan. 2016 (CET)
Ich bin auch manchmal genervt von missionarischen Ökos. Ein Fleischverbot haben die Grünen aber nie vorgeschlagen, es wurde nur genüsslich so ausgelegt. Hat aber alles eh nichts mit der Frage zu tun. Rainer Z ... 14:56, 10. Jan. 2016 (CET)
Schon allein der Gedanke, Fleisch- und Gemüsekonsum gesetzlich regeln zu wollen, zeugt vom überbordenden Bevormundungswahn mancher Politiker. --Rôtkæppchen₆₈ 15:11, 10. Jan. 2016 (CET)
Nunja, wenn man allerdings solche Untersuchungen sieht, die zeigen, dass, wenn alle so weiter Fleisch konsumieren wie sie wollen, wir den Klimawandel nicht abwenden können: http://link.springer.com/article/10.1007/s10584-014-1197-x und noch besser: http://link.springer.com/article/10.1007%2Fs10584-014-1104-5 Insofern ist wären entsprechende Regelungen keine Bevormundung, sondern eine Abwägung kurzfristige Konsumegoismen der aktuellen Generationen vs. Klimasitation der nachfolgenden Generationen. Zukunftigere Generationen können sich halt noch nicht äussern. Da ist es doch nut begrüssenswert, wenn jemand ihre Stimme für sie erhebt. 90.184.23.200 16:01, 10. Jan. 2016 (CET)
Öhm, du glaubst ernsthaft, dass es für öffentliche Kantinen und Schulen jetzt keine Vorgaben gibt, was die machen dürfen und sollen? Tatsächlich. -- southpark 16:06, 10. Jan. 2016 (CET)

"Panierung"?--Heletz (Diskussion) 14:05, 10. Jan. 2016 (CET) Das heißt doch Panade?

Ne. Panierung wird knusprig. Panade ist Pampe. Geoz (Diskussion) 14:17, 10. Jan. 2016 (CET)
Also, ich glaube, die Pampe Panade wird durch Einlegen in heißes Fett knusprig. --Heletz (Diskussion) 14:37, 10. Jan. 2016 (CET)
Lies die Artikel Panieren und Panade und lass Dich vom Gegenteil überzeugen. --Rôtkæppchen₆₈ 14:41, 10. Jan. 2016 (CET)
Umgangssprachlich sagen die meisten Panade. Küchensprachlich korrekt ist aber Panierung – der Eindeutigkeit wegen. Rainer Z ... 14:56, 10. Jan. 2016 (CET)
Die Panierung (Vorgang und Resultat) ist das Ergebnis der Behandlung des Fleisches mit Panade (Mittel) - auch hier. Play It Again, SPAM (Diskussion) 15:22, 10. Jan. 2016 (CET)
Klar, aber warum findet man im Netz praktisch kein Panademehl, aber sehr wohl Paniermehl? Geoz (Diskussion) 17:38, 10. Jan. 2016 (CET)

 Info: Nicht alles was in Wikipedia steht ist richtig. Dafür war das annerkannte Wissen der Welt historisch schon viel zu oft Humbug. --Graf Umarov (Diskussion) 15:38, 10. Jan. 2016 (CET)

Selbstverständlich habe ich vorher den Artikel gelesen, daher Panierung. Öko nein, aber fettvermeidend ja. Aber dann geht - wie bei vegetarischer Ernährung - eben keine (vernünftige) Panierung.--Wikiseidank (Diskussion) 16:46, 10. Jan. 2016 (CET)

Umlaufgitter

Fußgängerüberwege über Straßenbahntrasse in Krefeld

Zählt dieses Bild entgegen der Beschreibung zu den Umlaufgittern? Mit Drängelgitter wird ähnliches gefunden, aber der Artikeltext beschreibt das Umlaufgitter mit „aus U-förmigen Teilen“. Ist da der Weg durch das Gitter oder die Rohrform des Gitters beim Bau gemeint? Der Z-förmige Weg ist ja bereits die wirksame Bremse für Passanten. --Hans Haase (有问题吗) 18:50, 10. Jan. 2016 (CET)

GoogleBildersuche => Umlaufgitter <= bestätigt es. Play It Again, SPAM (Diskussion) 18:53, 10. Jan. 2016 (CET)
Was sagt die DIN oder sonstige Richtlinie? --Hans Haase (有问题吗) 19:46, 10. Jan. 2016 (CET)
Siehe Umlaufgitter#Mindestbreiten, dort verlinkte Artikel und in diesen Artikeln behandelte Werke. --Rôtkæppchen₆₈ 20:06, 10. Jan. 2016 (CET)

Russisches bekanntes Lied

Ich suche den Name ein russisches bekanntes Lied, aber ich finde es nicht. Vielleicht ihr könnt mir helfen. Teil des Liedes ist am Ende dieser Video zu hören, ab der 1:53:00. Kommt es euch bekannt?

--Leonprimer (Diskussion) 19:50, 10. Jan. 2016 (CET)

Das ist Moskau (Lied) der deutschen Band Dschinghis Khan. --Komischn (Diskussion) 20:04, 10. Jan. 2016 (CET)

Danke dir. Leonprimer (Diskussion) 20:21, 10. Jan. 2016 (CET)

Und es ist kein russisches Lied. --j.budissin+/- 20:33, 10. Jan. 2016 (CET)

@Leonprimer Ja am Schluss wird "Moskau" gespielt, aber in der Minute 1:53:00 höre ich Funiculì, Funiculà und If You're Happy and You Know It ... --King Rk (Diskussion) 08:12, 11. Jan. 2016 (CET)

Archivierung dieses Abschnittes wurde gewünscht von: Speravir (Disk.) 23:38, 12. Jan. 2016 (CET)

zwei Fragen Physik: Lichtgeschwindigkeit/Gravitation

Hallo, ich hab zwei Fragen an euch:

1) Angenommen man hätte einen 500.000 km langen Stab und bewegt das eine Ende. Dass das andere Ende sich natürlich nicht sofort bewegt, ist mir schon klar. Nur warum nicht? Was führt dazu, dass die Bewegung des Stabs trotzdem langsamer als die Lichtgeschwindigkeit ist? Ist es der Impuls oder was verbirgt sich physikalisch dahinter? --2003:7A:ED4C:AD68:6934:E78E:EE61:D9CD 19:22, 8. Jan. 2016 (CET)

2) Im Schulunterricht wurde uns gezeigt, dass Körper im Vakuum gleichschnell fallen. Wenn wir zwei Körper unterschiedlicher Masse nehmen und diese im Vakuum fallen, fällt der schwerere Körper dann (theoretisch?) schneller? Im newtonschen Gravitationsgesetz tauchen ja beide Massen auf.--2003:7A:ED4C:AD68:6934:E78E:EE61:D9CD 19:22, 8. Jan. 2016 (CET)

zu Frage Nr. 2 gab es ein berühmtes Experiment: https://www.youtube.com/watch?v=5C5_dOEyAfk Im Vakuum fallen die Körper (Hammer und Feder) gleich schnell. Mit Newtons Gesetz wird die Kraft berechnet und nicht die Geschwindigkeit. Um die größere Masse des Hammers auf die gleiche Geschwindigkeit wie die der Feder zu bringen, ist die größere Kraft aus Newtons Formel nötig. --Heldenzeuger (Diskussion) 19:35, 8. Jan. 2016 (CET)
Zu Frage 1, dein Stab ist niemals 100% starr (im Sinne von die Atome sind gegeneinander unbeweglich). Daher breitet sich die Information wie eine Welle durch den Körper aus. Da der Stab (bzw. die Atome aus denen er besteht) eine Masse besitzen, welche Träge ist, kann die Information auch nicht die Lichtgeschwindigkeit erreichen. --Jogo.obb (Diskussion) 19:54, 8. Jan. 2016 (CET)
Ein 500.000 km langer Stahlstab mit 1cm Durchmesser würde übrigens 300.000 t wiegen. Selbst wenn dieser Stab schwerelos im Weltraum schwebt, bleibt da noch die Massenträgheit. Die größte Rakete, Saturn V, wog nur 1/100stel davon. Mal eben so bewegen klappt also nicht. --Optimum (Diskussion) 20:22, 8. Jan. 2016 (CET)
Das ist für ein Gedankenexperiment unerheblich.
Stab: Reale Stäbe bestehen ja aus Atomen, die durch unterschiedliche elektromagnetische Kräfte (meist in kovalenten Bindungen) miteinander verbunden sind. Schon von Atom zu Atom breitet sich ein Kraftfeld, die für eine Beschleunigung erforderlich ist, nicht instantan aus. Fiktive Sräbe könnte man sich als starren Körper vorstellen. Wie sich ein solcher starrer Körper in relativistisch relevanter Größe bei Beschleunigung relativistisch verzerrt, ist nicht einfach zu beschreiben. Ein ähnliches Problem ist die Winkelbeschleunigung eines langen Stabes bis zu einer Winkelgeschwindigkeit, bei der die Bahngeschwindigkeit des Endes des Stabes sich der Lichtgeschwindigkeit annähert.
Zwei unterschiedlich schwere Körper beschleunigen dann gleich schnell, wenn schwere Masse und träge Masse einander äquivalent sind. Davon geht man aus. Ob es stimmt, wird noch erforscht, zum Beispiel in Falltürmen. --BlackEyedLion (Diskussion) 23:29, 8. Jan. 2016 (CET)
Die Stahlstabfrage wurde ja im Prinzip schon beantwortet, nochmal in meiner Formulierung: Es gibt keine starren Materialien. Es gibt nur Materialien, die dir in kleinen Dimensionen starr erscheinen, weil sich an einem Ende eingebrachte Bewegungen so schnell auf das andere Ende übertragen, daß du den Eindruck hast, der Körper bewege sich „als Ganzes“. Das tut er aber nie. Auch ein 1-m-Stab, der an einem Ende bewegt wird, wird von einer Körperschallwelle durchlaufen, die die übrigen Teilchen, aus denen er besteht, dieser Bewegung folgen läßt – aber das ist so weit unter der Wahrnehmungsgrenze, daß es einem nicht bewußt ist. Ich gehe mal, bis mir einer widerspricht, davon aus, daß sich diese Welle grundsätzlich mit der entsprechenden Schallgeschwindigkeit ausbreitet (in Stahl wären das knapp 8 km/s). --Kreuzschnabel 08:27, 9. Jan. 2016 (CET)
Die Fallfrage ist ganz einfach mit Logik zu beantworten: Ein schwerer Körper erfährt zwar eine höhere Gewichtskraft als ein leichter, ist aber auch entsprechend träger, benötigt also um denselben Faktor mehr Beschleunigungsarbeit, um auf dieselbe Geschwindigkeit zu kommen. So kürzt sich die Masse raus – zwei Körper im freien Fall werden in der gleichen Zeit um die gleiche Geschwindigkeit beschleunigt, auch wenn einer 1000-mal schwerer ist als der andere. Der schwerere hat dann allerdings im Vergleich zum leichten die 1000-fache kinetische Energie, tut beim Auftreffen auf Newtons Schädel also mehr weh. --Kreuzschnabel 08:33, 9. Jan. 2016 (CET)
Zur Fallfrage gibt es auch das folgende einfache (Gedanken- oder tatsächliches) Experiment: Wenn der schwerere Körper schneller fallen würde, was würde passieren, wenn wir zwei verschieden schwere Körper mit einer dünnen Schnur verbinden und fallen lassen? Fallen sie zusammen langsame als der schwerer Körper allein, weil der leichtere das Gesamtobjekt ausbremst? Oder schneller, weil sie zusammen ja noch schwerer sind? Das passt nicht zusammen, also muss die Falldauer von der Masse unabhängig sein. --132.230.195.196 10:25, 9. Jan. 2016 (CET)
Kreuzschnabel: Schallgeschwindigkeit ist richtig, beim oben diskutierten Stab dauert es also grob ein Tag, bis sich das andere Ende bewegt. In Neutronensternen kann die Schallgeschwindigkeit in die gleiche Größenordnung wie die Lichtgeschwindigkeit kommen, bleibt aber immer langsamer. --mfb (Diskussion) 11:22, 11. Jan. 2016 (CET)

Alle Wege führen zum Mond

Hallo! Klar, das folgende ist keine allgemeine Wissensfrage, aber es geht eher um Einschätzung von Distanzen und Relationen. Wenn man davon ausgeht, daß in ein paar Jahrzehnten diverse Länder und Unternehmen in der Lage sein werden zum Mond zu fliegen, stellt sich für mich die Frage, wie dies praktisch vonstatten gehen soll. Gehen wir einfach mal davon aus, daß alle über ähnliche Technik mit den selben Eigenschaften verfügen. Wie beim Flugverkehr dürfte es nicht zu einer Komplikation kommen, was die Koordination der Starts auf der Erde angeht. Aber ich frage mich seit längerem, gibt es da nicht ein "Nadelöhr", andem sich alle Flugobjekte unabhängig von den Startbedingungen treffen, bevor sie in die Annäherung/Landung an verschiedenen Stellen geraten?`Oder ist es entsprechend der Mondumlaufbahn eher eine Kette von Nadelöhren? Denn trotz Weltall und den 3 Dimensionen ist es ja eine Start-Zielreise auf einer Ebene.Oliver S.Y. (Diskussion) 15:56, 9. Jan. 2016 (CET)

Demnächst wird es so oder so, eher ein Problem sein eine Lücke im Weltraummüll zu finden, als das man auf den Routen zum Mond "Stau" hat. Aber es stimmt schon, es gibt nicht manche Route zum Mond, eigentlich nur eine idealle und enegrie effizente Bahn, wo die Raumschiffe sogar noch die selbe Geschwindigkeit haben werden. Aber diese Route ist also eher mit einer Einbahn zu vergleichen -di erst noch mit Tempomat befahren wird), denn mit einer engen Strasse mit Gegenverkeher. Entsprechend wäre es mit geregeltem Einfädeln zu lössen. Stau gäbe es am ersten auf der Warte-Umlaufbahn (Orbit) bevor die "letzte" Stufe (Apollo 11 Zündung Stuffe 3) gezündet wird uns die Erdumlaufbahn Richtung Mond verlassen wird. àhnliches gilt für den Abbrems Orbit beim Mond. Aber eben das setzt praktisch voraus, dass zur selben Sekunde zwei Raumschiffen zum Mond wollen (ICh glaub das ist definitiv zu regelen). --Bobo11 (Diskussion) 16:19, 9. Jan. 2016 (CET)
Ehrlich gesagt rechne ich nicht damit, dass das jemals zu einem größeren Problem werden wird. Reisen zum Mond werden frühestens dann in nennenswertem Umfang unternommen werden, wenn man Bodenschätze auf dem Mond gewinnbringend abbauen kann (oder wenn man seinen Müll da verklappen kann). Ansonsten hat sich da in den letzten Jahrzehnten wenig getan, und das wird sich auch in den nächsten Jahrzehnten nicht groß ändern. Warum auch: Der Mond ist a große nackerte Kugel - es gibt kein Leben auf dem Mond. Zur Zeit streiten sich auch nicht zwei Supermächte darum, wer es schneller und besser hinkriegt. Die Fortschrittsgläubigkeit wird in der kommenden Generation einen gewaltigen Dämpfer erleiden; die Probleme, sich hier auf der Erde um die Ressourcen zu streiten (aus Wirtschaftsflüchtlingen werden Umweltflüchtlinge); die wirtschaftlich benachteiligten Völker haben inzwischen verlässlichen Zugang zu Informationen und Kommunikationswegen, und mit Waffen werden sie ohnehin bestens versorgt - Spielereien wie Flüge zum Mond oder gar zum Mars werden in erster Linie stattfinden, um die Bevölkerung der Industrienationen von den wahren Problemen abzulenken, und das wird nicht zu Verkehrsstaus im Weltall führen. Wenn es überhaupt je so weit kommt. --Snevern 16:59, 9. Jan. 2016 (CET)
Hallo! Ich war eigentlich auch lange Deiner Meinung, bis ich eine interessante These las. Demnach entspricht es sowohl dem Macht- als auch Forschungsdrang der Menschheit ansich, neue Wege zu gehen. Nach der Eroberung von Mond und Mars stehen die Asteroiden und Monde der Großplaneten auf dem Plan. Es soll plausibler sein, Produktionsstätten für Bauteile von Raumschiffen auf dem Mond zu errichten, als diese einzeln hochzuschicken. Wenn ich mal den Vergleich mit einem Schiff wie der neuen Fregatte F125 aufstelle, 120 Mann Besatzung, Vorräte für 2 Jahr = 7000 Tonnen Gewicht. Da wird schon der Transport vonn Vorräten, Elektronik und Ausrüstung einen enormen Aufwand verschlingen. Genauso stellt sich ethisch die Frage, wie lange Menschen im All verbringen können, und anschließend wieder auf das Erdleben anpassbar sind. Muß nicht eine Art "Alterssitz" geschaffen werden, wo die -nauten ihr Leben konstruktiv weiterführen? Zur Frage, es ist ja auch keine "Einbahnstraße", sonder es wird ja auch einen Rückverkehr geben, der genauso zu regulieren ist, und das bedeutet über längere Distanzen im Raum einen "Parallelverkehr" in einem gewissen "Schlauch", nicht nur "Öhr", wo mind. 2 am Ende dieses Schlauchs/Tunnels stehen. Nur wie "breit" kann dieser sein, wenn man die Entfernungen sieht. Wenn man von Berlin nach fliegen Köln will, und nur 1,2 Grad daneben liegt, kommt man in Düsseldorf oder Aachen an, und wer will das schon :), beim Mond dürften geringere Abweichungen zum Verpassen führen. Was ungünstig ist, wenn er erst in 27 Tagen wieder vorbeikommt.Oliver S.Y. (Diskussion) 17:53, 9. Jan. 2016 (CET)
Also, das mit dem "Gegenverkehr" kann ich mir nicht so recht vorstellen. Mondflüge, jedenfalls die bisher üblichen, sind ja keine Direktflüge mit Fluchtgeschwindigkeit, sondern erweiterte Umlaufbahnen mit Trans-Lunar-Injection aus dem Parkorbit um die Erde und entsprechender Trans-Earth-Injection aus der Mondumlaufbahn. Dabei ändert sich der Drehsinn doch nicht, bzw. im Umkehrsinn um den Mond und dann wieder umgekehrt zur Erde, also vereinfacht: wie eine 8. Das ist doch eine Einbahnstraße, oder? (Ich bin aber kein Experte; it's all rocket science to me...). Grüße Dumbox (Diskussion) 18:38, 9. Jan. 2016 (CET)
P.S.: Das (sicher nicht ganz ernst Gemeinte) mit den 27 Tagen ist natürlich sehr irdisch gedacht. In irgendeine, wenn auch ungeplante, Mondumlaufbahn würde man schon kommen, wenn man nicht komplett in die falsche Richtung schießt, aber dann ist eh die Dingens am Dampfen. Hätte die TLI gar nicht geklappt, wären die Apollos, so war es ausgerechnet, auf einer Ellipse automatisch zur Erde zurückgeflogen ("free return"). Aber natürlich kann man unterwegs immer ein wenig nachregulieren, in Zukunft, mit besseren Antriebssystemen, sicher noch viel besser als damals bei Apollo - und die haben immerhin mit viel Improvisation Nr. 13 heimgebracht; eine Jahrhundertleistung, wenn du mich fragst. Grüße Dumbox (Diskussion) 19:05, 9. Jan. 2016 (CET)
P.P.S. Eigentlich wäre ich aktuell vielleicht wirklich lieber in D'dorf oder Aachen. ;) Grüße Dumbox (Diskussion) 19:05, 9. Jan. 2016 (CET)
Die Kurve die sie bei der Appolo geflogen sind ist auch die, die bei der Zeit-Energie-Rechung gewinnt. Und das schöne daran, dass es auch noch die Auto-Zurückkehr-Kurfe. Denn es ist ein der seltenen stabile Flugbahnen um zwei Schwerpunkte. Und weil sie stabil ist, wurde sie gewählt (Dazu kommt alles was im Weltraum nicht stabil ist, ist mit zusätzlichem Energieaufwand verbunden). Und Richtig, diese Flugbahn ist vereinfacht gesagt eine langezogene Acht zwischen Mond und Erde. Die eben dazu nur in eine Richtung gut funktioniert. Weil du von der Erde aus nur in eine Richtung den Schwung der Erdrotation mitnehmen kannst, in der Gegenrichtung musst du dagegen ankämpfen. Das heisst also du brauchst einiges mehr an Energie um auf Fluchtgeschwindigkeit zu kommen, wenn du gegen die Rotation startest. Dazu rotiert ja der Mond um die Erde und damit auch der Flugweg. Auserhalb der "normalen" Sateliten- und Müll-Orbite, wird es devinitv nicht wirklich eng sein. --Bobo11 (Diskussion) 20:10, 9. Jan. 2016 (CET)
Stabil ist die Umlaufbahn nicht - wenn man nichts weiter unternimmt, kommt man einmal zum Mond, zurück und danach zu nichts sinnvollem mehr.
Zur Ausgangsfrage: Das Weltall ist riesig. Selbst wenn plötzlich einmal am Tag irgendwas zum Mond fliegt (derzeit sind wir bei nur 1-2 Weltraumstarts pro Woche, und fast alle in niedrige Erdumlaufbahnen), dann haben die Objekte noch einen typischen Abstand von ~100,000 km. 100 Flüge zum Mond pro Tag? Dann sind es immer noch viele tausend Kilometer. Zum Vergleich: In der niedrigen Erdumlaufbahn kreisen einige tausend Satelliten und sonstige Objekte in einem kleineren Volumen als es bei der Reise zum Mond jedes Objekt hätte. Die ISS macht Ausweichmanöver, sobald diese Objekte näher als ein paar Kilometer kämen, und braucht nur ein paar davon pro Jahr. --mfb (Diskussion) 11:11, 11. Jan. 2016 (CET)

Wissenschaftssprache

Wann haebn deutsche (Natur-)wissenschaftler aufgehört, auf Deutsch zu publizieren bzw. allgemeiner, ab wann wird nur noch auf Englisch publiziert? Gibt es denn heute noch Fälle, wo Originalarbeiten nicht auf Englisch sind?--Antemister (Diskussion) 15:20, 10. Jan. 2016 (CET)

Artikel Wissenschaftssprache gelesen? --Buchling (Diskussion) 15:26, 10. Jan. 2016 (CET)
Ja, aber der beantwortet die Frage ja nicht.--Antemister (Diskussion) 15:31, 10. Jan. 2016 (CET)
Hier ein paar Zahlen, Argumente und Situationsberichte (der Artikel ist in Deutsch). Play It Again, SPAM (Diskussion) 15:41, 10. Jan. 2016 (CET)

Zumindest bei uns in den Wirtschaftswissenschaften gibt es heute noch Fachzeitschriften auf Deutsch. Seit etwa wohl zehn Jahren nimmt deren Bedeutung aber rapide ab. Ich habe im Hinterkopf, dass Deutsch spätestens mit dem Ende des Zweiten Weltkrieges durch Englisch abgelöst wurde. Einstein hat seine Artikel zuvor ja noch auf Deutsch publiziert, was es und heute ermöglicht seine Gedanken in unserer Muttersprache zu lesen. 90.184.23.200 15:55, 10. Jan. 2016 (CET)

In der Rechtswissenschaft (gehört zugegebenermaßen eher weniger zu den Naturwissenschaften...) gibt es natürlich auch englischsprachige Artikel - aber die beziehen sich meist nicht auf das deutsche Recht. Fachliteratur zur deutschen Rechtswissenschaft ist fast ausschließlich in deutscher Sprache geschrieben. --Snevern 16:30, 10. Jan. 2016 (CET)
In den Geowissenschaften hat Deutsch seinen Vorrang als wichtigste Sprache seinen Vorrang wohl erst in den 60ern endgültig verloren, mit dem Paradigmenwechsel hin zur Plattentektonik. In der Physik (aber auch in der Psychologie) war aber sicherlich der Exodus der deutschen Wissenschaftler jüdischer Abstammung in der Nazizeit ausschlaggebend. Geoz (Diskussion) 18:33, 10. Jan. 2016 (CET)
In vielen Geisteswissenschaften seit etwa 20 Jahren schleichend.. --Hachinger62 (Diskussion) 18:51, 10. Jan. 2016 (CET)
In vielen Wissenschaften schleichend (nicht nur Geistes-w) und auf "seit" kann ich mich nicht festlegen. Aber mit dem Bologna-Prozess erreichte dies wohl schon einen ersten Höhepunkt. Mittlerweile werden viele (von den Amis abgekupferte) Graduate Schools schon auf Englisch gehalten. Mein Filius (Sohn) hat in einer intern. Studiengruppe seine Bachelor-Arbeit bereits freiwillig auf Englisch geschrieben (Theoretische Physik), obwohl die Vorlesungen da noch auf Dt. waren. Bei den Master-Studien und -arbeiten ist Englisch vorgeschrieben (Universität zu Köln)--G-Michel-Hürth (Diskussion) 19:10, 10. Jan. 2016 (CET)
@Antemister: vielleicht bekommst Du hier eine Antwort auf Deine Frage. --91.89.10.94 23:05, 10. Jan. 2016 (CET)
Man kann ja über den Siegeszug des Englischen jammern, aber früher hätte man auf Latein publizieren müssen, um von der Wissenschaftsgemeinde rezipiert zu werden, sicher auch nicht leichter. Grüße Dumbox (Diskussion) 19:20, 10. Jan. 2016 (CET)
Noch früher war Altgriechisch hip und es soll auch eine Zeitlang Arabisch lingua franca der Wissenschaft gewesen sein, aber weniger in Europa. --Rôtkæppchen₆₈ 22:33, 10. Jan. 2016 (CET)
Hätte wohl besser die Klammern um (Natur-) weglassen sollen... Es war ja siche rnicht so das ab 1945 nichts mehr auf Deutsch publiziert wurden, sondern das eher allmählich auslief. Aber wann war das. Und klar, Publikationen zu deutschen Themen von deutschen Autoren werden auch in Zukunft meist auf deutsch geschrieben werden.--Antemister (Diskussion) 23:54, 10. Jan. 2016 (CET)
Das kommt bestimmt auf das Fachgebiet an. In der organischen Chemie war die Wissenschaftssprache lange Zeit Deutsch. Erst als Beilsteins Handbuch der Organischen Chemie zur englischen Sprache wechselte, wurde in der Chemie Englisch zur Wissenschaftssprache. Im Bereich Steuerrecht ist weltweit die meiste Fachliteratur auf Deutsch, weil das deutsche Steuerrecht das komplizierteste der Welt ist. --Rôtkæppchen₆₈ 01:52, 11. Jan. 2016 (CET)
Für die Physik: Archiv der Zeitschrift für Physik A (Kernphysik): https://link.springer.com/journal/volumesAndIssues/218. 1960 noch alles deutsch, 1965 erste Artikel von deutschsprachigen Autoren auf englisch, 1970 etwa halb-halb, 1975 noch einige Artikel auf deutsch (für jedes Jahr eine Ausgabe willkürlich herausgesucht). Die anderen Teile der Zeitschrift für Physik (andere Fachgebiete) sind auch einsehbar. --BlackEyedLion (Diskussion) 10:36, 11. Jan. 2016 (CET)

IE11: PDF ohne speichern öffnen

Wenn ich einen PDF-Link anklicke, fragt der IE seit kurzem "Wie möchten Sie mit xx.pdf verfahren?" Auswahlmöglichkeiten sind

- Öffnen (Die Datei wird nicht automatisch gespeichert)

- Speichern

- Speichern unter

Wähle ich die erste Option aus, geschieht aber nicht das Angekündigte, sondern am unteren Rand erscheint ein Balken "Möchten Sie xx.pdf von www.xyz.com speichern?" Nach der Auwahl von "Speichern" kann die Datei geöffnet werden und wurde auch im Downloads-Ordner abgespeichert.

Wie bekomme ich das alte Verhalten wieder, wo direkt ohne Nachfrage geöffnet wird (nicht im Browser, sondern im Acrobat Reader), ohne dass vorher gespeichert werden muss?

Danke --11:14, 5. Jan. 2016 (CET) (ohne Benutzername signierter Beitrag von IEprob.123 (Diskussion | Beiträge))

In der Systemsteuerung unter Indischen Optionen Internetoptionen oder im IE unter Optionen Programme und Dateien bzw. Dateien den PDF-Betrachter entfernen.
Sicherheitshinweis nebenbei: Im Acrobat Reader in den Voreinstellungen (STRG+K), unter „Java Script“ die die Ausführung dessen untersagen (Checkbox abwählen=Haken rausnehmen). --Hans Haase (有问题吗) 13:03, 5. Jan. 2016 (CET) Sag mal „Internet“ – „Inder nett!“
Kann dort nirgends den PDF-Betrachter entfernen. --IEprob.123 (Diskussion) 16:38, 5. Jan. 2016 (CET)
Welches Windows verwendest Du? IE11 gibt es für Windows 7, 8, 8.1 und 10. Welchen Adobe/Acrobat Reader verwendest Du? --Rôtkæppchen₆₈ 01:38, 6. Jan. 2016 (CET)
Win 7. Adobe Reader XI. --IEprob.123 (Diskussion) 10:20, 7. Jan. 2016 (CET)
Diese Kombination hab ich nicht. Ich kann nur Vista / XI oder 7 / DC bieten. --Rôtkæppchen₆₈ 20:24, 7. Jan. 2016 (CET)
[28] verweist auf [29] und dürfte Deine Lösung sein. --Hans Haase (有问题吗) 13:30, 10. Jan. 2016 (CET)
Das hatte ich schon erfolglos versucht. Aktiviere ich das Addon, wird die PDF im Browser angezeigt, deaktiviere ich es, kommt wieder die Speichern-Abfrage. --IEprob.123 (Diskussion) 08:43, 11. Jan. 2016 (CET)
Du möchtest, dass der Arcobat Reader nach dem Download die Datei öffnet. Mit der App tut er eingebettet das im Fenster des IE per ActiveX. Soll der Arcobat Reader selbst öffnen, lade eine größere PDF-Datei herunter und wähle öffnen. Mit der Checkbox auswählen, dass dies immer mit dieser Art von Datei (PDF) geschehen soll. Zuvor muss PDF. Firefox macht es so,[30] IE11 muss es auch können. Dieser Dialog[31] erscheint beim neueren IE unten. Die Optionen sind dort ebenso auswählbar. --Hans Haase (有问题吗) 12:19, 11. Jan. 2016 (CET)
Danke für deine Bemühungen, leider nicht zielführend. Wie ganz oben beschrieben, führt "Öffnen" nicht zum angekündigten Verhalten ("Die Datei wird nicht automatisch gespeichert" [32]), sondern es muss dann trotzdem gespeichert werden. Ich möchte, dass beim Klick auf einen Link die PDF im Adobe Reader geöffnet wird. Ohne irgendwelche Buttons dazwischen. --IEprob.123 (Diskussion) 23:27, 11. Jan. 2016 (CET)

Geschäftsmodell

Welche Fakten über ein Unternehmen (Geschäftsmodell) dürfen eigentlich juristisch legal kommuniziert werden?--Muroshi (Diskussion) 11:21, 5. Jan. 2016 (CET)

Alle, soweit nicht gesetzlich oder vertraglich verboten.--Vsop (Diskussion) 11:38, 5. Jan. 2016 (CET)
Google mal nach "Verschwiegenheitspflicht", ignoriere den WP-Artikel und schau Dir die Seiten der Handels-/Handwerkskammern an. Da stehen einige Beispiele für alles, was nicht geht. Gruß, --Benutzer:Apierta 18:28, 5. Jan. 2016 (CET)
Warum soll ich den WP-Artikel denn ignorieren? Ich spreche übrigens nicht aus der Mitarbeiterperspektive (Verschwiegenheitspflicht). --Muroshi (Diskussion) 22:38, 5. Jan. 2016 (CET)
Na, weil der WP-Artikel die ärztliche/anwältliche Schweigepflicht meint, und das was anderes als die Mitarbeiter-Verschwiegenheitspflicht ist. Wenn es Dir aber um die offenbar gar nicht geht, solltest Du vieleicht ein bisschen exakter sagen, was Du überhaupt wissen willst. Gruß, --Benutzer:Apierta 13:40, 6. Jan. 2016 (CET)
Ein Beispiel: Darf ich das Geschäftsmodell von Fielmann als Konsumentenschützer publik machen?--85.4.233.141 00:52, 7. Jan. 2016 (CET)
Das hilft immer noch nicht weiter. Was verstehst du hier genau unter „Geschäftsmodell“? --Jossi (Diskussion) 11:06, 8. Jan. 2016 (CET)
Dazu gibt es keine eindeutge Definition: https://de.wikipedia.org/wiki/Gesch%C3%A4ftsmodell Womit das Unternehmen Geld erwirtschaftet, wie seine Margen strukturiert sind, die Art und Weise wie es seine Produkte verkauft, Absatzkanäle, Nachhaltigkeit etc.--85.4.233.141 12:33, 8. Jan. 2016 (CET)
Solange du nicht gegen vertragliche Verpflichtungen verstößt, keine dir anvertrauten oder auf illegalem Wege beschafften Informationen benutzt, deine Behauptungen gerichtsfest belegen kannst und keine Schmähkritik betreibst, kannst du selbstverständlich die Geschäftspolitik einer Firma öffentlich kritisieren. --Jossi (Diskussion) 21:53, 10. Jan. 2016 (CET)
Illegal beschaffte Infos zu benutzen ist illegal? --Distelfinck (Diskussion) 22:03, 10. Jan. 2016 (CET)
Ich meinte natürlich Informationen, die du dir auf illegale Weise beschafft hast. Da illegal=ungesetzlich, könntest du damit Ärger bekommen. (Whistleblower gehen dieses Risiko bewusst ein.) --Jossi (Diskussion) 16:17, 11. Jan. 2016 (CET)
Hat der Deutsche Staat mit Schweizer Bankkundendaten auch schon gemacht. Sogar gekauft hat er sie. Aber Staatsakteure stehen vielleicht ohnehin in einer anderen Kategorie.--85.4.233.141 21:12, 11. Jan. 2016 (CET)
Was ist mit Kundenkommunikation im Geschäft (Beispiel Fielmanns übliche handschriftliche Preis-Notiz: Billig, Mittler, Spitzenoptik). Darf ich die Zettel im Laden fotografieren und die Bilder veröffentlichen?--85.4.233.141 21:10, 11. Jan. 2016 (CET)
Im Laden herrscht Hausrecht. Man kann Dir das Fotografieren dort untersagen. Es heimlich zu tun, kann zum Hausverbot führen. --Blutgretchen (Diskussion) 21:27, 11. Jan. 2016 (CET)
Das wäre dann schon zu spät. :) --85.4.233.141 23:48, 11. Jan. 2016 (CET)

Historische und wohl sehr schwierige Musikfrage

Gitarr(enkompon)ist Kaspar Joseph Märtz, der sonst in Wien tätig war, soll in der Saison 1841/1842 ein Konzert in Berlin gegeben haben, eventuell im Opernhaus (der heutigen Lindenoper), also irgendwann zwischen Herbst 1841 und Frühjahr 1842. Dabei heißt Konzert nach damaligen Gepflogenheiten nicht unbedingt, dass nur er gespielt hat; vermutlich gab es im Verlauf der Veranstaltung Darbietungen mehrerer Musiker. Hat jemand nähere Informationen? Ein Datum? Eine Quelle gar? Danke, --NfdA (Diskussion) 17:49, 5. Jan. 2016 (CET)

Hier einmal grob: [https://books.google.de/books?id=84INS9GBUCcC&pg=PA3=false#v=onepage&q&f=false Selected Operatic Fantasies. --G-Michel-Hürth (Diskussion) 18:21, 5. Jan. 2016 (CET)
Mit Details zu Berlin kann ich auch nicht dienen; aber ein paar Infos zu anderen Auftritten während dieser Reise findet man schon: Dresden, Breslau, Prag. lg, --Niki.L (Diskussion) 12:20, 6. Jan. 2016 (CET)
"For the dates and locations of Mertz's known concert activity see Stempnik, “Caspar Joseph Mertz,” 92-94" heißt es in Brian Torosians biographischen Abriss [33]. Berlin war zum damaligen Zeitpunkt 1841 m.W. nicht die führende Musikstadt in Deutschland, so daß vermutlich genauere Zeit- und Ortsangaben für das angebliche Konzert nicht in der Musikpresse überliefert sind, Rosenkohl (Diskussion) 13:37, 6. Jan. 2016 (CET)
Herzlichen Dank zusammen, ich wusste ja, dass es schwierig sein konnte... Sollte noch jemand das Berliner Datum finden, immer her damit; ich schaue ab und zu mal vorbei (bin nicht jeden Tag am Netz). NfdA (Diskussion) 15:43, 7. Jan. 2016 (CET)
Allerdings werden Themen hier nach dem dritten Tag der Untätigkeit archiviert. --Speravir (Disk.) 01:45, 8. Jan. 2016 (CET)
Hm. Ein Konzert 1841/1842 im Opernhaus sollte eigentlich einen Niederschlag in der Berliner Presse (zumindest für die "gebildeten Stände") gefunden haben. Was über die Bibliotheken nicht erreichbar ist, könnte vielleicht im Archiv des Dortmunder Insituts für Zeitungsforschung zu finden sein. (Der Artikel Institut für Zeitungsforschung spiegelt leider nicht angemessen die Einzigartigkeit und Bedeutung dieser Einrichtung). --2003:45:465A:9300:9C16:4AF5:EAC:E69 23:17, 8. Jan. 2016 (CET)

Danke für den Hinweis auf das Zeitschriftenarchiv; betrachtet man die deutschen Zeitschriftenlandschaft des 19. Jahrhunderts genauer, dann merkt man allerdings, daß viele erst nach 1841 gegründet worden sind. Dies ist auch wenig überraschend, lag doch die Presselandschaft nach den Karlsbader Beschlüssen in Folge von Restauration und Zensur darnieder.

Überdies sind unter den bereits digitalisierten Jahrgängen einer Zeitschrift oft kaum die frühen Jahrgänge der ersten Jahrhunderthälfte vorhanden.

Für Ankündigungen und Rezensionen Berliner Konzerte käme die Vossische Zeitung in Frage; bisher sind 9 ausgwählte Jahrgänge digitalisiert, darunter etwa 1839 und 1848, jedoch leider nicht 1841 oder 1842. [34]

Über Josephine Plantin heißt es: "für das Fachblatt der Münchner Gittaristen hatte sie eine biographische Skizze von J. K. Metz geschrieben. ['Gf.' 1901, Heft 10 ff.]" [35]. "Gf." ist wohl die Zeitschrift Der Gitarrefreund.

Allerdings sollen sich Mertz und Plantin erst Monate nach dem Berliner Auftritt kennengelernt haben, so daß eventuell über diese Phase nur wenig in Plantins biographischer Skizze steht.

Es wäre vielleicht hilfreich, aufgrund welcher Quelle vermutet wird, daß das Konzert im Operhaus stattgefunden haben soll, Rosenkohl (Diskussion) 12:28, 10. Jan. 2016 (CET)

Nochmal besten Dank zusammen.
Ja, auch die biographische Skizze der Witwe erwähnte das Konzert in Berlin; inzwischen habe ich das Konzertdatum gefunden - und zwar tatsächlich in der Vossischen Zeitung. Dieses wunderbare Blatt, das (zumindest für jene Jahren) von den Hofnachrichten ("Prinzessin ist abgereist") über einen reichhaltigen Auslandsteil bis hin zur Verlobungsanzeige und zum Haarwuchsmittel die damalige Zeit vor uns wiederauferstehen lässt, liegt weiterhin nicht digitalisiert vor. Man kann sich nur im Westhafen die Augen verderben, indem man den Mikrofilm in weiß auf Schwarz vor sich vorbeikurbelt, bis man fündig wird. Opernhaus stimmt übrigens nicht, es war im Schauspielhaus, vermutlich dem Schinkelbau, der ja noch bis vor wenigen Jahren so hieß (heute offiziell Konzerthaus). Damit hier erledigt, NfdA (Diskussion) 16:05, 11. Jan. 2016 (CET)

nulla scientia melior musica animae harmonia

Wie lautet die Übersetzung dieser lateinischen Phrase? Woher stammt sie ursprünglich? --Thomas Binder, Berlin (Diskussion) 21:05, 9. Jan. 2016 (CET)

Keine Wissenschaft ist besser als die Musik, die Harmonie der Seele. Diese Veröffentlichung (nicht der Swoboda, sondern weiter unten) könnte eventuell darauf hindeuten, dass es sich um ein Zitat von Hildegard von Bingen handelt. Laut dem New Grove Dictionary of Music trägt auch eine Motette von Cipriano di Rore aus dem Jahr 1544 den Titel Nulla scientia melior. --Jossi (Diskussion) 21:36, 9. Jan. 2016 (CET)
Vielen Dank für die Übersetzung. Der Weg zu Hildegard von Bingen ist mir allerdings zu gewagt. Nach dieser Seite würde ich es schon in die römische Antike einordnen, wüsste aber gerne (falls ermittelbar) den genaueren Ursprung. Dort steht als Quelle "Detto segnalato da Franco C.", womit ich aber nichts anfangen kann. Der Spruch scheint in Musiker-Kreisen verbreitet. Ich finde ihn u.a. als Wahlspruch eines Ensembles hier und habe ihn in einem Konzertmitschnitt kunstvoll als Aufschrift auf einem Cembalo angebracht gesehen. --Thomas Binder, Berlin (Diskussion) 06:54, 10. Jan. 2016 (CET)
Seltsamer Satz. Ich bin gespannt, ob sich die Quelle finden lässt; ich finde sie nicht. Man weiß aber auch nicht recht, wo man zu suchen anfangen soll, denn über die Zusammenhänge Musik/Harmonie/Weltseele/menschliche Seele wird ja seit vor Plato immer wieder geschrieben, über Aristoteles, Cicero, Boethius, Isidor, die Scholastiker, die Mystiker... Wenn der Ausspruch gutes Latein ist, verstehe ich ihn nicht wirklich. Scientia ist ja nicht die objektive Fachwissenschaft, sondern mehr das Wissen als solches; Musik hingegen ist eine ars, eine techne. Mal sehen, wer Licht ins Dunkel bringen kann. Grüße Dumbox (Diskussion) 10:47, 10. Jan. 2016 (CET)
Pseudo-Latein im Titel eines Albums der Florentiner Rockband Novonada, Fundstellen ab 2012 im Internet, hier anhören. --Pp.paul.4 (Diskussion) 12:54, 10. Jan. 2016 (CET)
Hm, glaube ich nicht. Auf der ersten von mir angegebenen Quelle steht es als "lateinisches Sprichwort", wenn ich das richtig deute und wird einem "Franco C." zugeschrieben. Im Bereich der klassischen Musik scheint der Spruch verbreitet zu sein (s. mein voriger Post). --Thomas Binder, Berlin (Diskussion) 15:45, 10. Jan. 2016 (CET)
Ich habe im Netz auch mehrere Konzertberichte gefunden, in denen der Spruch als Verzierung auf Cembalodeckeln, und zwar bei Kopien historischer Instrumente, auftaucht. Novonada ist keinesfalls die Quelle, der Spruch ist älter; das zeigt schon der von mir angeführte Motettentitel, der eine Kürzung dieses Satzes hier sein dürfte. Auffällig ist, dass die ganz große Mehrzahl der Fundstellen im Internet italienischer Herkunft sind. Dieser Website würde ich allerdings keinen großen Beweiswert zuschreiben, das ist einfach eine der Internet-üblichen Sammelsuriums-Seiten aller möglichen lateinischen Sprüche ohne jede Quelle. Das schon von Dumbox monierte etwas fragwürdige Latein scheint mir eher für einen mittel- oder neulateinischen Ursprung zu sprechen. Der Hildegard-Hinweis ist mehr als schwach, da stimme ich dir zu; andererseits würde der Satz ganz hervorragend in ihre Musiktheorie passen. Leider scheinen digitalisierte lateinische Schriften Hildegards, die man durchsuchen könnte, extrem rar gesät zu sein. (Ganz zu schweigen davon, dass man bei der Websuche in einen Riesenstrudel esoterischer Geschäftemacherei mit ihrem Namen gerät, vom Hildegard-Tee bis zum Hildegard-Seminar – irgendwann habe ich entnervt das Handtuch geworfen.) --Jossi (Diskussion) 22:09, 10. Jan. 2016 (CET)

Bei der Motette des Cipriano de Rore geht es um Selbsterkenntnis, nicht um Musik: „Nulla stientia <sic> melior, nulla doctrina salubrior qua quis se ipsum cognoscit“ - Es gibt kein besseres Wissen, keine heilbringendere Lehre, als daß einer sich selbst erkennt. Klaus Müller (Theologe): Voraussetzungen, Fragen und Fluchtlinien der Gegenwartsphilosophie. Vorlesung WS 2006/07

Das dürfte auf den tractatus de interiori domo des Bernhard von Clairvaux zurückgehen, wo es im 36. Kapitel heißt: „Cum inter omnia animalia, humanum genus tum digniori forma, tum digniori potentia dignius reperiatur; nulla scientia melior est illa, qua cognoscit homo se ipsum. Relinque ergo caetera, et te ipsum discute: per te curre, et in te consiste.“ --Vsop (Diskussion) 10:45, 11. Jan. 2016 (CET)

Bitte nicht drängeln, Benutzer:Pp.paul.4! --Vsop (Diskussion) 18:26, 11. Jan. 2016 (CET)
Bitte Butter bei die Fische: Wer kann einen Beleg für den Spruch anbieten, der vor das Jahr 2010 zurückreicht? Wer kennt einen Beleg, dass es mit Hildegard von Bingen zu tun hat? Hildegards Diskographie umfasst bestimmt 1000 Einträge, aber nichts dergleichen. Der Blog-Eintrag mit dem Cembalo-Deckel vom 23. Juli 2015 schreibt, dass Giovanni Togni beim Festival Urbino Musica Antica am 20. Juli 2015 auf dem Nachbau eines Dulcken-Cembalos von 1740 gespielt hat. Der Spruch war auf dem Deckel des Cembalo-Nachbaus, das zählt nur als Beleg für 2015, nicht als Beleg für 1740, da gerade bei Cembalo-Nachbauten der Käufer das Deckel-Motto vorgibt (siehe Customer choice of Latin lid mottoes, bzw. weitere Mottos für Cembalos). Und bei einem anderen Hersteller heißt es zu einer Auswahl ähnlich verquerer Mottos: Mottos marked by † in the list below don’t appear on surviving original instruments or iconography, but were chosen specially by our customers to reflect their own particular philosophies. (Und bitte aus dem † keinen Strick drehen - es ist nicht antisemitisch gemeint.) --Pp.paul.4 (Diskussion) 11:30, 11. Jan. 2016 (CET)


Wie ich erst jetzt bemerke (sorry), liefert meine erstgenannte Quelle auch eine Übersetzung ins Italienische. Diese mit Google-Translate verdeutscht, liefert: "Keine Erkenntnis übersteigt die Musik für das Wohl der Seele." Sicher etwas holprig, aber in der Tendenz noch etwas anders als die Version von Jossi2. --Thomas Binder, Berlin (Diskussion) 16:14, 11. Jan. 2016 (CET)

harmonia kann nur (Gleichsetzungs-)Nominativ oder Ablativ sein. Für die Übersetzung des Italieners müsste es Dativ sein (harmoniae), ganz abgesehen davon, dass ich es für recht gewagt halte, „harmonia“ mit „Wohlbefinden“ (benessere) zu übersetzen. Die Übersetzung ist falsch. --Jossi (Diskussion) 16:22, 11. Jan. 2016 (CET)
Na ja, es könnte schon sein, dass "Franco C." (Wer ist das???) den Gedanken auf diese Weise latinisiert hat, mit "animae harmonia" als angebautem ablativus limitationis. Aber im Ernst, elegantes Latein ist das nicht... Grüße Dumbox (Diskussion) 16:37, 11. Jan. 2016 (CET)
Und die Aussage (freie Deutung meinerseits) "Die Musik leistet mehr für das Wohl der Seele als jede Wissenschaft" könnte im Kontext ja ganz gut passen. --Thomas Binder, Berlin (Diskussion) 16:42, 11. Jan. 2016 (CET)
Wenn das gemeint sein sollte, dann kommen wir aber weit weg von dem 2500-jährigen philosophischen Kontext dieser Begriffe und nähern uns der Welt der Entspannungs-CDs inkl. Kräutertee... ;) Grüße Dumbox (Diskussion) 16:47, 11. Jan. 2016 (CET)
Nicht unbedingt. Aus meiner Sicht ist es mehr ein Ausdruck von Hochachtung vor dieser Kunst. Aber egal - es handelt sich offenbar nicht um eines der relativ geläufigen lateinischen Zitate, die man kennen müsste. Insofern ist ja meine Ausgangsfrage mehr oder weniger beantwortet. --Thomas Binder, Berlin (Diskussion) 17:01, 11. Jan. 2016 (CET)

Kanada Einwanderung Auskunft

Hallo, also wir hatten Verwandte, die in 1930er Jahren aus dem Sudetenland nach British Columbia in Kanada ausgewandert sind. Bis zu ihrem Tod hatten wir mit ihnen noch Kontakt, sie hatten auch Kinder gehabt, mit denen haben wir leider keinen Kontakt mehr. Daher zu meiner Frage, gibt es in Kanada irgend eine Behörde, bei der man Anfragen kann, wo diese eventuell gemeldet sind? --Gruß, dersachse95You can say you to me! 21:56, 9. Jan. 2016 (CET)

Das erste was ich probieren würde, ist das Telefonbuch. Die Namen sollte man aber natürlich wissen und dann einfach macl sich höflich vorstellen und nachfragen. Bestimmte deutsche Namen sind nämlich nicht sehr häufig im Ausland, so dass man auf diese Weise die Chance hat jemand zu finden. Denke auch an die Möglichkeit, dass die den Namen etwas abgewandelt haben zwecks erleichterter Aussprache. In Canada ist man für gewöhnlich bei solchen Sachen recht offen und hilfsbereit. Falls da nichts rauskommt, müsstest du dort vorsprechen, wo das Einwohnermeldeamt ist, die können dann deine Adresse an die Kinder weitergeben, dann bleibt nur zu warten, ob die an einem Kontakt interessiert sind. Es gibt auch in jeder größeren Stadt irgendwelche Stammtische, Treffen oder Vereine von Deutschstämmigen, auch da hat man die Chancen Personen oder zumindest Namen in Erfahrung zu bringen, natürlich nur, wenn diese Leute an solchen Sachen teilgenommen haben. --Giftzwerg 88 (Diskussion) 22:25, 9. Jan. 2016 (CET)
Danke erstmal für die Infos, wir fahren im Sommer auch mal den Ort, von dem wir die letzte Adresse haben, vielleicht weiß dort irgendjemand noch was, meine Eltern waren das letzte mal 1989 dort. Also der Name ist eher Tschechisch, war ja alles Böhmen. Im Internet findet man die Todesdaten der Verwandten, aber halt auch nicht mehr. Wir werden es mal versuchen! --Gruß, dersachse95You can say you to me! 22:31, 9. Jan. 2016 (CET)
Es wäre vielleicht effizienter, eine solche Anfrage in aller Freundlichkeit an die Zeitungsredaktion zu stellen, die diesen Ort mit einer Lokalzeitung bedient. Und schaden wird es garantiert nicht. Journalisten kennen sich vor Ort am besten aus, es gibt vielleicht sogar eine schöne Story (die europäischen Wurzeln auf der Suche... oder so). Und die Tipps zum Vorgehen nach der Suche sind sicher substantieller als auf der de.wp-Auskunft zu fragen. Und wenn es schon Wikipedia-Bezug haben soll: die en.wp hat soch auch ein Auskunftssystem, oder? --2003:45:467E:DC00:D8F1:5D96:4A63:69CA 22:46, 9. Jan. 2016 (CET)
Auch eine (Internet-)Recherche in Genealogie-Dateien könnte was bringen, zB.in family-search oder auch eine Anfrage in einem genealogischen Board. Gerade die Sudeten haben da ja einiges. In den USA sind sehr viele Verstorbene über findagrave.com zu finden, einfach mal die Namen und ggf. Geburtsjahre eingeben. --Hachinger62 (Diskussion) 18:49, 10. Jan. 2016 (CET) Oh, Zwischentext nicht gelesen, aber dennoch für andere evtl. sinnvoll.
Wenn man „die Todesdaten der Verwandten“ im Internet findet, könnte man dieser Spur nachgehen, und den Eigentümer des Grabes oder den Aufgeber der Todesanzeige ermitteln. Eine „brute force“-Suche mit Canada411 könnte Adressen der Kinder liefern. Man könnte den Nacheigentümer des Hauses (falls Eigenheim) ermitteln und dort anrufen (lassen). --Pp.paul.4 (Diskussion) 15:10, 11. Jan. 2016 (CET)

Mathematische Disziplin gesucht

Es geht um Logistik: Gegeben sei ein verdichtetes Siedlungsgebiet (Kleinstadt, Stadtteil einer Großstadt o. ä.), in dem ein Lieferdienst (Post, Zeitungszustellung) die Zustellung organisieren will. Der muß dann genügend viele Zusteller beschäftigen, damit die ihr tägliches Pensum in einer gegebenen Zeit schaffen können. Dazu wird die Region in einzelne Zustellbezirke eingeteilt, die täglich von jeweils einem Zusteller bedient werden sollen.

Und die Frage ist jetzt, welche mathematische Disziplin sich mit dem Problem befaßt, die Zustellbezirke möglichst optimal zuzuschneiden, damit die Zustellung mit minimalem Personaleinsatz gewährleistet werden kann. Was gibt es denn dafür für Lösungsansätze bzw. -methoden?

(Die Randbedingungen kann man sich beliebig und einigermaßen praxisnah überlegen - Stichworte wären z. B. Bewohner bzw. Wohnungen/Briefkästen pro Zustelladresse, Weglänge zwischen einzelnen Adressen, Qualität der Zuwegungen (Pflasterung vs. "Stoppelacker"), Dichte der Wegeverbindungen, Anmarschweg vom Zustellstützpunkt usw.) (nicht signierter Beitrag von 92.228.251.119 (Diskussion) 18:24, 10. Jan. 2016 (CET))

Mathematische Planungsrechnung (siehe dort "Teilgebiete") Play It Again, SPAM (Diskussion) 18:30, 10. Jan. 2016 (CET)
Algorithmik von da geht es weiter und spezieller. (Ich nix verstehen) --G-Michel-Hürth (Diskussion) 18:36, 10. Jan. 2016 (CET)

Operations Research. 90.184.23.200 22:09, 10. Jan. 2016 (CET)

THX

Rumprobieren scheint die aussichtsreichere Vorgehensweise zu sein. (Die Sache ist natürlich mitbestimmungspflichtig. Aber der Betriebsrat hat auch keine Ahnung von Operations Research. Und läßt sich also relativ einfach dadurch austricksen, indem man ihm anbietet, er könne gerne einen besseren Vorschlag machen. Kann er eben nicht... (Und falls doch: Auch gut.)) (nicht signierter Beitrag von 92.224.73.76 (Diskussion) 22:33, 10. Jan. 2016 (CET))

Vielleicht hilft dir GraphHopper weiter. --GeorgDerReisende (Diskussion) 22:35, 10. Jan. 2016 (CET)

Wenn Du es schaffst, das aufzuteilende Gebiet auf dem PC mathematisch zu modellieren, kannst Du das Ausprobieren auch in Form einer Monte-Carlo-Simulation zufälliger Partitionierungen des aufzuteilenden Gebiets automatisieren. --Rôtkæppchen₆₈ 23:12, 10. Jan. 2016 (CET)
Optimierung (Mathematik), wahrscheinlich sogar Lineare Optimierung. --BlackEyedLion (Diskussion) 10:40, 11. Jan. 2016 (CET)
Modellierung geht eigentlich - ich habe bereits alle Straßen in Abschnitte zwischen Querstraßen oder Kreuzungen seitenweise aufgeteilt und die "Belegungsdichten" (Anzahl der Kunden) für jeden Einzelabschnitt, geographische Koordinaten ließen sich leicht ergänzen. Jetzt müßte ich mir nur noch Gütekriterien überlegen. (nicht signierter Beitrag von 92.224.159.94 (Diskussion) 14:22, 11. Jan. 2016 (CET))
Wenn English OK ist: graph theory, travelling salesman problem. de:WP Äquivalente bestehen für beide Artikel, wenngleich etwas kürzer. --Cookatoo.ergo.ZooM (Diskussion) 11:29, 11. Jan. 2016 (CET)
Vielleicht hilft Spionieren: Wie macht die Deutsche Post das denn eigentlich? (Mit Pech: Durch Ausprobieren - die seit Thurn und Taxis existierenden Zustellgebiete wurden nach und nach "gepatched", also graduell verändert, und sind wie das Eisenbahnnetz im Grunde historisch bedingt; das wäre plausibel, weil sich Siedlungsstrukturen meistens nur langsam verändern.)
Inwiefern sollte GraphHopper von Nutzen sein? Es geht nicht darum, optimale Routen in einem Gebiet zu finden, sondern die Gebietseinteilung selbst. Deshalb ist Travelling salesman auch die falsche Baustelle. (nicht signierter Beitrag von 92.224.159.94 (Diskussion) 14:22, 11. Jan. 2016 (CET))
War von mir nicht weit genug gedacht. Ich meinte dies hier: Open Door. --GeorgDerReisende (Diskussion) 16:34, 11. Jan. 2016 (CET)
@92.224.159.94, was Dir zu Deiner Modellierung noch fehlt außer den Gütekriterien ist eine Adjazenzrelation, damit Du Dein Zustellgebiet in zusammenhängende Teilgebiete aufteilst. Du müsstest also zu Deiner Aufteilung in Straßenabschnitte noch eine Liste der Nachbarabschnitte hinzufügen. --Rôtkæppchen₆₈ 18:30, 11. Jan. 2016 (CET)
Die Bezirksteile müssen nicht zwangsläufig zusammenhängen. Aber wenn sie zu weit voneinander entfernt liegen, dann werden die gesamten Weglängen zu hoch. Eine optimale Aufteilung braucht dennoch nicht aus zusammenhängenden Bezirken zu bestehen, weil dadurch die Weglängen in anderen Bezirken ggf. zunehmen. Beispiel: Eine lange Hauptstraße, zu der jeweils vorne und hinten je zwei Querstraßen, die die Hauptstraße kreuzen. Sinnvoll könnte ggf. eine Einteilung in drei Bezirke sein: Bezirk 1 ist die gesamte Hauptstraße, die anderen beiden sind je zwei Querstraßen vorne und hinten. Die letzten beiden Bezirke hängen nicht zusammen, aber die Wege zwischen den Teilstücken sind kurz. Der lange Weg entlang der Hauptstraße ist unvermeidlich, aber relativ leicht zu bewältigen: der Zusteller fängt an einem Ende auf der Straßenseite an, die mehr Zustelladressen hat, stellt bis zum Ende zu und anschließend die andere Straßenseite in Gegenrichtung. Diese Strecke kann nicht unterboten werden. Anders sieht das auf dem flachen Land aus, wo die Entfernungen zwischen den einzelnen Anwesen sehr hoch sind - mit Pech umfaßt ein Zustellbezirk nur eine Handvoll Adressen und erfordert trotzdem die gesamte Arbeitszeit eines Zustellers. (Ärgerlich für das Unternehmen: Es kann den unvermeidbaren hohen Mehraufwand dem Auftraggeber normalerweise nicht in Rechnung stellen, weil es Pauschalpreise gibt und das halt durch eine Mischkalkulation aufgefangen werden muß.)

hochrequente röntgenstrahlung,elektromagnetische frequenzen

Ich war 10jahre in haft bin seit 8monaten draußen,5jahre bedingt(21.2maßnahme).Und ich bin noch immer eingeschalten ..röntgenstrahlung,frequenzen?..,gedankenkontrolle,körperschmerzen/beeinflußung. Wie lange dauert es bis die abschalten?,Was ist das?,Wenn ich ins ausland fahre geht das weiter? Bitte um eine antwort. --178.165.129.58 13:58, 6. Jan. 2016 (CET)

Hö? Was ist die Frage? Aluhut? --Magnus (Diskussion) für Neulinge
ich würde mich da bei der Ursache nicht so konkret festlegen... meine Ärzte nennen es „Kopfschmerzsyndrom“ und Kraniomandibuläre Dysfunktion... dagegen soll man die Muskeln im Nacken und auf der Stirn und den Wangen selbst massieren... und nachts gegebenenfalls ne Aufbissschiene tragen... --Heimschützenzentrum (?) 14:19, 6. Jan. 2016 (CET)
Aluhüte helfen nicht gegen Röntgenstrahlung, D'oh! Gegen die muss es schon ein Bleihut sein, wegen der ROHS-Regeln (fuck EU!) aber inzwischen kaum noch beschaffbar, am besten der OP gräbt eine Abraumhalde um und verhüttet sein Blei selbst. Aber Vorsicht. Durch das ständige Tragen des Bleihutes kann es zu Nackenschmerzen kommen. Siehe unten! -- Janka (Diskussion) 14:46, 6. Jan. 2016 (CET)

Aber wie lange machen die das nach der haft mit mir noch? (nicht signierter Beitrag von 178.165.129.58 (Diskussion) 15:15, 6. Jan. 2016 (CET))

Versuche es mit Grundlagenwissen! Dann geht die Angst weg, die aus dem Unwissen und der damit verbundenen Verunsicherung resultiert. Wenn Du diesen Verschwörungstheorien länger unterliegst, wirst Du eigenmotiviert fremdgesteuert und ein geeigneter Kandidat zur Rückfälligkeit bleiben. Siehe Panoptismus als eine These. Es scheint als wärst Du gedanklich noch eingesperrt. --Hans Haase (有问题吗) 15:19, 6. Jan. 2016 (CET)
Es kommt nicht auf das Material, sondern auf die Masse des Hutes an. Ein dünner Osmiumhut ist also genausogut wie ein extra dicker Aluhut. --Rôtkæppchen₆₈ 15:40, 6. Jan. 2016 (CET)
Chronologie wiederhergestellt --Vsop (Diskussion) 02:54, 7. Jan. 2016 (CET)

Ja ich bin irgenwie noch gedanklich eingesperrt.Ich bin mehrere bücher durchgegangen;haarp,bewusstseins-und gedankenkontrolle,internet...ect.Es stehen 10jahre und 5wochen bis 5monate danach bis abgeschalten wird.In haft hat mir jemand gesagt 6monate nach der haft ist es vorbei.Und jetzt habe ich ein buch (internet)gelesen über ein folteropfer(ähnlich wie bei mir),daß es nach einen jahr nach der haft vorbei war.Aber leider haben die nicht abgeschalten,kann mir wer weiterhelfen? (nicht signierter Beitrag von 178.165.129.58 (Diskussion) 16:01, 6. Jan. 2016 (CET))

Meine Spekulation ist, dass du etwas hast, was ich als Laie "Verfolgungswahn" nennen würde. Du solltest mit einem Psychologen über diese Möglichkeit sprechen. --Eike (Diskussion) 16:07, 6. Jan. 2016 (CET)
Gerade gegen dieses Paranodingsbums soll doch ein Aluhut helfen. --Rôtkæppchen₆₈ 16:10, 6. Jan. 2016 (CET)

Nein ich habe keine paranoja und ein aluhut hilft nicht,habe mir schon ein rho stab mini,masterchip anhänger,ray guard mobil gekauft(hilft nicht).

Du hast einen Psychologen gefragt? --Eike (Diskussion) 16:30, 6. Jan. 2016 (CET)
Wende Dich an Dr. Gernot Hochbürder. Der kan Dir mit ziemlicher Sicherheit helfen. Das ist zwar etwas teurer, aber effektiv und effizient. --Rôtkæppchen₆₈ 16:39, 6. Jan. 2016 (CET)

Ich habe schon einen psychologen/therapeuten wegen meiner persöhnlichkeit,bin ein ehemaliger gewalttäter.Das ist eine auflage vom gericht. Nur es ist der von der haft. (nicht signierter Beitrag von 178.165.129.58 (Diskussion) 16:47, 6. Jan. 2016 (CET))

Falls du mit dem nicht klarkommst, kannst du versuchen einen anderen zu bekommen, dabei müsste aber wohl der bisherige Therapeut und auch das Gericht zustimmen. Solange die Röntgenstrahlen von außen anhalten, solltest du auf keinen Fall die Therapie abbrechen, und dich immer an alle Auflagen halten, sonst sitzt du sehr schnell wieder drinnen und Bewährung gibts dann keine mehr. Im Übrigen gilt, dass die Auskunft von Wikipedia keine Gesundheitsfragen beantworten kann und darf.--Giftzwerg 88 (Diskussion) 18:01, 6. Jan. 2016 (CET)

Bei ernsthaften Hintergrund ernsthafte Angaben: --- ? Möglich. da obiges auch möglich war! --KleinerTimmy (Diskussion) 18:10, 6. Jan. 2016 (CET)

Der Fragesteller hat nach eigenen Angaben als früherer Gewalttäter bis zur Nachsehung eines Strafrests zehn Jahre in einer Anstalt für geistig abnorme Rechtsbrecher verbracht, weil er, „ohne zurechnungsunfähig zu sein, unter dem Einfluß seiner geistigen oder seelischen Abartigkeit von höherem Grad eine Tat“ begangen hat, die mit einer ein Jahr übersteigenden Freiheitsstrafe bedroht ist, § 21 Abs. 2 StGB Österreich. Er meint, dort, aber auch in den acht Monaten seit seiner Haftentlassung Bestrahlung, Gedankenkontrolle ausgesetzt gewesen zu sein. Er fragt, wie lange das dauert, bis die abschalten, und ob er davor im Ausland sicher sei. Diese Frage ist keine „Gesundheitsfrage“, sondern wie folgt zu beantworten:
Eine heimliche Bestrahlung von Anstaltsinsassen und/oder Haftentlassenen zum Zweck von Gedankenkontrolle und Beeinflussung findet auch in Österreich nicht statt. Zum einen wäre eine solche Bestrahlung als Körperverletzung strafbar. Zum anderen sind Strahlen zu einer wirksamen Gedankenkontrolle bisher nicht bekannt. Die Beschwerden des Fragestellers müssen deshalb andere Ursache haben, die vielleicht mit der ihm gerichtlich attestierten Abartigkeit im Zusammenhang stehen. Diese Ursachen festzustellen, sollte der Fragesteller sich mit seinem Psychologen bemühen, statt auf irgendwelche Geldschneider zu hören, die ihm in betrügerischem Gewinnstreben abstruse Verschwörungstheorien und wirkungsloses Strahlenschutzspielzeug andrehen. --Vsop (Diskussion) 02:54, 7. Jan. 2016 (CET)

Das ist der 21.1 von dem du schreibst;unzurechnungsfähig,geiestig abnorm,gefährlich und möglich in der psychatrie abzusitzen! Meiner ist der 21.2;zurechnungsfähig,geistig abnorm,gefährlich und nicht möglich in der psychatrie abzusitzen! Entlassung gibt es erst wenn die gefährlichkeit gering ist bei beiden,nur beim 21.1 mit einahmen durch medikamente (flüssig oder spritze). (nicht signierter Beitrag von 178.115.130.247 (Diskussion) 12:02, 7. Jan. 2016 (CET))

Bitte sorgfältiger lesen: ich schrieb „ohne zurechnungsunfähig zu sein“, wie es wörtlich in § 21 Abs. 2 steht. --Vsop (Diskussion) 13:29, 7. Jan. 2016 (CET)

Wie auch immer: Es gibt keine Strahlung, die irgendwie aus der Ferne Gedanken beeinflussen, gar kontrollieren kann. Selbst, wenn man das wollte, wäre es nicht möglich. Rainer Z ... 20:09, 7. Jan. 2016 (CET)

Dann sind das frequenzen oder was anderes ich weiß es nicht genau was das ist.Aber ich weiß wie es wirkt,auf verschiedenen körperteilen schmerzen(abwechselnd),öfters urinieren,an den penis kratzen beim gehen,im gehirn redet er mit sich selbst und weist mich ab und zu auf personen in meiner nähe hin die auch angeschlossen sind,?körperkontrolle-gedankenempfang?(bewegungsabläufe sagt er mir voraus und die geräusche am körper).Die 10jahre haft wurden mir in meinen gedanken auch mitgeteilt,obwohl 8und 9jahre mir auch mitgeteilt wurden.Und jetzt; 6-12monate nach der haft werde ich abgeschalten.Ich habe viel schriftliches darüber gefunden;10jahre mindestens,5wochen bis 5monate danach und jetzt habe ich über ein ehemaligen gefolterten gelesen das es erst ein jahr nach der haft abgeschalten wurde (ähnlichkeiten wie bei mir). Und ich werde überwacht,bemerkbar. (nicht signierter Beitrag von 77.119.130.231 (Diskussion) 14:10, 8. Jan. 2016 (CET))

und was spricht nun gegen Kraniomandibuläre Dysfunktion und Folie à deux...? --Heimschützenzentrum (?) 08:37, 9. Jan. 2016 (CET)

Weil das zu offen passiert und ich habe mehrere gesundheitsuntersuchungen gemacht mit guten ergebnissen.

Ich denke, dass Du Dir unbedingt noch einmal eine zweite Meinung von einem anderen Psychater suchen solltest, wenn das möglich ist. Vieles spricht doch dafür, dass Dir eine innere Stimme etwas einredet und zwar so realistisch, dass Du es glaubst. Mit anderen Worten: Du wirst nicht bestraft, aber Dein Gehirn macht es Dir so realistisch vor, dass Du denkst, dass Du wirklich bestrahlt wirst. Dies deutet auf eine psychatrische Erkrankung hin. Hast Du früher mal härtere Drogen genommen? Das ist oft der Auslöser für solche Erkrankungen. Wie gesagt, versuche den Rat eines anderen Psychaters zu bekommen. 90.184.23.200 14:15, 10. Jan. 2016 (CET)

Nein ich habe keine psychatrische erkrankung.Aber ich habe früher härtere drogen genommen und hatte früher eine schwere persöhnlichkeitsstörung beim gutachten und gefährlich,einen teil dazu beigetragen haben die opfer weil sie gesagt haben das sie mich nicht kennen.Was aber nicht stimmt.Ich habe das ganze nicht gewußt beim gutachten erst später wie bush ca.2005 oder 2006 wien besuchte haben die losgelegt(und beamten,häftlinge).Ich habe geglaubt das die gas eingeschalten hatten,und hatte geglaubt das die anderen es besser verkraften als ich weil ich geschwächt war durch die drogeneinnahme früher.Dabei war ich es alleine (Elektromagnetismus...?),psychologisch kontrolliert!

Du scheinst elektrosensibel zu sein. Hast Du schon einmal Urlaub an einem elektrofeldarmen Ort wie Eisenschmitt gemacht? --Rôtkæppchen₆₈ 19:36, 10. Jan. 2016 (CET)

Nein war ich noch nicht(Eisenschmidt).Aber wenn das doch stimmen würde was ich schreibe in welches land müsste ich fahren das das abgeschalten wird (bin aus österreich). Nein war ich noch nicht(Eisenschmidt).Aber wenn das doch stimmen würde was ich schreibe in welches land müsste ich fahren das das abgeschalten wird (bin aus österreich).

http://www.prepaid-flat.net/eisenschmitt-fehlender-mobilfunk-fuehrt-zur-abwanderung/ Nach Aluhut und http://www.hochbuerder.org/ findet user:Rotkaeppchen68 es nun also amüsant und zu verantworten, dem Fragesteller „Urlaub im Funkloch“ vorzuschlagen. Gelegenheit dazu bietet indes auch Österreich mit seinen Bergen zuhauf: https://www.netztest.at/de/Karte. Nur wird das dem Fragesteller leider genau so wenig helfen wie „rho stab mini,masterchip anhänger,ray guard mobil“. Wie viel Zeit will er denn noch vertun, bevor er sein Misstrauen gegenüber dem ihm zugeteilten Psychologen endlich aufgibt? --Vsop (Diskussion) 21:38, 11. Jan. 2016 (CET)
Kommt ein Paranoiker in eine Buchhhandlung und sagt zum Verkäufer: "Ich suche etwas über Verfolgungswahn." Darauf der Buchhändler: "Sie stehen hinter Ihnen!"--Astra66 (Diskussion) 10:46, 12. Jan. 2016 (CET)

Der ist nun mal von der haft,da gibt es keine hilfe sondern psychatrie,zurück in haft oder mehr elektrizität.Ich bin frei und kann mir nicht weiterhelfen in dieser Sache außer abwarten das abgeschalten wird. (nicht signierter Beitrag von 91.141.1.1 (Diskussion) 11:39, 12. Jan. 2016 (CET))

Wortfindung Missbrauch öffentlichen Parkraums zu Werbezwecken

Missbrauch öffentlichen Parkraums zu Werbezwecken

Wie nennt man dieses Phänomen? Gibt es dazu einen Begriff?

http://www.express.de/ordnungsamt-machtlos-werbung-blockiert-500-parkplaetze-5175880 --Tankwart (Diskussion) 16:39, 9. Jan. 2016 (CET)

"Rollende Werbung", "Abstellen eines Werbezwecken dienenden Kraftfahrzeuges als gebührenpflichtige Sondernutzung". [36] --Joyborg 16:44, 9. Jan. 2016 (CET)
Im Bild? Parken im Parkverbot. Dem Licht nach ist es zwischen 6-18h --87.148.75.22 18:28, 9. Jan. 2016 (CET)
Da ist kein Parkverbot, nur eingeschränktes Haltverbot. Und der Führer des Kfz ist gerade im 3. Stock und holt die Kühltruhe, die er im Smart transportieren will. Dumbox (Diskussion) 20:39, 9. Jan. 2016 (CET)
Ein mit Werbung beklebtes Fz irgendwo abstellen ist glaube ich keine Sondernutzung. In diesem Fall könnte es z. B. sein zum Ein- und Ausstieg von Personen. Wenn es länger steht, gelten da die üblichen Verkehrsregeln mit Knöllchen und Abschleppen. Ich denke genehmigungspflichtige Sondernutzung wäre es, wenn das Teil abgemeldet is und irgendwo auf öffentlicher Fläche rumsteht, so ähnlich wie eine Litfaß-Säule oder eine Plakatwand. Ähnliche Gerätschaften sind mir aber auch schon aufgefallen, die aber weit weniger fixe und laufende Kosten verursachen: Große Anhänger, die irgendwo monatelang auf öffentlichen Parkplätzen stehen, jedoch so, dass sie vom vorbeifahrenden Verkehr gut gesehen werden und für irgendeinen Puff werben. Wundert mich eh, dass es dazu noch keine Organisation gibt, die diese nutzlosen Nutzanhänger regelmäßig einer Weiternutzung in irgendeneiner ehemaligen Sowjetrepublik zuführen.--Giftzwerg 88 (Diskussion) 19:14, 9. Jan. 2016 (CET)
Es kann laut OVG Hamburg v. 20.12.1999 (Mein Link oben, fünfter Punkt) durchaus eine Sondernutzung sein: "Ob das Abstellen eines zugelassenen und betriebsbereiten Fahrzeugs im Einzelfall noch als lediglich vorübergehende Unterbrechung des fließenden Verkehrs ein dem Gemeingebrauch unterfallendes zulässiges Parken darstellt oder schon eine über den Gemeingebrauch hinausgehende Sondernutzung beinhaltet, hängt dabei u. a. davon ab, ob das Kraftfahrzeug zu einem anderen Zweck als dem der späteren Inbetriebnahme abgestellt wurde. Dies muss an Hand der Einzelumstände entschieden werden." - Wie konsequent das verfolgt wird, ist natürlich eine andere Frage. Allerdings hat die Ordnungsbehörde unseres Städtchen meinen ehemaligen Arbeitgeber aufgefordert, einen Werbeanhänger genau deshalb zu entfernen. --Joyborg 19:57, 9. Jan. 2016 (CET)
Und genau das ist der Grund, warum diese Anhänger normalerweise nicht wie oben behauptet auf öffentlichen Parkplätzen stehen, sondern vorzugsweise auf irgendwelchen privaten Grundstücken, deren Eigentümer dafür eventuell einen Obulus kriegen. Dafür müssen sie (die Anhänger meine ich jetzt) nämlich auch weder eine Zulassung noch TÜV noch sonstwas haben, was sie für die Teilnahme am öffentlichen Straßenverkehr qualifiziert - nichtmal Räder sind unbedingt nötig. Nur der Aufdruck muss gut lesbar sein. --Snevern 21:21, 9. Jan. 2016 (CET)
Gelegentlich findet man in der Werbung (dafür) und auch hier den weniger offiziellen Begriff "parkende Werbung". Play It Again, SPAM (Diskussion) 23:07, 9. Jan. 2016 (CET)
Dort, Birkenstraße 52, 40233 Düsseldorf, Koordinaten: 51.226962, 6.803893 war nach Streetview noch nicht immer Halteverbot. Unsere Städte klagen über leere Kassen. Las Vegas hat – zumindest auf dem Strip – die Straßenbeleuchtung quasi-privatisiert. NY am Time Square ebenso. --Hans Haase (有问题吗) 23:22, 9. Jan. 2016 (CET)

Aus Marketingsicht nennt man das Ambient-Marketing: Überraschende Veränderungen im direkten Lebensumfeld einer Zielgruppe durch Werbebotschaften wie z.B. Streetbranding erzeugen hier Aufmerksamkeit. Schwierig zu planen kann jedoch die perfekte Umsetzung einen enorm nachhaltigen Effekt innerhalb der Zielgruppe haben. --Graf Umarov (Diskussion) 01:20, 10. Jan. 2016 (CET)

Illegale Werbetafeln - In NRW gibt es die gesetzliche Regelung, dass über den Gemeingebrauch hinaus gehende Nutzungen von Straßen und Wegen eine genehmigungspflichtige Sondernutzung darstellen. Dies wird grundsätzlich dann angenommen, wenn eine Standzeit von 14 Tagen überschritten wird. Jetzt kann man sich denken, nach wie viel Tagen dann üblicherweise die Dinger von den Besitzern umgesetzt werden ... Hier in der Stadt war es ein Anhänger mit Angeboten für erotische Dienstleistungen mitten auf einer Autobahnüberführung, die Bürger, Lokalpresse und Verwaltung in Wallung brachte (hier ein Bericht mit Foto). Letztere gab sich zunächst machtlos, mittlerweile scheint das Ding aber nicht mehr aufgestellt zu werden. Benutzerkennung: 43067 10:23, 10. Jan. 2016 (CET)

"Aus Marketingsicht..." nennt man das wohl "Ambient-Marketing" oder "Guerilla"-Marketing. Aus Sicht der Gemeinschaft(smehrheit) wird das wohl unterschiedlich bewertet. Das das Ordungsamt machtlos ist, liegt entweder an der fehlenden rechtlichen Grundlage oder der Ermessensauslegung des Ordnungsamts. Wenn das Ordnungsmat das Ermessen "falsch" auslegt, sollte der Gesetzgeber dafür sorgen, das Eindeutigkeit besteht.--Wikiseidank (Diskussion) 11:06, 10. Jan. 2016 (CET)
Ambient Media-Marketing ist etwas anderes. Das ist der Hinweis auf die Pizzeria nebenan auf der Rückseite des Parkscheins. Play It Again, SPAM (Diskussion) 11:13, 10. Jan. 2016 (CET)

Ich kann da keinerlei Mißbrauch erkennen. Wieso wird davon ausgegangen? --Pölkkyposkisolisti 11:27, 10. Jan. 2016 (CET)

Wenn das Auto mit der Werbung nur auf dem Parkplatz steht, um Werbung zu machen und nicht benutzt wird, blockiert es einen Parkplatz.--Antemister (Diskussion) 15:17, 10. Jan. 2016 (CET)
Na und? Dafür sind Parkplätze da. Außerdem kann niemand eine Grenze ziehen. So gut wie alle Dienstwagen von Handwerkern haben Werbung. Und das ist auch gut so, so weiß man als Bauleiter, wem welches Auto gehört. Und man hat Telefonnummern immer verfügbar. Darf ich jetzt nicht mehr vor der Haustür parken, wenn ich einen Aufkleber ans Auto pappe? --Pölkkyposkisolisti 15:53, 10. Jan. 2016 (CET)
Doch, natürlich. Es geht um den Sinn des Abstellens, und der besteht beim (mit Werbung bedruckten) Handwerkerfahrzeug in erster Linie darin, den Handwerker und sein Handwerkszeug zu transportieren. Dafür sind Parkplätze da - nicht um Werbung für Werbetreibende zu ermöglichen. Die Tatsache, dass Grenzen schwer zu ziehen sind, ändert daran nichts: Häufig müssen Verwaltung, Vollzugsbeamte oder Richter von den äußeren Umständen auf die Motive des Handelnden schließen und dann entsprechende Konsequenzen ziehen - oder eben nicht. --Snevern 16:41, 10. Jan. 2016 (CET)
Wenn ein Auto ein paar Stunden am Straßenrand steht, tut es das wohl nicht für Werbung. Wenn ein abgewracktes Gefährt, bei dem man sich wundert, wie es da hingekommen ist, und sich fragt, wie es je aus eigener Kraft da wegkommen sollte, seit Monaten an derselben Stelle steht und ein Plakat trägt, das größer ist als das Gefährt selbst, ist der SInn vermutlich nicht der Transport. Klar gibt es dazwischen Zweifelsfälle, aber das ändert doch nichts daran, dass manche Parkraum für Werbung missbrauchen. --Eike (Diskussion) 16:49, 10. Jan. 2016 (CET)
Wenn ein zugelassenes und fahrbereites Auto irgendwo parkt, dann kann man nichts dagegen tun oder sagen, wieso auch? Dabei ist es unerheblich, ob das Fahrzeug irgendwie beschriftet ist oder nicht. Das ist in StVO und StVZO nicht vorgesehen. Werbung wird dann (eventuell) problematisch, wenn dadurch der Charakter eines hoheitlichen Fahrzeuges entsteht. Feuerwehrrot ist verboten, die Aufschrift "Pozilei" auf einem ehemaligen Polizeiauto jedoch nicht. Dachwerbung muß sicher angebracht sein. Alles Andere ist rechtlich vollkommen irrelevant. Ist es eine offensichtliche Schrottkarre, wird die Polizei den TÜV-Aufkleber kontrollieren und wenn der in Ordnung ist, wird sie nur etwas unternehmen, wenn ein Mangel offensichtlich ist (auslaufendes Öl, fehlende Bremsen oder so). Bei schrottreifen Fahrrädern mit Werbung wird es einfacher, weil die Verkehrssicherheit einfacher als beim Auto feststellbar ist. Die Motive, warum ich ein Auto auf gerade diesem zugelassenen Parkplatz parke, gehen niemanden etwas an. --Pölkkyposkisolisti 10:58, 11. Jan. 2016 (CET)
"Die Motive, warum ich ein Auto auf gerade diesem zugelassenen Parkplatz parke, gehen niemanden etwas an." Pölkky, auch wenn Du es nicht glauben willst: Du irrst. Siehe oben. --Joyborg 11:52, 11. Jan. 2016 (CET)
Vielleicht solltest du das den Gerichten mitteilen. [37] [38] --Eike (Diskussion) 12:05, 11. Jan. 2016 (CET)
Ja, Pölkky, du irrst. Nicht alles, was man mit einem fahrbereiten, zum Straßenverkehr zugelassenen Fahrzeug tun kann, ist auch erlaubt. Das mag daran liegen, dass es nicht nur für das Fahrzeug Vorschriften gibt, die man zu beachten hat, auch nicht nur für den Fahrer, der es bewegt, sondern auch für die Straße und den erlaubten Gebrauch derselben. So ist zum Beispiel unnützes Hin- und Herfahren innerhalb geschlossener Ortschaften verboten, wenn Andere dadurch belästigt werden (nachzulesen in § 30 Absatz 1 Satz 3 StVO). Und Parkplätze sind eben zum Parken da, und wenn ein Parkplatz als Dauerwerbefläche genutzt wird, steht er der Allgemeinheit nicht mehr zum Parken zur Verfügung. Zugegeben, die Grenzen sind fließend, und die Grenze des Zulässigen ist schwer festzustellen; ein Missbrauch ist daher nur schwer zu verhindern. Aber die Rechtslage ist dennoch so. Man sollte Dinge nicht allein deshalb erlauben, weil man sie im Einzelfall nur schwer beweisen kann. --Snevern 17:10, 11. Jan. 2016 (CET)
Ok, wieder was gelernt. Hab ich so nicht erwartet. --Pölkkyposkisolisti 19:27, 11. Jan. 2016 (CET)

Ich kenne einen Fall, wo ein Fahrrad mit angebrachtem Schild geparkt ist, das auf das gegenüberliegende Geschäft hinweist. Das steht da immer, wenn ich vorbeikomme. --Distelfinck (Diskussion) 17:14, 11. Jan. 2016 (CET) Wenn da Köln keine Handhabe hat ist das ein Internes Problem. Wahrscheinlich hat das Ordnungsamt Anweisung keine Gerichtsverfahren anzustreben oder so. Selbstverständlich kann immer geklagt werden, falls die festgelegt wird das eine Sondernutzung vorliegt. Dann muss das halt vor Gericht. Aber dort findet immer noch eine Abwägung durch den Richter statt und die sind nun mal auch nicht ganz weltfremd. Ein bisschen die Werbetafel alle 2 Wochen bewegen hilft dann auch nicht mehr. Das ist wie mit der Parkscheibe und dem Kreidestrich auf dem Reifen. Da hatte auch einer geklagt weil er ja sein auto aus der Lücke rausgefahren und dann wieder rein gefahren hatte. Und wen wurdert es der Richter hat festgestellt, dass auch wenn der Parkplatz verlassen wurde nur um direkt wieder hineinzufahren eben kein erneutes parken ist. Bauernschlau ist hat nicht alles. -- Mephisto - Disk Ich bin der Geist, der stets verneint 12:28, 12. Jan. 2016 (CET)

Kamera gesucht

Ich suche was, was es nicht geben kann - aber vielleicht etwas, was dem nahekommt? Kompaktkameras haben kleine Sensoren und kleine Objektive, deshalb kann die Qualität nie die von SLR erreichen, das weiß ich. Vor allem die winzigen Sensoren taugen nicht viel. Nin gibt es bei Bridge aber Modelle, die (z. B.) APS-C-Sensoren haben, die auch in Spiegelreflex verbaut werden - so wie Fujifilm X-Serie. Beim Stöbern auf Amazon finde ich jedoch seltenst Angaben über den Sensor, dort wird nur mit Megapixeln geprotzt, die aber egal sind. Gibt es Kompaktmodelle mit größeren Sensoren, wie sucht und findet man die? Mein Traummodell wäre: APS-C-Sensor oder etwas kleiner, R6-Batterien, lichtstarkes Objektiv. --Pölkkyposkisolisti 00:10, 10. Jan. 2016 (CET)

Wenn Du sie kühlst, rauscht der Bildsensor weniger. Sonst siehe http://www.dpreview.com/ --Hans Haase (有问题吗) 00:18, 10. Jan. 2016 (CET)
Klar, am Nordpol kann man jede Knipse nehmen - aber da macht man selten Urlaub. Soll für meine Frau sein. Und Leica D-Lux ist dann aber doch ein wenig außerhalb meiner finanzieller Vorstellungen :( --Pölkkyposkisolisti 00:20, 10. Jan. 2016 (CET)
http://www.dpreview.com/reviews?category=compacts ← die Kompakten sind dort. Für einen attraktiven Preis musst Du wohl 2 bis 3 Jahre alte Modelle heranziehen. Die Akkus gibt es da nachzukaufen. Canon PowerShot SD 4000 IS hat eine kleine Linse aber einen relativ großen Sensor und kann man sich im Urlaub auch mal stehlen lassen. Beides größer hat die PowerShot SX260 HS. Von den Bilder hat mich die Panasonic Lumix DMC-FZ150 nicht überzeucgt, aber sie kann wohl für einen Foto relativ gut HD-Videos mit 50 fps filmen und ist fernauslösbar. Sony Cyber-shot DSC-HX200V ist etwas größer aber im Schärfenvergleich mE nicht besser. Mit der Canon PowerShot G1 X (Mark II) ist Dein Wunsch nach Qualität und Preis vllt vereinbar. --Hans Haase (有问题吗) 01:30, 10. Jan. 2016 (CET)
Etwas kleiner als APS-C wäre der Four-Thirds-Sensor. Ich persönlich bin vernarrt in die Olympus-OM-D-Serie, die stehen APS-C in der Bildqualität in absolut nichts nach, sind handlich und durchdacht und bieten vor allem eine breite Palette guter lichtstarker Optiken. Wenn OM-D schon zu groß ist, schau dir die PENs an – gleiches System ohne festen Sucher, daher kleiner. Würde ich heute was Kompaktes kaufen wollen, wäre es eine PEN plus das M.Zuiko Pancake 14–42. In der Größenklasse dürfte es kaum was Besseres geben. Aber Kameras, die mit AA-Zellen laufen, findest du da wahrscheinlich nicht. Die werden auf Kleinheit getrimmt, da bietet nur LiIon ein brauchbares Ladungs-Volumen-Verhältnis. Bei Reisekameras ist mir wichtig, daß ich sie per USB in der Kamera laden kann und nicht noch ein extra Ladegerät mitschleppen muß.
Aber für Kameras gilt heute: Richtig schlechte gibt es nicht, es kommt auf a) den Einsatzzweck und b) die subjektive Bedienbarkeit für dich an. Die Kamera, mit der du am besten klarkommst, ist die beste für dich. --Kreuzschnabel 07:45, 10. Jan. 2016 (CET)
Wie findet man sowas? Hier gewünschte Sensorgröße(n), "kompakte" Länge/Breite/Höhe, gewünschte Lichtstärke, gewünschten Preis eingrenzen und kucken, was übrigbleibt. --Eike (Diskussion) 11:36, 10. Jan. 2016 (CET)
Guter Vergleich, die gefundenen Modelle kann man dann mit den Beispielbildern des Falschenetiketts beim DPReview (oben verlinkt) vergleichen. --Hans Haase (有问题吗) 17:44, 10. Jan. 2016 (CET)
Danke, dieser Vergleich ist sehr hilfreich. --Pölkkyposkisolisti 08:56, 11. Jan. 2016 (CET)
Dieser Vergleich sagt dir, wie scharf bzw. gut aufgelöst die Bilder sind, die die Kameras unter den dort gegebenen Studiobedingungen liefern. Du willst aber eine Kamera, die unter den von dir gesetzten Bedingungen gute Bilder liefert (und hohe Auflösung und Schärfe sind nur kleine Teilaspekte von „gut“). Da spielen sehr viele Faktoren mit rein – was nützt einem z.B. bei der Bühnenfotografie, wo alles auf den richtigen Moment ankommt, eine Kamera, die man vom Auge absetzen muß, um wichtige Einstellungen zu ändern oder zu überprüfen? Was nutzt einem in der Porträtfotografie ein 20-fach-Telezoom, das zwar jede Pore phantastisch nachzeichnet, aber eine total häßliche Unschärfe in den Hintergrund zeichnet? Was nutzen einem in der Landschaftsfotografie Sensoren mit 36 Megapixel Auflösung, wenn das Rauschen von einer Software weggeschmiert werden muß, was bei 12 oder 18 Megapixel nicht nötig gewesen wäre, weil größere Pixel einfach mehr Rauschabstand bieten? Daher immer mein Hinweis: Es gibt keine absolut beste Kameraausrüstung, aber es gibt eine beste Kameraausrüstung für das, was du damit machen willst. Und da hilft meist nur: ausprobieren. --Kreuzschnabel 08:02, 12. Jan. 2016 (CET)

Das Ultimative zurzeit soll ja die Sony Cyber-shot DSC-RX100 IV sein mit einem 1 Zoll Exmor RS CMOS Sensor (keine Ahnung) (1000 Euro) (Beispielbilder). Daneben gibt es deren Vorgängermodell die Sony Cyber-shot DSC-RX100 III (Beispielbilder), die dem Nachfolgemodell fotografisch eigentlich nicht nachstehen soll (wohl so 650 Euro), ebenso die anderen Vorgängermodelle II und I sollen sehr gut sein. Denke ich auf jeden Fall mal einen Blick wert. --87.140.194.4 03:38, 11. Jan. 2016 (CET)

Asylbewerberleistung

Was noch völlig im relevanten Artikel Asylbewerberleistungsgesetz fehlt, ist der Fall, daß das Asylverfahren bereits vor Ablauf der 15-Monats-Frist positiv beschieden wird. Werden dann die Leistungen automatisch von Asylbewerberleistung auf das höhere ALG II umgestellt (um arbeiten zu dürfen, kann ja erst nach 15 Monaten überhaupt erst eine Erlaubnis beantragt werden, die jedoch aufgrund von § 39 AufenthG wegen Vorrangigkeit von Arbeit für deutsche Staatsbürger grundsätzlich abgelehnt werden kann), oder ist die 15-Monats-Frist vorrangig? --79.242.220.27 21:38, 10. Jan. 2016 (CET)

Mir wäre nicht bekannt dass Asylbewerber 15 Monate lang nicht arbeiten dürfen (es gab mal eine 12-Monats-Grenze, aber die wurde vor ein paar Jahren abgeschafft). Wo hast du das her? -- Liliana 01:09, 12. Jan. 2016 (CET)
Hm. Ich lese: „Am 11. November 2014 traten im Asylverfahrensgesetz und in der Beschäftigungsverordnung Rechtsänderungen in Kraft, die den Arbeitsmarktzugang für die Personengruppen deutlich verbessert haben: Zuvor galt für Asylbewerberinnen und -bewerber ein neunmonatiges und für Personen mit einem Duldungsstatus ein einjähriges Arbeitsverbot. Dieses wurde verkürzt auf drei Monate. Auch die Residenzpflicht wurde auf drei Monate gekürzt und stattdessen einer Wohnsitzauflage ersetzt. Zudem wurde die Vorrangprüfung von vier Jahren auf 15 Monate verringert.“ (BAMF: Zugang zum Arbeitsmarkt für geflüchtete Menschen, Stand: Juni 2015) --2003:45:463D:AD00:9138:4504:180A:DBE3 02:47, 12. Jan. 2016 (CET)

Was hält son Betonrohr aus?

Hi! Also hier sagt der Onkel, dass es „283,3 Tonnen“ aushält. Das wäre dann ja (IIRC) auf der Erde eine Gewichtskraft von ≈2,8 Mega Newton. Aber: Kurz darauf ist ein Messgerät zu sehen, auf dem „283kN“ steht. Machen die ihre Versuche auf dem Mond mit nem nicht so ganz akkurat kalibrierten Messgerät? Oder hat dem Onkel n bächtig pöser Ingeniör den falschen Text souf­f­lie­rt? Thx. Bye. --Heimschützenzentrum (?) 17:40, 10. Jan. 2016 (CET)

Der Druck auf dem Prüfstand ist das eine. Eingegraben drüfte das ganz anders aussehen, da der Drück weitergegeben werden kann. Auch Hobbymechaniker machen den Fehler, durch Hohlräume durch zuspannen. --Hans Haase (有问题吗) 17:48, 10. Jan. 2016 (CET)
Links am Messgerät ist ein festgeklemmtes Kalibrierungspoti zu sehen. Es ist nicht auszuschließen, dass das Gerät nicht auf kN, sonderm auf 10 kN = 1,01972 Mp oder 1 Mp = 9,80665 kN kalibriert ist. --Rôtkæppchen₆₈ 18:02, 10. Jan. 2016 (CET)
das wär ja ne Überraschung... da hätte man ja wenigstens „kN“ durchstreichen sollen... bei sowas soll schonmal jmd ne Hand verloren haben (ich mein: bei der Manipulation dieser Sicherheitsknöpfe an Stanzen/Pressen...)... --Heimschützenzentrum (?) 18:10, 10. Jan. 2016 (CET)
Was soll denn daran nicht stimmen? 283 kN ist doch nicht viel. Was irritiert, ist die Tatsache, daß vorher die Produktion bewehrter Rohre gezeigt wird, geprüft wird ein unbewehrtes. Die 283 sind eher etwas knapp, ein Gabelstapler Gesamtmasse 13t (Tragfähigkeit 5t) hat eine zul. statische Achslast von 120kN. Brückenklasse 30/30 hat Achslast 130kN und 60/30 hat 200 kN. Da wäre dieses Rohr nicht mehr zugelassen. --Pölkkyposkisolisti 09:16, 11. Jan. 2016 (CET)
naja... 283t bringen eben auf der Erde viel mehr als 283kN... oder verstehe ich wieder 'mal was falsch? --Heimschützenzentrum (?) 12:36, 11. Jan. 2016 (CET)
Achsoooo, der Sprecher. Das ist natürlich falsch. --Pölkkyposkisolisti 15:27, 11. Jan. 2016 (CET)
ich hab denen schon letztes Jahr ne eMail geschickt, aber die haben nich geantwortet... da dachte ich, dass ich was falsch verstanden haben könnte... aber ist wohl auch egal... ich lass die dann wohl in Ruhe... --Heimschützenzentrum (?) 16:08, 11. Jan. 2016 (CET)
Es ist durchaus realistisch, daß Betonrohre dieser Dimension 300 Tonnen aushalten, dann aber bewehrt und mit hochfestem Beton, was in dem Video nicht der Fall ist. --Pölkkyposkisolisti 22:17, 12. Jan. 2016 (CET)

Mittels Duplicati verschlüsselte Daten vor Ransomware sicher?

Ich sichere meine Daten mittels Duplicati und einer AES-Verschlüsselung in einer Cloud. Nun stellte ich fest, dass das Passwort in Duplicati gespeichert wird und dies nicht geändert werden kann. Besteht dadurch die Gefahr, dass Ransomware die Daten in der Cloud ebenfalls befallen kann? --46.126.45.1 23:05, 10. Jan. 2016 (CET)

1. die Sicherheit von Advanced Encryption Standard beruht wie sooft auf Vermutungen, die man derzeit weder beweisen noch widerlegen kann... 2. hinzukommen diverse Sicherheitslücken/Schwächen... 3. in ner Cloud fügt man zu den normalen Risiken einen ganzen Sack neuer Risiken hinzu... selbst Yahoo!Mail ist/war davor nicht sicher... dafür kann man aber noch seine Daten benutzen, auch wenn einem das Haus ausgeräumt wurde... 4. Passwörter mag ich allgemein nicht, weil man sie vergessen und auch raten kann... --Heimschützenzentrum (?) 23:12, 10. Jan. 2016 (CET)
Es ist für das Risiko egal, ob deine Daten in der Cloud von dir absichtlich verschlüsselt werden oder nicht - im Zweifel würden die verschlüsselten Daten von der Ransomware halt nochmal verschlüsselt. Es wäre gut, wenn du Backups verschiedenen Alters anlegen könntest (so dass im Zweifel noch etwas von vor dem Befall da ist) und wenn die Ransomware an die alten Backups nicht so einfach rankommt, sie also zum Beispiel nicht automatisch als Laufwerk eingebunden werden. --Eike (Diskussion) 23:16, 10. Jan. 2016 (CET)

Als Laufwerk eingebunden habe ich die Cloud nicht. Ich greife nur mit Duplicati darauf zu. Die Frage ist, ob die Ransomware die Software ebenfalls "bedienen" könnte. Auslesen sollte sie das Passwort m.W. nicht können.[39] 46.126.45.1 23:39, 10. Jan. 2016 (CET)

Wenn eine Laufwerk als Laufwerk im Zugriff steht, wird es selbstverständlich von der Malware bearbeitet. Etwas anderes ist es, wenn die gesicherten Dateien nicht im Explorer erscheinen und nur über WWW oder eine spezielle Software gemanagt werden können. Inkrementelle Backups haben die Eigenschaft alte Versionen von Dateien aufzubewahren. Auf die noch nicht verschlüsselten gäbe es dann Zugriff. Die Verschlüsselten sind allenfalls exkrementelles Backup. An Deiner Stelle würde ich ein Betriebssystem benutzen und keine Laufzeit für Malware auf dem Computer installieren. --Hans Haase (有问题吗) 00:11, 11. Jan. 2016 (CET)
Bei deinem letzten Satz verstehe ich Bahnhof. 46.126.45.1 01:24, 11. Jan. 2016 (CET)
Distrowatch ist eine Übersicht über Betriebssysteme. --Hans Haase (有问题吗) 02:26, 11. Jan. 2016 (CET)
Erinnert an einen Satz, der vor 15 Jahren oder so mal im Usenet stand: „Als erstes installierst du ein Betriebssystem. Achte aber darauf, wirklich ein Betriebssystem zu installieren und kein Grafikadventure.“ --Kreuzschnabel 08:24, 11. Jan. 2016 (CET)
das kommt drauf an, wie „schlau“ die Ransomware ist, und wie sehr die Cloud mitmacht... lässt die Cloud denn verändernden Zugriff überhaupt zu? oder nur Lesen+Anhängen? --Heimschützenzentrum (?) 00:24, 11. Jan. 2016 (CET)
Die Cloud[40] arbeitet mit WebDAV. 46.126.45.1 01:24, 11. Jan. 2016 (CET)
Damit hätte die Ransomware Vollzugriff auf die Cloud, inklusive Löschen unverschlüsselter Dateien, sofern die Ransomware das WebDAV-Kennwort abgreifen konnte. --Rôtkæppchen₆₈ 01:43, 11. Jan. 2016 (CET)
Und, wenn sies denn überhaupt ?? 46.126.45.1 23:53, 12. Jan. 2016 (CET)
? „wenn“ oder „kann“? und ja: wenn Duplicati das WebDAV-Kennwort „abgreifen“ kann, dann kann es die Ransomware prinzipiell auch... z. B. wenn n Entwickler abtrünnig geworden ist... grins --Heimschützenzentrum (?) 08:45, 13. Jan. 2016 (CET)

Radon in Gebäuden und WU-Beton

Hallo, im Artikel WU-Beton vermisse ich die Information, inwieweit dieser auch das Eindringen von Radon-Strahlung verhindert/vermindert. Diese Präsentation legt das eventuell nahe. Das finde ich aber schwierig zu beurteilen, weil diese Quelle sehr stichwortartig ist und keine Zahlen nennt. Kennt jemand belastbare Quellen und kann ggf. auch den Artikel entsprechend ergänzen? --85.177.158.32 15:05, 9. Jan. 2016 (CET)

Ich halte das für Theoriefindung. Die physikalisch-chemischen Eigenschaften von Radon und Wasser (Partikelgröße, Dipolmoment, molare Masse, …) sind zu unterschiedlich, um aus der Dichtigkeit für das eine auf eine Dichtigkeit für das andere zu schließen. --Rôtkæppchen₆₈ 18:55, 9. Jan. 2016 (CET)
Auf jeden Fall geht es nicht um das Eindringen von Strahlung, sondern um das Eindringen von Radon selbst. --Digamma (Diskussion) 20:35, 9. Jan. 2016 (CET)
Wasserundurchlässig heißt noch lange nicht gasundurchlässig, WU-Beton würde sonst nicht trocknen. Die Zuschläge von WU-Beton sind vor allem gegen die kapillare Weiterleitung von Wasser gerichtet, nicht gegen Transport von Gas oder Wasserdampf. Außerdem kann je nach Zusammensetzung auch durch Beton selbst Radon freigesetzt werden. Vielleicht hat es eine gewisse verzögernde Wirkung, aber womöglich ist das auch einfach nur ein netter Versuch.--Giftzwerg 88 (Diskussion) 22:08, 9. Jan. 2016 (CET)
Vor allem hat Wasser ein Dipolmoment. Es interagiert also mit den atomaren Bestandteilen des Betons. Somit können Wassermoleküle an den Beton adsorbiert werden, was die Diffusion von Wasser durch den Beton verhindert oder verlangsamt. Radon hat kein Dipolmoment, ist chemisch fast völlig reaktionsträge und interagiert so sehr wenig mit dem Beton. Es diffundiert ungehindert durch die Gitterlücken der Beteonbestandteile. SiO2, einer der Hauptbestandteile des Betons, hat so große Lücken zwischen den Bestandteilen des Kristallgitters, dass da Radonatome locker zwischenpassen. --Rôtkæppchen₆₈ 23:31, 9. Jan. 2016 (CET)
Hier ein Papier, was sich mit der Diffusion von Radon durch Beton befasst. --Rôtkæppchen₆₈ 23:34, 9. Jan. 2016 (CET)
Das ist die falsche Frage. Die Abdichtung von erdberührenden Bauteilen erfolgt ausser in einigen wenigen Sonderfällen (Weiße Wanne ohne zusätzliche Sperren ) nicht über den Beton sondern über Horizontal- und Vertikalabsperrungen aus PVC- PIB- oder Bitumenwerkstoffen. Die sind nicht diffusionsoffen. Solche Sperreen haben einen sd-Wert von 50 - 500. 20 cm Beton hatt ca. 30 - Die Frage ist also die der Porengöße der Folien. Ein Radon Atom hat etwas 0,22 nm ein Wassermolekül 0,28 nm Graf Umarov (Diskussion) 01:46, 10. Jan. 2016 (CET)
Wassermolküle sind nur so klein in gasförmigem Zustand. Flüssig bildet Wasser über Wasserstoffbrücken Konglomerate von hunderten Molekülen, die an diversen Substanzen haften bleiben oder größere Löcher einfach verstopfen Darauf beruht z. B. das Prinzip, dass ein Filzhut kein Wasser durchlässt oder die Dichtigkeit von Goretex gegenüber Regen. Gegen Radon hilft aber beides ganz sicher gar nichts. Kunststofffolien sind z. B. je nach Material und Dicke manchen Gasen gegenüber ziemlich diffusionsoffen.--Giftzwerg 88 (Diskussion) 22:55, 11. Jan. 2016 (CET)
Zum Schutz vor Radon muß der Werkstoff nicht gasdicht sein, sondern nur einen hinreichenden Diffusionswiderstand (also "kleine Poren") bieten, weil Radon eine kurze Halbwertszeit hat und binnen Tagen nach seiner Entstehung aus Radium zerfällt. Entsprechend bietet auch eine dichte Oberflächenbeschichtung (z. B. Anstrich) eines radiumhaltigen Werkstoffs einen guten Schutz. (Die Tochterprodukte des Radons sind übrigens auch radioaktiv, aber weniger problematisch, weil nicht gasförmig.) (nicht signierter Beitrag von 92.224.159.94 (Diskussion) 14:22, 11. Jan. 2016 (CET))
Sorry, aber das ist falsch. Die Gefahr geht beim Radon gerade von den festen, hochradioaktiven Tochternukliden wie dem 218Po aus. Diese Nuklide setzen sich im Gewebe fest und schädigen es durch die unmittelbar im Gewebe entstehende Strahlung. Siehe Radonbelastung#Einfluss des Radons auf den Menschen. Die Tochternuklide des Radons sind mitnichten weniger problematisch, sondern häufiger Krebsauslöser bei Bergarbeitern, siehe Schneeberger Krankheit. --Rôtkæppchen₆₈ 22:56, 11. Jan. 2016 (CET)
BK Das Gegenteil zu deiner Vermutung ist richtig. Das Problem ist, dass auf diese Weise Uran und Thorium anfangen zu "wandern" und die Zerfallsprodukte an allen möglichen Stellen auftauchen können. Wenn Radon zerfällt, bleibt Polonium zurück, das z. B. in der Lunge durchaus einigen Schaden anrichten kann und nicht einfach wieder ausgeatmet werden kann. Man denke sich, dass man ein Radonatom einatmet und nach 20 Sekunden wieder ausatmet. Schaden = 0. Es kann auch sein, dass genau in dem Moment der Zerfall eintritt, dann habe ich einmal den Schaden durch den Radon-Zerfall und durch den anschließenden mPoloniumzerfall einen mehrfachen weiteren Schaden. Weiterhin kann das Radon auch außerhalb des Körpers in der Umgebungsluft zerfallen und ich atme die Zerfallsprodukte mit der Luft ein. Dann fällt der Schaden durch den Zerfall von Radon weg, der Schaden durch Polonium bleibt. Siehe Radon-Zerfallsprodukte.--Giftzwerg 88 (Diskussion) 23:13, 11. Jan. 2016 (CET)
Ist Begriffsstutzigkeit notwendige Bedingung zur Mitarbeit in der Wikipedia? Die Gefahr beim Radon geht davon aus, daß es gasförmig ist, sich dadurch in der Umgebungsluft ausbreitet und deswegen leicht eingeatmet werden kann. Sobald es - außerhalb des menschlichen Körpers und der Atemluft natürlich - zerfallen ist, sind die Tochterprodukte fest und damit immobil. Also besteht ein sinnvoller und wirksamer Schutz darin, die Radon-Diffusion durch Sperren zu be- oder verhindern. Und Uran und Thorium "wandern" natürlich nicht, dafür sind die Halbwertszeiten viel zu hoch. Die Quelle von Radon ist Radium, sonst nichts. Und natürlich tritt das in Baustoffen isoliert von seinen Vorprodukten auf, weil es nämlich wasserlöslich ist und deswegen aus den primären Lagerstätten ausgewaschen und anderswo angelagert wird - schließlich verwendet kein Mensch Uranerz als Baumaterial, also ist das Argument mit den Zerfallsreihen lediglich hohles Gefasel. Übrigens wird Radon nicht "einfach wieder ausgeatmet". Es lagert sich nämlich an die feuchte Lungenschleimhaut an und bleibt dort gebunden - daß man genauso viel Stickstoff wieder aus- wie einatmet liegt daran, daß Stickstoff wenig wasserlöslich und das Körpergewebe damit bereits gesättigt ist; das ist also auch eher nur ein statistisches Gleichgewicht; Sauerstoff wird hingegen zu signifikanten Anteilen vom Körper aufgenommen, die ausgeatmete Luft enthält deutlich weniger Sauerstoff als die Umgebungsluft. Radon wird hingegen aus dem Körper nicht so häufig wieder abgegeben, weil es darin vergleichsweise selten ist - die relativen Partialdruckdifferenzen zwischen Außenluft und Konzentration im Körper sind hoch (obwohl der absolute Partialdruck von Radon auch bei relativ hohen Radioaktivitätskonzentrationen immer nur im homöopathischen Bereich liegt, es sich also um unwägbare Mengen handelt).
Danke für die zahlreichen Beiträge und die rege Diskussion.
Kann man dann davon ausgehen, dass bei einer WU-Betonwanne Folgendes gilt: Wenn es überhaupt eine über die normale Luftbelastung hinausgehende aus dem Boden diffundierende Radonbelastung an einem konkreten Ort geben sollte, dann dürfte die WU-Betonwanne (mit ihren wasserundurchlässigen Folien) die Radonbelastung signifikant mindern, selbst wenn einzelne Radonatome den Weg durch einzelne Poren finden. Oder ist es so, dass (angesichts der geringen Größe) die eventuelle Belastung vermutlich genauso groß wäre? --78.54.142.169 21:38, 14. Jan. 2016 (CET)
Schon mal darüber nachgedacht, woher das extrem seltene Radium kommt? Es stammt aus einer Zerfallsreihe, aus langlebigen radioaktiven Isotopen. Uran und Thorium selbst oder deren Mineralien wandern natürlich nicht, aber sobald das Uran oder Thoriumatom zerfällt und sich über Radium in Radon verwandelt hat, kann es wandern. Wenn gasförmiges Radon zerfällt und zu Polonium wird, verschwindet es nicht automatisch aus der Luft. Es beleibt dort noch längere Zeit, bindet sich irgendwann an Staub oder an Aerosole, bis es sich irgendwann irgendwo niederschlägt (oder von dort mit dem Staub wieder aufgewirbelt wird). Es sind ja ultrafeine einzelne Atome, keine wesentlich gröberen Staub oder Feinstaubpartikel. Als Radonquellen in den Baumaterialien kommt keiner auf die Idee Uranerz zu verwenden, jedoch enthalten viele verbreitet vorkommende Gesteine immer eine gewisse Menge an Uran oder Thorium wie z. B. Granit oder Monazitsand, abhängig von den geologischen Verhältnissen. Somit hast du immer wenn Granitschotter oder Monazitsand im Beton enthalten ist, eine Quelle für Radon im Beton und somit kann der Beton selbst die Quelle für Radon sein. Kein Mensch prüft solche Massengesteine auf Radioaktivität, geschweige denn macht man permanent Analysen über den Uran- oder Thoriumgehalt von Sand, wozu auch, man weiß dass diese Gesteine eine bestimmte natürliche radioaktive Strahlung haben. Radon kann auch aus tieferen Schichten Verwerfungen und Klüfte aufsteigen, so dass an einzelnen Stellen im Boden die Konzentration signifikant erhöht sein kann. Das Radon kann so aus einer Gesteinschicht stammen, die nicht an der Oberfläche zu finden ist. --Giftzwerg 88 (Diskussion) 17:24, 16. Jan. 2016 (CET)
Man kann es auch deutlich sagen: Wer in bestimmten Gebiten so richtig "urig" mit "gesunden" Naturmaterialien, sprich Granit, gebaut hat, hat eine höhere Radonbelastung als wenn er mit Holz, Kalksandstein oder Beton gebaut hätte. --84.135.154.99 10:01, 18. Jan. 2016 (CET)
Danke. Es geht (mir zumindest) aber nicht um die Baumaterialien. Das ist überwiegend Beton. Es geht um die natürliche Belastung im/aus dem Boden. Nochmal die Frage: Mindert der WU-Beton diese vermutlich auch dann um einen gewissen Wert, wenn diese Moleküle durch die Poren der Folie passen würden oder gibt es 0 Minderung? Kann man eventuell Rückschlüsse aus dem Verhalten anderer Materialien ziehen? --85.176.189.49 17:34, 18. Jan. 2016 (CET)